Gastrointestinal System/Adomen/Abdomen Wall Flashcards by Lubi Mupwaya (2024)

1

Q

In a small town in Sydney, suddenly a number of people fell sick with bloody diarrhea, severe abdominal pain, and oliguria. In the Emergency Department, most of them are severely dehydrated and confused. The panel of doctors recommended emergency blood tests. Which one of the following, if present on test results, is diagnostic of the condition?

A. Thrombocytopenia.
B. Microangiopathic hemolytic anemia.
C. High creatinine levels.
D. Severe ADAMTS 13 deficiency.
E. High LDH levels.

A

B. Microangiopathic hemolytic anemia.

  1. Classic Presentations:
    • HUS: Renal failure (oliguria), abdominal pain following invasive diarrhea.
    • TTP: Similar symptoms to HUS but with additional CNS symptoms like confusion.
  2. Causes:
    • HUS: Often follows a diarrheal illness caused by bacteria like enterohemorrhagic E. coli O157:H7 and Shigella dysenteriae, which produce Shiga toxins.
    • TTP: Linked to a deficiency in the enzyme ADAMTS13, leading to large von Willebrand factor (VWF) multimers causing microthrombi.
  3. Bacterial Toxins:
    • Shiga toxin and Shiga-like toxin can:
      • Trigger chemokine/cytokine secretion in colonic and renal cells.
      • Activate monocytes and platelets.
      • Cause secretion of large VWF multimers in glomerular endothelial cells.
  4. Clinical Differentiation:
    • HUS: More severe renal involvement.
    • TTP: More common CNS involvement.
    • History of diarrheal illness often precedes HUS.
  5. Treatment Differences:
    • HUS in children: Supportive care, dialysis if needed.
    • TTP: Plasma exchange.
    • HUS in adults: Often treated with plasma exchange like TTP.
  6. Diagnosis:
    • No single test; diagnosed based on clinical presentation and presence of microangiopathic hemolytic anemia.
    • Key diagnostic signs include:
      • Anemia
      • Elevated bilirubin and LDH
      • Presence of schistocytes on a peripheral smear
  • Microangiopathic Hemolysis: Indicates either HUS or TTP.
    • Not Diagnostic Alone:
      • Thrombocytopenia: Seen in HUS, TTP, and other conditions like ITP.
      • Renal Function Tests (e.g., creatinine): High in many conditions including dehydration.
      • ADAMTS13 Test: Time-consuming, more prognostic than diagnostic, not routinely ordered.
      • LDH: Elevated in hemolysis, not specific for HUS/TTP.
  • HUS: Renal failure, abdominal pain, follows diarrhea, supportive care in children.
  • TTP: CNS symptoms, plasma exchange treatment.
  • Diagnosis: Based on clinical presentation, anemia, elevated bilirubin and LDH, and schistocytes in blood smear.

By focusing on these key points and understanding the differences and overlaps between HUS and TTP, it becomes easier to remember and apply this information clinically.

Renal failure reflected by oliguria and abdominal pain following invasive diarrhea is classic presentation of hemolytic uremic syndrome (HUS). Addition of CNS symptoms (confusion) to this constellation makes thrombotic thrombocytopenic purpura (TTP) another possibility if it is caused by ischemia because of thrombi and not sodium derangement caused by severe dehydration. The exact etiology of HUS and TTP is not clear, but the role of Shiga toxin in HUS and ADAMTS13 (a metalloproteinase) in TTP have been implicated.

HUS, and to some extent TTP, commonly occur following a diarrheal illness with enterohemorrhagic Escherichia coli O157:H7 and Shigella dysenteriae serotype I. These bacteria, in addition to causing bloody diarrhea, are capable of secreting Shiga (Shigella) and Shiga-like toxin (E-coli). These toxins can bind to certain cell membrane receptors, which, depending on the cell type, can result in:
1. Chemokine or cytokine secretion (colonic and renal epithelial cells)
2. Cellular activation (monocytes and platelets)
3. Secretion of unusually large Von Willebrand multimers (glomerular endothelial cells)

Clinical differentiation of hemolytic-uremic syndrome (HUS) and TTP can be problematic; however, central nervous system involvement is more common in TTP, and more severe renal involvement in HUS.

In HUS, an antecedent history of diarrheal illness is often present. In fact, some authors suggest a clinical classification of HUS based on the presence or absence of diarrhea.

In children, the distinction between HUS and TTP may be more important because general supportive measures (with dialysis as needed) are the standard therapy for HUS, while TTP is treated with plasma exchange. In adults, however, HUS is also often treated with plasma exchange; therefore, differentiating between HUS and TTP is not as important as it is in children.

There is not a single diagnostic test for HUS and TTP. These are diagnosed based on clinical presentation and presence of microangiopathic hemolytic anemia presenting with:
1. Anemia, elevated bilirubin and LDH (often significantly high) 2. Presence of schistocytes on peripheral smear

In this case, presence of micro-angiopathic hemolysis, is the most important finding that suggests either HUS or TTP. The distinction between the two, however, neither is possible with certainty, nor is necessary as the therapeutic approach is almost the same for both HUS and TTP in adults.

Option A: Thrombocytopenia is almost always a feature of HUS and TTP, as it is in other conditions such as ITP; therefore, it is not diagnostic.

Option C: Renal function tests, including creatinine are part of workup for suspected HUS or TTP, but not diagnostic because high creatinine levels are seen in TTP and HUS as well as dehydration and many other conditions.

Option D: ADAMTS13 is a metalloproteinase that cleaves Von Willebrand factor (VWF). Its deficiency results in circulating large multimers of VWF. Large molecules of VWF multimers by adhering circulating platelets together leads to microthrombi in the organs, ischemia, and end organ damage. Majority of patients (>90%) with acquired TTP have circulating antibodies against ADAMTS13 making them ADAMTS13 deficient; however severe ADAMTS13 deficiency is more common in sporadic forms rather than outbreaks. As the test is time-consuming and more prognostic rather than diagnostic, it is not routinely ordered. Furthermore, ADAMTS13 deficiency alone does not seem to cause TTP, and a contributing factor such as pregnancy, infection, drugs, etc. is required to trigger TTP.

Option E: Regardless of the etiology, elevated LDH is seen in hemolysis. LDH is neither sensitive, nor specific for HUS/TTP.

Thrombotic Thrombocytopenic Purpura (TTP)

How well did you know this?

1

Not at all

2

3

4

5

Perfectly

2

Q

Which one of the following is not correct regarding hemochromatosis?

A. The incidence of hemochromatosis in Australia is 1:200.
B. C282Y hom*ozygous accounts for approximately 90% of hemochromatosis cases in Australia.
C: The majority of patients with one copy of C282Y and H63D mutation never develop hemochromatosis.
D. Ninety percent of those with hom*ozygous C282Y will develop symptoms at some stage of their lives.
E. Carriers of only one copy of the mutated HFE gene will not be affected clinically.

A

D. Ninety percent of those with hom*ozygous C282Y will develop symptoms at some stage of their lives.

Option A: Correct - The incidence of hereditary hemochromatosis (HH) in Australia is 1 in 200-300.

Option B: Correct - Appromixately 80-90% of HH cases in Australia have hom*ozygous C282Y mutations of their HFE gene.

Option C: Correct - Most of patients with heterozygous C282Y/H63D mutations never develop clinical symptoms or will have only mild symptoms.

Option D: Incorrect - Only 28.4% of males and 1.2% of the females with C282Y mutation will develop clinically significant presentation of iron overload some time in their lives.

Gastrointestinal System/Adomen/Abdomen Wall Flashcards by Lubi Mupwaya (1)

TOPIC REVIEW
Hemochromatosis has two forms. It is either due a genetic condition, namely hereditary hemochromatosis (HH), or secondary to conditions leading to iron overload in the body such as chronic hemolysis and multiple transfusions. Secondary iron overload is referred to as hemosidrerosis.

HH is an autosomal recessive genetic disease with variable penetrance and delayed age of onset (rare before the age of 30 years), in which the body iron content exceeds enormously beyond the normal limit (20-60g compared to normal amount of 4g) due to increased absorption through gastrointestinal tract. HH is the result of a mutation in HFE gene, which is located on the short arm of chromosome 6.

The two most common mutations of HFE gene are termed C282Y and H63D. Different possible combinations of these mutations and their risk for development of clinical HH is summarized in the following table. (See photo)

hom*ozygous C282Y is the most common form of HH (80-90%), followed by heterozygous C282Y/H3D. 28.4% of males and 1.2% of the females with C282Y mutation will develop clinically significant presentation of iron overload some time in their lives, but rarely before the age of 30 years.

Those with hom*ozygous H63D are very unlikely to develop clinical disease. Heterozygosity of C282Y and H63D leads to HH with milder clinical forms.

Epidemiology
The prevalence of hemochromatosis in the Australian of Northern Europe background is 1 in 200-300 (250). Every 1 in 8 is a silent carrier of one mutated HFE gene.

Clinical disease is seen more in men than women. The age of onset for women is more advanced. The age of onset is rarely before 30 years.

Pathophysiology
HH results in deposition of iron in different body organs leading to a multi-organ/multisystem involvement and presentation.

Presentation
Hemochromatosis, through iron overload and deposition of iron in several organ systems, can present with the following features:

  • Chronic hepatitis and cirrhosis (most common cause of mortality)
  • Abdominal pain
  • Arthralgia – often MCPs and large joints, due to chondrocalcinosis and peusogout. The pain is similar to the pain associated with osteoarthritis.
  • Skin darkening (tanned skin) – deposition of iron in the skin.
  • Small testes, infertility, impotence and decreased libido –small testes are due to hypopituitarism and/or liver disease. Iron deposition in gonads occurs, but is not the cause hypogonadism.
  • Damage to the pancreas and diabetes mellitus (known as bronze diabetes).
  • Restrictive cardiomyopathy and congestive heart failure (15% of patients). Other less common cardiac manifestations are supraventricular tachycardias, atrial fibrillation, atrial flutter and varying degrees of atrioventricular block.
  • Panhypopituitarism - caused by iron deposition in the pituitary
  • Hepatocellular carcinoma (hepatoma) – in 10% of cases with liver involvement.
  • Osteoporosis (25% of cases) and osteopenia (41% of case)
  • Sparse body hair especially pubic hair (62% of patients)
  • Spoon nails (50% of cases)

Diagnosis
The transferrin saturation (ratio of serum iron to iron binding capacity) reflects increased absorption of iron, which is the underlying biological defect in HH. A fasting transferrin saturation >45% is the most sensitive test for detecting early iron overload, but not diagnostic of HH. Ferritin can be used to assess iron overload, but it is not as accurate because it is an acute phase reactant and may be elevated in response to several physiologic stresses, alcohol consumption, and liver disease. Serum ferritin is abnormal when it is >250 μg/L in pre-menopausal women and >300 μg/L in men and post-menopausal women.

If fasting transferrin saturation or serum ferritin is increased on more than one occasion, HH should be suspected, even if there are no clinical symptoms or abnormal LFTs. In this situation, the HFE gene test should be considered as the next diagnostic tool.

Although liver biopsy is the most accurate test to diagnose hemochromatosis, an MRI of the liver in conjunction with HFE gene testing for mutations are diagnostic enough to eliminate the need for liver biopsy.

NOTE - Iron studies may be normal in individuals with a genetic predisposition to HH, who have not developed iron overload. Up to 40% of hom*ozygotes have normal iron studies, which may be due to overt (blood donation) or covert (gynecological or gastrointestinal) blood loss.

How well did you know this?

1

Not at all

2

3

4

5

Perfectly

3

Q

A 42-year-old woman comes to your clinic seeking advice on screening for hemochromatosis. Her 32-year-old brother has been recently diagnosed with hereditary hemochromatosis. She has two children, aged 9 and 18 years old. Which one of the following is the best action regarding screening for hemochromatosis and assessing the chance of her children developing the disease?

A. Screen her for hemochromatosis.
B. Screen both children for hemochromatosis.
C. Screen the 18-year-old child for hemochromatosis.
D. Screening is not needed at this stage.
E. Screen her and his husband.

A

A. Screen her for hemochromatosis.

The gene involved in hereditary hemochromatosis (HH) is the HFE gene. Mutations in the HFE gene can lead to problems with iron storage, causing hemochromatosis. The two main types of HFE gene mutations are C282Y and H63D.

  • Heterozygote: Has one copy of the mutated HFE gene (either C282Y or H63D).
  • hom*ozygote: Has two copies of the C282Y mutation. (Note: Two copies of H63D mutation do not cause clinical hemochromatosis.)
  • Compound Heterozygote: Has one C282Y mutation and one H63D mutation. These individuals are usually asymptomatic or have mild symptoms.
  • HH follows an autosomal recessive pattern, meaning both parents must be carriers for their children to be at risk.
  • If both parents are carriers (heterozygous), there is a 25% chance their children will be affected by HH.
  • The woman’s brother has HH, so both parents are at least carriers of the HFE mutation.
  • She could be a heterozygote, hom*ozygote, or compound heterozygote. Genetic testing will determine her status.
  • First-degree and second-degree relatives of individuals with HH should be tested with iron studies and the HFE gene test.
  • Testing is necessary to understand the risk and decide on further steps.
  • If she does not have a faulty HFE gene, no further testing of her children is needed.
  • If she is hom*ozygous for the C282Y mutation, her children should be tested, starting with the father. If the father has no HFE mutations, the children are only carriers.
  • Testing the mother (the patient) for the HFE gene mutations is the next best step in management. This will help understand the risk for her children and determine if further testing is necessary.

The gene involved in hereditary hemochromatosis (HH) is the HFE gene. Mutations in the HFE gene can lead to problems with iron storage, causing hemochromatosis. The two main types of HFE gene mutations are C282Y and H63D.

  • Heterozygote: Has one copy of the mutated HFE gene (either C282Y or H63D).
  • hom*ozygote: Has two copies of the C282Y mutation. (Note: Two copies of H63D mutation do not cause clinical hemochromatosis.)
  • Compound Heterozygote: Has one C282Y mutation and one H63D mutation. These individuals are usually asymptomatic or have mild symptoms.
  • HH follows an autosomal recessive pattern, meaning both parents must be carriers for their children to be at risk.
  • If both parents are carriers (heterozygous), there is a 25% chance their children will be affected by HH.
  • The woman’s brother has HH, so both parents are at least carriers of the HFE mutation.
  • She could be a heterozygote, hom*ozygote, or compound heterozygote. Genetic testing will determine her status.
  • First-degree and second-degree relatives of individuals with HH should be tested with iron studies and the HFE gene test.
  • Testing is necessary to understand the risk and decide on further steps.
  • If she does not have a faulty HFE gene, no further testing of her children is needed.
  • If she is hom*ozygous for the C282Y mutation, her children should be tested, starting with the father. If the father has no HFE mutations, the children are only carriers.
  • Testing the mother (the patient) for the HFE gene mutations is the next best step in management. This will help understand the risk for her children and determine if further testing is necessary.

The gene involved in hereditary hemochromatosis (HH) is called the HFE gene. Mutations in the HFE gene can lead to impaired regulation of iron storage and clinical manifestations of hemochromatosis. There are two types of mutation in HFE gene: C282Y and H63D.

Terminology

  • Those with only one copy of the mutated HFE (either C282Y or H63D) gene are called heterozygote.
  • Those with two copies of C282Y mutation are called ‘hom*ozygote’. Since those with two copies of H63D mutation never develop clinical hemochromatosis, hom*ozygote, refers to a person with both copies of HFE with C282Y mutation only.
  • Those with one copy of HFE gene with C282Y mutation and the other with H63D mutation are called ‘compound heterozygote’. These individuals are often asymptomatic and if symptoms are present they are mild.

About 90% of people of Northern European ancestry with symptoms of HH have the C282Y mutation in both copies of their HFE gene (hom*ozygote). Two percent are compound heterozygote (see above).

Since HH follows an autosomal recessive pattern of inheritance, there is often no family history, or affected family members may appear to be scattered in generations. If both parents heterozygous (carriers for a mutation in the HFE gene), there is a 25% chance for their children to be affected and genetically predisposed to HH.

This woman’s brother is diagnosed with hereditary hemochromatosis; meaning that both their parents have been at least carriers of HFE gene mutation. She can be a heterozygote, hom*ozygote or compound heterozygote. Genetic testing will reveal that.

It is recommended that first-degree and second-degree relatives of individuals, who have HH or are hom*ozygous for the C282Y gene mutation, are tested with iron studies and the HFE gene test. Based on the recommendation she needs testing. Further steps depend on her test results:

  • If she does not have a faulty HFE gene, no further testing of the children would be indicated, because even if the father is hom*ozygote, the children would only be carriers in the worst case scenario.
  • If she is found to be hom*ozygote, then the next step would be testing the father. If the father is found to have no HFE mutations the children are not at risk of HH and could only be carriers and not susceptible to HH.

Of the options, testing the mother for HH would be the next best step in management.

How well did you know this?

1

Not at all

2

3

4

5

Perfectly

4

Q

A 35-year-old woman presents to your practice with complaint of right upper quadrant discomfort for the past 3 months. She smokes 10 cigarettes a day and drink alcohol at weekends. On examination, she is otherwise healthy, with no palpable abdominal mass or tenderness. An abdominal CT scan is arranged and obtained, which is shown in the accompanying photograph. Which one of the following is the most likely diagnosis?

A. Liver abscess.
B. Simple hepatic cyst.
C. Hepatic hemangioma.
D. Hepatocellular carcinoma.
E. Hydatid cyst.

Gastrointestinal System/Adomen/Abdomen Wall Flashcards by Lubi Mupwaya (2)

A

B. Simple hepatic cyst.

The hom*ogenous hypoattenuating (darker than the surrounding liver parenchyma) oval-shaped lesion in the photograph is characteristic of a simple hepatic cyst. Simple hepatic cysts are common benign liver lesions and have no malignant potential. They can be diagnosed on ultrasound, CT, or MRI.

Simple hepatic cysts are one of the most common liver lesions, occurring in approximately 2-7% of the population. It is slightly more common in women. Hepatic cysts are typically discovered incidentally and are almost always asymptomatic, unless they are large enough to cause symptoms (such as in this patient).

Simple hepatic cysts may be isolated or multiple and may vary from a few millimeters to several centimeters in diameter. Simple hepatic cysts are benign developmental lesions that do not communicate with the biliary tree. They can occur anywhere in the liver, but there may be a greater predilection for the right lobe of the liver.

Certain diseases are associated with multiple hepatic cysts and include:

  • Polycystic liver disease
  • Autosomal dominant polycystic kidney disease (ADPKD) - hepatic cysts may be seen in ~40% of those with ADPKD
  • Von-Hippel-Lindau disease

Findings on ultrasonography include:

  • Round or ovoid anechoic lesion (may be lobulated)
  • Well-marginated with a thin or imperceptible wall and a clearly defined back wall
  • May show posterior acoustic enhancement if large enough
  • A few septa may be possible, but no wall thickening is present
  • A small amount of layering debris is possible
  • No internal vascularity on color Doppler

On CT scan, a hepatic cysts is characterized by its hom*ogenous hypoattenuation (water attenuation). The wall is usually imperceptible, and the cyst is not enhanced after intravenous administration of contrast material.

Option A: Liver abscess is associated with fever, leukocytosis and more pronounced symptoms. They are solid and hyperattenuated on CT scan.

Option C: hemangiomas have less hom*ogenocity and well-demarcation compared to hepatic simple cysts.

Option D: Hepatocellular carcinoma (HCC) present with less demarcated hepatic lesions that are often hypoattenuated on CT and hypoechoic on ultrasonography. The radiologic findings are inconsistent with HCC as a possible diagnosis.

Option E: Hydatid cysts presents as a multiloculated cyst (daughter cysts within the main cyst)

How well did you know this?

1

Not at all

2

3

4

5

Perfectly

5

Q

Which one of the following is the most common cause of a hyperechoic mass on liver ultrasonography?

A. Hepatoma.
B. Simple cyst.
C. Hemangioma.
D. Echinococcal cyst.
E. Metastatic liver disease.

Hyperechoic: light gray on the ultrasound (air, fat, fluid)

A

C. Hemangioma.

Hemangiomas, benign proliferation of vascular tissue, are the most common cause of a hyperechoic liver mass on ultrasound.

Hepatic hemangiomas (also known as hepatic venous malformations) are benign non-neoplastic hypervascualr lesions. They are frequently diagnosed as an incidental finding on imaging in asymptomatic patients. It is very important to differentiate hemangiomas from hepatic neoplasms.

On ultrasound, they typically manifest as well-defined hyperechoic lesions; however, a small proportion (10%) are hypoechoic, which may be due to a background of hepatic steatosis, where liver parenchyma has increased echogenicity.

On CT scan, most hemangiomas are relatively well defined.

Features of typical lesions on three phasic CT scan include:

  • Noncontrast: often hypoattenuating relative to liver parenchyma
  • Arterial phase: typically show discontinuous, nodular, peripheral enhancement (small lesions may show uniform enhancement)
  • Portal venous phase: progressive peripheral enhancement with more centripetal fill in
  • Delayed phase further irregular fill in and therefore iso- or hyper-attenuating to liver parenchyma

Hepatic Hemangioma

Gastrointestinal System/Adomen/Abdomen Wall Flashcards by Lubi Mupwaya (3)

How well did you know this?

1

Not at all

2

3

4

5

Perfectly

6

Q

A 28-year-old man presents with increasing dysphagia and odynophagia. Endoscopy reveals inflamed esophagus, which easily bleeds on contact. Several biopsies are taken showing eosinophilic infiltrations on histology. Which one of the following would be the most appropriate next step in management?

A. Proton pump inhibitors.
B. Swallowed fluticasone.
C. Oral prednisolone.
D. Albendazole.
E. Helicobacter pylori eradication.

A

A. Proton pump inhibitors.

Primary eosinophilic esophagitis (EoE) is an increasingly recognized medical condition characterized clinically by symptoms related to esophageal dysfunction, and histologically by eosinophilic inflammation in the esophagus.

EoE is hypothesized to be an atopic inflammatory disease caused by an abnormal immune response to antigenic stimulation, mostly foods. Normally, eosinophils are normal component of mucosal infiltrates in all-length of the gastrointestinal tract except the esophagus. Eosinophils in the esophageal mucosa are always pathologic.

Generally, the clinical symptoms of EoE are nonspecific, and the patients are in good physical condition resulting in a delayed diagnosis (years) in some cases.

The presenting symptoms vary depending on the age of the onset:
Children - children tend to present with nausea and vomiting, weight loss, anemia, and failure to thrive. In neonates and infants, refusal of food is the most common presenting symptom.

Adults - the characteristic symptoms in adults include dysphagia for solid foods, retrosternal pain and food impaction. Some patients also present with gastroesophageal reflux disease (GERD) symptoms unresponsive to medical anti-reflux therapy. A subset of patients have been recognized to have a typical clinical presentation of EoE in the absence of GERD who show a clinicopathologic response to PPIs. This condition is currently referred to as PPI-responsive EoE.

There is no Australian guidelines for diagnosis and management of EoE and current recommendation is based on the guidelines by the American College of Gastroenterology (ACG).

According to the ACG, diagnostic criteria for EoE include all of the following:

  • Symptoms related to esophageal dysfunction.
  • ≥15 eosinophils/hpf on esophageal biopsy
  • Persistence of eosinophilia after a proton pump inhibitor (PPI) trial
  • Secondary cause of esophageal eosinophilia excluded

This patient has symptoms related to esophageal dysfunction (dysphagia and odynophagia) and established eosinophilia on histological studies. In order for EoE to be the definite diagnosis, it is necessary that eosinophilia persists after an 8-week trial of a PPI as well and other causes of eosinophilia are excluced.

The rationale behind the trial of PPI is that GERD may mimic EoE, coexist with it, or contribute to it. Conversely, EoE may contribute to GERD; therefore, the diagnosis of EoE is generally made after the symptoms persist after an 8-week course of proton pump inhibitors (PPIs) as the best initial step in management. PPIs may benefit patients with EoE either by reducing acid production in patients with co-existent GERD, or by other unknown anti-inflammatory mechanisms.

The main three components of treatment in established EoE are (1) dietary advice and alteration, (2) pharmacotherapy, and (3) surgical intervention. For pharmacological intervention topical swallowed steroids (e.g., fluticasone, budesonide) are considered the main treatment options, once the diagnosis of EoE is established either after failed PPI therapy or normal pH studies.

For patients unresponsive to the above measure, oral (systemic) predniso(lo)ne maybe indicated.

TOPIC REVIEW
Causes of esophageal eosinophilia:

  • Eosinophilic esophagitis
  • GERD
  • PPI-responsive eosinophilic esophagitis
  • Achalasia
  • Crohn’s disease
  • Parasitic infections
  • Drug hypersensitivity
  • Connective tissue disease (e.g., scleroderma, dermatomyositis)
  • Celiac disease
  • Hypereosinophilic syndrome

Eosinophilc esophagitis

How well did you know this?

1

Not at all

2

3

4

5

Perfectly

7

Q

A 70-year-old man presents with difficulty in swallowing for the past 6 months and 4 kilogram weight loss in this period. He describes that the most difficult part of swallowing for him is when he tries to start getting the food down his mouth. He had been a smoker for most of his adult life but has quit 10 years ago. Which one of the following would be the most appropriate management option at this point?

A. Endoscopy.
B. Surgery.
C. Upper series barium study.
D. Helicobacter pylori testing.
E. Manometry.

A

C. Upper series barium study.

No matter what the clues point towards, every patient with dysphagia should undergo appropriate investigation. Just because of weight loss, the patient’s cannot be told to have esophageal cancer. Although the patient’s age is a red flag for dysphagia, the fact that it occurs at initiation of swallowing makes oropharyngeal dysphagia a more likely probability. On the other hand, every patient with dysphagia, regardless of the etiology, may have weight loss due to decreased calorie intake; nonetheless, a thorough and judicious assessment should be considered for every patient with dysphagia.

The best initial step in management of dysphagia depends on provisional diagnosis based on the history and clinical findings. When esophageal cancer is suspected, evaluation starts with upper endoscopy and biopsy. With oropharyngeal and motility-related dysphagia, barium studies should come first.

In this scenario, oropharyngeal dysphagia, probably caused by a retropharyngeal pouch (Zenker’s diverticulum), is the most likely diagnosing; therefore, barium swallow would be the best initial assessment tool. If a retropharyngeal pouch is diagnosed on barium studies, endoscopy should be avoided due to the significant risk of the scope perforating the pouch.

Option A: Endoscopy is the initial investigation when cancer is suspected based on history and clinical features.

Option B: Surgery is indicated if the cause of dysphagia is found to be cancer or Zenker’s diverticulum. Achalasia unresponsive to conservative measures may eventually need surgical intervention as well.

Option D: Helicobacter pylori can cause peptic ulcer and consequently strictures of the gastric outlet (more common) or inlet (less common). Stricture at the junction of the esophagus to the stomach may cause dysphagia, but difficulty in initiation of swallowing goes against this diagnosis.

Option E: Manometry can be used once barium meal study suggests a motility disorder such as achalasia.

How well did you know this?

1

Not at all

2

3

4

5

Perfectly

8

Q

A mother of two children aged 9 and 18 years is concerned about her children developing hemochromatosis, as his husband has recently been diagnoses with the disease. Regarding screening for hemochromatosis, which one of the following is the next best in management?

A. Screen the 9-year-old boy.
B. Screen the18-year-old boy.
C. Screen both children.
D. Screen the mother.
E. No screening is needed as the disease becomes clinically evident between 30-60 years.

A

D. Screen the mother.

The gene involved in hereditary hemochromatosis (HH) is called the HFE gene. Mutations in the HFE gene can lead to impaired regulation of iron storage and clinical manifestations of hemochromatosis. There are two types of mutation in HFE gene: C282Y and H63D.

Terminology

  • Those with only one copy of the mutated HFE (either C282Y or H63D) gene are called heterozygote.
  • Those with two copies of C282Y mutation are called ‘hom*ozygote’. Since those with two copies of H63D mutation never develop clinical hemochromatosis, hom*ozygote, refers to a person with both copies of HFE with C282Y mutation only.
  • Those with one copy of HFE gene with C282Y mutation and the other with H63D mutation are called ‘compound heterozygote’. These individuals are often asymptomatic and if symptoms are present they are mild.

About 90% of people of Northern European ancestry with symptoms of HH have the C282Y mutation in both copies of their HFE gene (hom*ozygote). Two percent are compound heterozygote (see above).

Since HH follows an autosomal recessive pattern of inheritance, there is often no family history, or affected family members may appear to be scattered in generations. If both parents heterozygous (carriers for a mutation in the HFE gene), there is a 25% chance for their children to be affected and genetically predisposed to HH.

Current guidelines advise that first-degree and second-degree relatives of individuals, who have HH or are hom*ozygous for the C282Y gene mutation, are tested with iron studies and the HFE gene test. Based on this recommendation, both children should be considered for testing; however, in this case testing the mother would be more convenient and reasonable. If she does not have a faulty HFE gene, no further testing of the children would be indicated, because even if the father is hom*ozygote, the children would only be carriers in the worst case scenario.

How well did you know this?

1

Not at all

2

3

4

5

Perfectly

9

Q

The photograph is one of a barium swallow series performed in a 78-year-old man. He has presented with symptoms of 12 months duration. Which one of the following could be the most likely presenting symptom?

A. Recurrent chest infection.
B. Progressive weight loss.
C. Retrosternal burning sensation.
D. Gurgling in the neck.
E. Food regurgitation.

Gastrointestinal System/Adomen/Abdomen Wall Flashcards by Lubi Mupwaya (4)

A

E. Food regurgitation.

The photograph shows a pocket of contrast material at the root of the neck, as well as the contrast in the esophagus characteristic of retropharyngeal pouch (Zenker’s diverticulum). The condition is most commonly found in the elderly population.

Patients with Zenker’s diverticulum usually have dysphagia because the primary problem is an overactive upper esophageal sphincter, which fails to relax. Despite long-standing dysphagia, patients usually do not have significant weight loss.

When the pharyngeal pouch becomes large enough to retain contents such as mucus, pills, sputum and food, the patient may complain of pulmonary aspiration and recurrent chest infections, foul-smell breath, gurgling in the throat, appearance of a mass in the neck, or regurgitation of food into the mouth.

Of these symptoms, however, food regurgitation is the most distressing symptoms for which medical attention is usually sought.

Retrosternal burning sensation is a characteristic feature of gastro-esophageal reflux disease (GERD) and is not an associated symptom in Zenker’s diverticulum.

AMC Handbook of Multiple Choice Questions – pages 449-450

How well did you know this?

1

Not at all

2

3

4

5

Perfectly

10

Q

A 69-year-old woman is diagnosed with carcinoma of the cecum. Which one of the following is more likely to have been her initial presenting symptoms?

A. Right iliac fossa (RIF) mass.
B. Altered bowel habit.
C. Weakness and fatigue.
D. Melena.
E. Bright rectal bleeding.

A

C. Weakness and fatigue.

Colorectal cancers present with different symptoms depending on their location within the colon:

  • Right-sided tumors typically cause symptoms such as anemia and fatigue due to chronic blood loss. This is because the proximal colon (right side) has a larger lumen and can accommodate liquid feces, leading to less noticeable changes in bowel habits.
  • Left-sided tumors, including those in the rectum, commonly present with altered bowel habits (such as constipation or diarrhea) and rectal bleeding. This is because the left side of the colon has a narrower lumen, which can lead to obstruction or changes in stool consistency, causing symptoms like constipation or diarrhea.

Understanding these differences helps in recognizing and diagnosing colorectal cancers based on the presenting symptoms associated with the tumor’s location in the colon.

Colorectal cancers may present with a wide variety of symptoms. The presenting symptoms, to a great extent, depends on the location of the tumor.

A change in bowel habits is a less common presenting symptom for right-sided tumors because feces is liquid in the proximal colon and the lumen caliber is larger. Right-sided tumors present with anemia and fatigue due to chronic blood loss, while tumors of the left side are associated with altered bowel habits and rectal bleeding.

Option A: If a right-sided tumor is large enough, a right iliac fossa mass may be palpated. But a tumor that large has already caused significant symptoms for which the patient has already sought medical attention.

Option B: Altered bowel habit is more commonly seen in left-sided colon cancers, including rectal tumors.

Option D: Melena is associated with upper gastrointestinal (GI) bleeding with prolonged passage time of the blood through the gastrointestinal tract. Colorectal tumors are very unlikely to cause melena.

Option E: Rectal bleeding is often caused by a rectal cancer or more distal left-sided colon cancers.

How well did you know this?

1

Not at all

2

3

4

5

Perfectly

11

Q

Which one of the following is the most useful investigation for detection of gallstones and dilatation of the common bile duct?

A. HIDA scintigraphy.
B. Ultrasound.
C. Endoscopic retrograde cholangio pancreatography (ERCP).
D. X-ray.
E. Liver isotope scan.

A

B. Ultrasound.

Ultrasound is the most useful investigation for detection of gallstones and dilatation of common bile duct. It is also useful in detection of hepatic metastases and some liver diseases.

Option A: HIDA scan is used if sonographic studies are equivocal. On scan, the liver, CBD, and duodenum light up; the gallbladder will not if inflamed.

Option C: ERCP is used for visual detection and retrieval of stones in the CBD as well as other causes of obstructive jaundice.

Option D: Since only 10% of gallstones are radio-opaque, abdominal X-ray will be able to pick up gallstones in only up to 10% of cases; hence, not a preferred method.

Option E: Liver isotopic scan is useful for evaluation of hepatic cirrhosis.

How well did you know this?

1

Not at all

2

3

4

5

Perfectly

12

Q

A 48-year-old male presents to your practice for evaluation of his liver disease. He has the past medical history of chronic alcoholism, intravenous drug abuse and hepatic cirrhosis. Laboratory studies show deranged liver function tests. Which one of the following would be the best indicator of chronic liver disease?

A. Alkaline phosphatase.
B. Albumin.
C. Alanine amino transferase(ALT).
D. Aspartate amino transferase(AST).
E. Bilirubin.

A

B. Albumin.

Albumin is synthesized in the liver and has a half-life of around 20-22 days. Of the given options, albumin is the only indicator of chronic liver disease.

Option A: Alkaline phosphatase is NOT specific to the liver and can be elevated in the following conditions:

  • Paget’s disease
  • Fractures
  • Cholestasis (bile duct obstruction, cirrhosis, etc)
  • Malignant diseases with bony metastasis

Option C and D: Alanine aminotransferase is specific to the liver and is raised in metabolic syndrome, obesity, fatty liver and liver failure. ALT and AST are indicators of hepatocellular damage and can be elevated in both acute (e.g. viral hepatitis) and chronic liver disease. Interestingly, ALT and AST may be normal until the very last stages of chronic liver disease.

Option E: Bilirubin can be elevated in both acute and chronic liver diseases.

How well did you know this?

1

Not at all

2

3

4

5

Perfectly

13

Q

You are one of the senior residents in surgery. You are called to see Mr. Kingsley, a 67-year-old man, who has just been diagnosed with acute cholangitis. Which one of the following statements is not correct regarding the management of acute cholangitis?

A. Plan for immediate decompression if the patient does not respond to initial measures.
B. Plan for biliary decompression on semi-urgent basis (< 72 hours) if the patient is responding to initial resuscitation.
C. Plan for urgent decompression (within 24-48hrs) if the patient is older than 70 years.
D. The most appropriate method of biliary decompression is ERCP, sphincterectomy and stenting.
E. Initial aggressive resuscitation and antibiotics usually fail to get good response in majority of cases.

A

E. Initial aggressive resuscitation and antibiotics usually fail to get good response in majority of cases.

Acute ascending cholangitis is initially managed with aggressive fluid resuscitation and intravenous antibiotics followed by biliary decompression.

Since the infectious organisms responsible for acute ascending cholangitis are enteric gram negative bacteria, the selected antibiotic of choice should provide appropriate coverage against these germs.

All patients with ascending cholangitis require biliary drainage. In about 85-90% of patients, there is respond to medical therapy. In this group decompression may be performed semi-electively during the same admission (and ideally within 72 hours); however for the following patients urgent decompression may be considered:

  • Patients older than 70 years
  • Patients with diabetes
  • Patients with other comorbid conditions

Approximately 10% to 15% of patients (not the majority) fail to respond within 12 to 24 hours or deteriorate after initial medical therapy and need urgent biliary decompression. Delay to do so increases the chance of an adverse outcomes.

How well did you know this?

1

Not at all

2

3

4

5

Perfectly

14

Q

A 42-year-old man is admitted with a hematemesis. On examination he has a blood pressure of 95/60 mmHg and a pulse rate of 104 bpm. He has yellowish sclerae, scratch marks and bruises on his arms. Several spider nevi are noted on his chest and abdomen. His abdomen is distended and soft with evidence of shifting dullness. Dilated veins are easily visible in the subcutaneous tissue and the liver is palpable two finger-breadths below the costal margin. Which one of the following could be the most likely underlying cause of his problem?

A. Alcoholic liver disease.
B. Biliary cirrhosis.
C. Schistosomiasis.
D. Budd-Chiari syndrome.
E. Hepatitis C cirrhosis.

A

A. Alcoholic liver disease.

Upper gastrointestinal bleeding may have different causes, but I this man having the clinical features of liver disease and cirrhosis, bleeding from esophageal varices would be the most likely cause of bleeding. Cirrhosis results in portal vein hypertension.

When the portal vein is obstructed, shunting between the portal vein and systemic veins start to develop. This shunts lead to dilated veins around the umbilicus (caput medusa), in the lower gastrointestinal tract (hemorrhoids), or in the upper gastrointestinal tract (esophageal varices).

In Australia alcoholic liver disease is the most common cause of chronic liver disease and cirrhosis, followed by hepatitis C infection.

Biliary cirrhosis, schistosomiasis, and Budd-Chiari syndrome can all potentially lead to liver cirrhosis, but are much less common than alcoholic liver disease.

How well did you know this?

1

Not at all

2

3

4

5

Perfectly

15

Q

A 55-year-old man presents with a 6-month history of increasing dysphagia for solid foods. He has a previous history of gastroesophageal reflux for many years. He has managed his reflux with antacids but since the dysphagia started, his reflux has not been so troublesome. Which one of the following is the most likely diagnosis?

A. Esophageal cancer.
B. Achalasia.
C. Gatro-esophageal junction stricture.
D. Para-esophageal hernia.
E. Ulcerative esophagitis.

A

C. Gatro-esophageal junction stricture.

Dysphagia to solid food is more likely to be caused by mechanical obstruction due to strictures, tumors, rings or webs.

With the history of protracted gastro-esophageal reflux disease (GERD), the most likely cause of this presentation would be stricture. Inflammation and scarring of the esophagus result in stenosis of the esophagus most often at the site of junction to the stomach. Resolution of GERD symptoms supports the diagnosis: when the stricture develops, the amount of acid reflux is decreased.

Option A: Esophageal cancer is another important differential diagnosis that has to be excluded, but not the most likely diagnosis given the absence of other symptoms such as significant weight loss, anemia, etc.

Option B: Achalasia causes dysphagia to both solids and liquids. Dysphagia to solids, but not to liquids is against out achalasia as a probability diagnosis.

Option D: Para-esophageal hernias may present with GERD and GERD, over time, may result in this clinical picture. The stricture, however, is caused by the GERD, not directly by the hernia.

Option E: Ulcerative esophagitis is very rarely complicated by strictures, especially in patients with HIV; hence, an unlikely diagnosis in this HIV negative patient.

Esophageal Stricture

How well did you know this?

1

Not at all

2

3

4

5

Perfectly

16

Q

A 66-year-old man presents with intermittent right upper quadrant pain. An ultrasound, performed for revealing the cause gallstones, reveals a lesion in the liver. Triple phase CT scan is performed for more evaluation showing a 35 mm subcortical lesion with early prominent dense enhancement, which spreads through the lesion in the late portal venous phase. Which one of the following conditions would fit this description best?

A. Isolated metastatic lesion.
B. Hemangioma.
C. Hepatocellular carcinoma.
D. Hepatic cyst.
E. Hydatid cyst.

A

B. Hemangioma.

The patient has the provisional diagnosis of the biliary colic, most likely due to biliary stones. The ultrasound scan – as the best initial diagnostic tool –has been used to confirm the diagnosis. Furthermore, any associated inflammation would be evaluated. Inflammation of the gallbladder manifests as the thickening of the wall of the gallbladder and the presence of pericholecystic fluid. Stones might be seen in the common bile duct as well; however, the sensitivity of ultrasound for detection of ductal stones is low (30%-50%).

As a routine procedure, when scanning for biliary problems, the sonographer will scan the liver as well. In this case the sonographer has encountered an incidental finding, irrelevant to the presenting symptoms, for which a triphasic CT scan of the liver has been performed.

Early prominent dense enhancement of the lesion during the arterial phase is characteristic of liver hemangioma (the most common benign liver tumor). Hemangiomas are seen in approximately 20% of the general population. They may be solitary or multiple. The lesions typically show intense enhancement during the arterial phase of triphasic CT scan and retain a blush of contrast during the portal venous phase.

Option A and C: Most malignant liver tumours (primary or metastatic) are hypovascular and will not have the early enhancement during the arterial phase; rather, they become more pronounced during the portal venous phase.

Option D: Cystic lesions in the liver may be simple, multiple (polycystic liver disease), neoplastic or infective (hydatid cysts). Simple cysts are extremely common and usually asymptomatic. On imaging, these cysts have a low-density hom*ogenous appearance. With polycystic disease, the number and size of the cysts often lead to symptoms.

Option E: Hydatid cysts have a characteristic septate appearance and heterogenous appearance if they contain daughter cysts. Liver abscesses are usually symptomatic and more likely to have a heterogenous appearance.

How well did you know this?

1

Not at all

2

3

4

5

Perfectly

17

Q

You are about to perform a femoral venepuncture and you should take precaution not to damage the adjacent structures. Which one of the following is the order of structures in the groin under the inguinal ligament from medial to lateral?

A. Lacunar ligament, femoral artery, femoral vein, femoral nerve.
B. Lacunar ligament, femoral vein, femoral nerve, femoral artery.
C. Lacunar ligament, femoral vein, femoral artery, femoral nerve.
D. Femoral vein, femoral artery, femoral nerve, lacunar ligament.
E. Femoral vein. Femoral artery, lacunar ligament, femoral nerve.

Gastrointestinal System/Adomen/Abdomen Wall Flashcards by Lubi Mupwaya (5)

A

C. Lacunar ligament, femoral vein, femoral artery, femoral nerve.

Femoral triangle consists of three borders:

  • Upper border: inguinal ligament
  • Medial border: lateral border of adductor longus
  • Lateral border: medial border of sartorius

The contents of the femoral triangle from medial to lateral are:
1. Lacunar ligament and deep femoral lymph nodes
2. Femoral vein
3. Femoral artery
4. Femoral nerve

Remember LEVAN: Lymph node chain / lacunar ligament, Empty space, Vein, Artery and Nerve for the order of contents from medial to lateral.

The lacunar ligament is a ligament in the inguinal region that connects the inguinal ligament to the pectineal ligament near the point where they both insert on the pubic tubercle. This ligament comprises the medial border of the femoral canal.

Gastrointestinal System/Adomen/Abdomen Wall Flashcards by Lubi Mupwaya (6)

How well did you know this?

1

Not at all

2

3

4

5

Perfectly

18

Q

Which one of the following is the most common early complication of hemorrhagic pancreatitis?

A. Pseudocyst.
B. Infection.
C. Obstructive jaundice.
D. Pancreatic fistula.
E. Renal failure.

A

E. Renal failure.

Acute pancreatitis can be classified into acute interstitial (most common) and acute hemorrhagic (least common). In the first type, the gland architecture is preserved but is edematous. Inflammatory cells and interstitial edema are prominent within the parenchyma. In hemorrhagic type, there is marked necrosis, hemorrhage, and fat necrosis. There is marked pancreatic necrosis along with vascular inflammation and thrombosis.

Hemorrhagic pancreatitis can rapidly result in severe hemorrhage, hypovolemia, shock and acute renal failure. Other options are also potential complications of acute pancreatitis but often do not occur as early as acute renal failure.

How well did you know this?

1

Not at all

2

3

4

5

Perfectly

19

Q

A 57-year-old man presents to your practice complaining of abdominal discomfort and pain for the past 6 months. The pain is predominantly felt in the epigastric area. He does not smoke but admits to chronic alcohol use. On examination, no abdominal tenderness is elicited. The remainder of the exam is inconclusive. An abdominal ultrasound scan is arranged that reveals the presence of a 10 cm cystic lesion in the epigastric area. Which one the following is the most appropriate management option?

A. Endoscopic gastrostomy.
B. Laparotomy.
C. Percutaneous drainage.
D. Drainage through ERCP.
E. Conservative management and re-evaluation in 6 months.

A

D. Drainage through ERCP.

A pancreatic pseudocyst is a collection of pancreatic juice encased by reactive granulation tissue (not epithelial tissue) in or around the pancreas. Pseudocysts can be single or multiple, small or large, and can be located either within or outside the pancreas. Most pseudocysts communicate with the pancreatic ductal system and contain high concentrations of digestive enzymes such as amylase and lipase.

The walls of pseudocysts are formed by adjacent structures such as the stomach, transverse mesocolon, gastrocolic omentum, and pancreas. The lining of pancreatic pseudocysts consists of fibrous and granulation tissue; the lack of an epithelial lining distinguishes pseudocysts from true pancreatic cysts.

The mechanism by which a pseudocyst is formed is necrosis and liquefaction of the pancreatic necrosis of pancreatic or peripancreatic tissue.

Pseudocyst can be seen in the following situations:

  • After an episode of acute pancreatitis (in 10% of patients) - necrosis of peripancreatic tissue progresses to liquefaction and pseudocyst formation. Alternatively, a pseudocyst may result from parenchymal necrosis leading to the complete ductal disruption, and gross leakage of pancreatic juice.
  • In patients with chronic pancreatitis – pseudocysts may develop after acute attacks of pancreatitis or after the pancreatic duct is obstructed. The latter causes increased intraductal pressure and leakage of pancreatic juice.
  • After blunt or penetrating abdominal trauma (including iatrogenic injuries) - The injury can directly disrupt the duct and causes leakage.

Clinical manifestations and complications of pancreatic pseudocysts include:

  • Expansion of the pseudocyst can produce abdominal pain, duodenal or biliary obstruction, vascular occlusion, or fistula formation into adjacent viscera, the pleural space, or pericardium.
  • Spontaneous infection.
  • Digestion of an adjacent vessel can result in a pseudoaneurysm, which can produce a sudden, painful expansion of the cyst or gastrointestinal bleeding due to bleeding into the pancreatic duct.
  • Pancreatic pleural effusion -can result from disruption of the pancreatic duct with fistulization into the chest.
  • Pancreatic peritonitis - can be caused by disruption of the pancreatic duct with fistulisation to the abdomen.

NOTE - up to 40% of pseudocysts resolve without intervention; however, they can produce a wide range of clinical problems depending on the location and extent of the fluid collection and the presence of infection.

Pancreatitis pseudocysts are diagnoses with CT or ultrasound scan. Where the diagnosis is in doubt, the content can be aspirated (under endoscopic ultrasonography or CT scan) and examined.

The old rule mentioning that intervention is needed if the cysts are larger than 6 cm or persist beyond 6 weeks is no longer in use, and surgical intervention should be considered if any of the following is present:

  • Compression of large vessels (clinical symptoms or seen on CT scan)
  • Gastric or duodenal outlet obstruction
  • Stenosis of the common bile duct due to compression
  • Infected pancreatic pseudocysts
  • Hemorrhage into pancreatic pseudocyst
  • Pancreatico-pleural fistula
  • Pancreatic pseudocysts and symptoms:
    — Early satiety
    — Nausea and vomiting
    — Pain
    — Upper gastrointestinal bleeding
  • Asymptomatic pancreatic pseudocyst AND either of the following:
    — Pseudocysts > 5cm, unchanged in size and morphology for more than 6 weeks
    — Diameter > 4cm and extrapancreatic complications in patients with chronic alcoholic pancreatitis
    — Suspected malignancy

Surgical drainage is the criterion standard against which all other interventions are measured in terms of success rate, mortality and recurrence rate. In recent years, however, endoscopic drainage has been introduced and can be applied provided that the cyst is near the stomach or duodenal wall:

There are two main types of endoscopic drainage:

  • Transmural drainage: in this method, using endoscopy, a small incision is made in the stomach (endoscopic cystgastrostomy [ECG]) or in duodenum (endoscopic cystduodenostomy [ECD]) to let the pseudocyst drain into the stomach or duodenum. ECD is preferred over ECG.

NOTE - pseudocysts should have a mature capsule (wall thickness>3mm and < 1cm), bulge the lumen and have minimum size of 5-6 cm to become eligible for endoscopic drainage.

  • Transpapillary drainage: this method is safer and more effective than transmural drainage, but requires that the cyst communicates with the pancreatic duct because this method includes entering the pancreatic duct by ERCP, and from there, into the pseudocyst. Stents may be left in place to facilitate drainage.

Generally, endoscopic drainage methods are preferred over open surgical treatment if eligibility is met and there is no contraindication because these methods are less invasive and associated with fewer complications.

Laparotomy with cyst excision and internal and external drainage is still the gold standard management option; however it is ONLY considered first-line therapy for surgical intervention in the following conditions:

  • Complicated pseudocysts i.e. infected or necrotic
  • Pseudocysts associated with pancreatic duct stricture and a dilated pancreatic duct
  • Suspected cystic neoplasia
  • Presence of pseudoaneurysm, unless it has been embolised before the procedure
  • Coexistence pseudocysts and bile duct stenosis
  • Complications such as compression of the stomach or the duodenum, perforation or pseudoaneursyms

NOTE - pseudoaneurysm is an absolute contraindication to endoscopic drainage unless it is embolized prior to the procedure.

Generally, patients with symptomatic pseudocysts should undergo interventional measures for pseudocyst drainage. The procedure of choice is endoscopic drainage. This cyst is 10 cm in size (>5cm) and amenable to endoscopic drainage either by endoscopic transmural or transpapillary drainage. Transpapillary drainage has the lowest complication rate of all the mentioned procedures and is the method of choice if the pseudocyst communicates with the pancreatic duct. Fortunately, 80% of pseudocysts communicate with the pancreatic duct.

Option A: Endoscopic cystgastrostomy (ECG) or duodenostomy (ECD) are methods of choice if the pseudocyst is not communicating with the pancreatic duct.

Option B: Laparotomy and surgical removal of the cyst is considered if endoscopic methods fail or there is a contraindication.

Option C: Percutaneous catheter drainage has low success rate and high recurrent rates. It is never considered for treatment of a pancreatic pseudocyst. However, in infected pseudocysts it is the procedure of choice for sampling and examining the material as the most appropriate initial step.

Option E: Conservative management is not an appropriate option for symptomatic pseudocysts.

Pancreatic Pseudocysts

20

Q

A 34-year-old alcoholic man, who survived an episode of acute pancreatitis 4 weeks ago, has presented with mild discomfort in the epigastrium for the past few days. He is concerned that the disease might have recurred. He denies nausea and vomiting and describes the pain as constant and nagging. Physical examination is unremarkable. A contrast abdominal CT scan is performed showing a 4-cm pancreatic pseudocyst. Which one of the following is the next best step in management?

A. Endoscopic decompression.
B. Open surface decompression.
C. Observation.
D. Percutaneous catheter drainage.
E. Urgently take him to the operating room.

A

C. Observation.

A pancreatic pseudocyst is a collection of pancreatic juice encased by reactive granulation tissue (and not epithelial tissue) in or around the pancreas. Pseudocysts can be single or multiple, small or large, and can be located either within or outside of the pancreas. Most pseudocysts communicate with the pancreatic ductal system and contain high concentrations of digestive enzymes such as amylase and lipase.

The walls of pseudocysts are formed by adjacent structures such as the stomach, transverse mesocolon, gastrocolic omentum, and pancreas. The lining of a pancreatic pseudocysts consists of fibrous and granulation tissue. Lack of an epithelial lining distinguishes pseudocysts from true pancreatic cysts.

The mechanism by which a pseudocyst is formed is necrosis and liquefaction of the pancreatic necrosis of pancreatic or peripancreatic tissue.

Pseudocyst can be seen in the following situations:

  • After an episode of acute pancreatitis (in 10% of patients) - necrosis of peripancreatic tissue progresses to liquefaction and pseudocyst formation. Alternatively, a pseudocyst may result from parenchymal necrosis leading to the complete ductal disruption, and gross leakage of pancreatic juice.
  • In patients with chronic pancreatitis - pseudocysts may develop after acute attacks of pancreatitis or after the pancreatic duct is obstructed. The latter causes increased Intraductal pressure and leakage of pancreatic juice.
  • After abdominal blunt or penetrating trauma (including iatrogenic injuries such as pancreatic surgery - injury can directly disrupt the duct and causes leakage.

Clinical manifestations and complications of pancreatic pseudocysts include:

  • Expansion of the pseudocyst can produce abdominal pain, duodenal or biliary obstruction, vascular occlusion, or fistula formation into adjacent viscera, the pleural space, or pericardium.
  • Spontaneous infection.
  • Digestion of an adjacent vessel can result in a pseudoaneurysm, which can produce a sudden, painful expansion of the cyst or gastrointestinal bleeding due to bleeding into the pancreatic duct.
  • Pancreatic pleural effusion -can result from disruption of the pancreatic duct with fistulization to the chest.
  • Pancreatic pleural effusion: can be caused by disruption of the pancreatic duct with fistulization to the abdomen.

NOTE - up to 40% of pseudocysts resolve without intervention; however, they can produce a wide range of clinical problems depending on the location and extent of the fluid collection and the presence of infection.

Pancreatitis pseudocysts are diagnoses with CT or ultrasound scan. If the diagnosis is in doubt, contents can be aspirated and examined.

The dictum mentioning that intervention is needed if the cysts are larger than 6 cm or persist beyond 6 weeks is no longer in use, and surgical intervention should be considered in (even one criterion is sufficient):

  • Compression of large vessels (clinical symptoms or seen on CT scan)
  • Gastric or duodenal outlet obstruction
  • Stenosis of the common bile duct due to compression
  • Infected pancreatic pseudocysts
  • Hemorrhage into pancreatic pseudocyst
  • Pancreatico-pleural fistula
  • Pancreatic pseudocysts and symptoms:
    — Early satiety
    — Nausea and vomiting
    — Pain
    — Upper gastrointestinal bleeding
  • Asymptomatic pancreatic pseudocyst AND either of the following:
    Pseudocysts > 5cm, unchanged in size and morphology for more than 6 weeks
    — Diameter > 4cm and extrapancreatic complications in patients with chronic alcoholic pancreatitis
    — Suspected malignancy

Surgical drainage (laparotomy and internal and external drainage) is the criterion standard against which all other interventions are measured in terms of success rate, mortality and recurrence rate. In recent years however endoscopic drainage has been introduced and can be applied provided that the cyst is near the stomach or duodenal wall:

There are two main types of endoscopic drainage:

  • Transmural drainage: in this method, using endoscopy, a small incision is made in the stomach (endoscopic cystgastrostomy [ECG]) or in duodenum (endoscopic cystduodenostomy [ECD]) to let the pseudocyst drain into the stomach or duodenum. ECD is preferred over ECG.
  • Transpapillary drainage: this method is safer and more effective than transmural drainage, but requires that the cyst communicates with pancreatic duct. Using ERCP and through the pancreatic duct, the cyst is reached and poked so that its content can drain into the pancreatic duct. Stents may be used to facilitate drainage.

NOTE - pseudocysts should have a mature capsule (wall thickness>3mm and <1cm), bulge the lumen and have minimum size of 5-6 cm to become eligible for endoscopic drainage.

Generally, endoscopic drainage methods are preferred over open surgical treatment if eligibility is met and there is no contraindication, because these methods are less invasive and associated with fewer complications.

Laparotomy with cyst excision and internal and external drainage is still the gold standard management option; however it is considered first-line therapy for surgical intervention if:

  • Complicated pseudocysts i.e. infected and necrotic pseudocysts
  • Pseudocysts associated with pancreatic duct stricture and a dilated pancreatic duct
  • Suspected cystic neoplasia
  • Presence of pseudoaneurysm unless it has been embolized before the procedure
  • Coexistence pseudocysts and bile duct stenosis
  • Complications such as compression of the stomach or the duodenum, perforation or pseudoaneursyms

NOTE - pseudoaneurysm is an absolute contraindication to endoscopic drainage unless it is embolized prior to the procedure.

Generally, patients with symptomatic pseudocysts should undergo interventional measures for pseudocyst drainage. The procedure of choice is endoscopic drainage, but as mentioned before, for a pseudocyst to be amenable to this procedure, it has to be at least 5-6 in size as well as bulging into the lumen. This cyst with 4 cm in size is unlikely to be drained by endoscopic measures; therefore, open decompression (surgical drainage) should be considered; however, since the pain is mild, a watchful observation would be best management here in an attempt to avoid the high rates complications associated with surgical drainage. If the symptoms were more pronounce, intolerable or indicative of more serious complications, surgical drainage (open surface drainage) would have been the option of choice.

Percutaneous catheter drainage has low success rate and high recurrent rates. It is never considered for treatment of a pancreatic pseudocyst. However, in infected pseudocysts it is the procedure of choice for sampling and examining the material as the most appropriate initial step.

How well did you know this?

1

Not at all

2

3

4

5

Perfectly

21

Q

A 51-year-old Aboriginal male with history of acute pancreatitis presents with persistent abdominal pain and loss of appetite. On examination, an abdominal mass is found. A contrast abdominal CT scan is performed one cut of which shown in the following photograph. Which one of the following is the most likely diagnosis?

A. Chronic pancreatitis.
B. Acute pancreatitis.
C. Pancreatic pseudocyst.
D. Gastric adenocarcinoma.
E. Gastric lymphoma.

Gastrointestinal System/Adomen/Abdomen Wall Flashcards by Lubi Mupwaya (7)

A

C. Pancreatic pseudocyst.

The round lesion with hypodense hom*ogenous content and the thin smooth wall in the vicinity of and obliterating the pancreas is highly suggestive of a pancreatic cyst or pseudocyst. With the history of previous acute pancreatitis, a pseudocyst would be the most likely diagnosis.

A pancreatic pseudocyst is a collection of pancreatic juice encased by reactive granulation tissue (and not epithelial tissue) in or around the pancreas. Pseudocysts can be single or multiple, small or large, and can be located either within or outside of the pancreas. Most pseudocysts communicate with the pancreatic ductal system and contain high concentrations of digestive enzymes such as amylase and lipase.

The walls of pseudocysts are formed by adjacent structures such as the stomach, transverse mesocolon, gastrocolic omentum, and the pancreas. The lining of pancreatic pseudocysts consists of fibrous and granulation tissue. Lack of an epithelial lining distinguishes pseudocysts from true pancreatic cysts.

The mechanism by which a pseudocyst is formed is necrosis and liquefaction of the pancreatic necrosis of pancreatic or peripancreatic tissue.

Pseudocyst can be seen in the following situations:

  • After an episode of acute pancreatitis (in 10% of patients) - necrosis of peripancreatic tissue progresses to liquefaction and pseudocyst formation. Alternatively, a pseudocyst may result from parenchymal necrosis leading to the complete ductal disruption, and gross leakage of pancreatic juice.
  • In patients with chronic pancreatitis - pseudocysts may develop after acute attacks of pancreatitis or after the pancreatic duct is obstructed. The latter causes increased Intraductal pressure and leakage of pancreatic juice.
  • After abdominal blunt or penetrating trauma (including iatrogenic injuries such as pancreatic surgery - injury can directly disrupt the duct and causes leakage.

Clinical manifestations and complications of pancreatic pseudocysts include:

  • Expansion of the pseudocyst can produce abdominal pain, duodenal or biliary obstruction, vascular occlusion, or fistula formation into adjacent viscera, the pleural space, or pericardium
  • Spontaneous infection
  • Digestion of an adjacent vessel can result in a pseudoaneurysm, which can produce a sudden, painful expansion of the cyst or gastrointestinal bleeding due to bleeding into the pancreatic duct
  • Pancreatic pleural effusion -can result from disruption of the pancreatic duct with fistulisation to the chest
  • Pancreatic peritonitis - can be caused by disruption of the pancreatic duct with fistulisation to the abdomen

How well did you know this?

1

Not at all

2

3

4

5

Perfectly

22

Q

A 45-year-old man presents with signs and symptoms of gastro-esophageal reflux disease (GERD) and is started on a course of omeprazole 20 mg BID for 6 weeks with complete resolution of the symptoms. Because of long-standing symptoms, an upper endoscopy is performed. The endoscopic view of the distal esophagus is shown in the following photograph. Histopathology shows intestinal metaplasia, but no dysplasia. Which one of the following is most appropriate next step in management?

A. Pneumatic dilation.
B. Continue PPIs with doubled dose.
C. Endoscopy after 24 months.
D. Endoscopy after 6 months.
E. No intervention is needed.

Gastrointestinal System/Adomen/Abdomen Wall Flashcards by Lubi Mupwaya (8)

A

C. Endoscopy after 24 months.

The photograph shows Barrett’s esophagus. Barrett’s esophagus refers to metaplasia of the squamous epithelium of the distal esophagus to columnar epithelium of the intestinal type. The metaplasia is believed to be due to GERD in a susceptible individual.

Barrett’s esophagus is often associate with severe reflux disease, esophagitis, or strictures, but is also seen in asymptomatic or minimally symptomatic patients.

Barrett’s esophagus is a premalignant condition with a 40- to 125-fold increase in the incidence of adenocarcinoma of the lower esophagus; however, the absolute risk is relatively small and approximately 0.5% per year.

Every patient with the alarming symptoms of weight loss, anemia, heme-positive stool, and dysphagia, or GERD for more than 5 years should undergo endoscopy and biopsy. Further management depends on the histological findings and is summarized in the following:
ENDOSCOPIC FINDINGS - MANAGEMENT

  • Barrett’s esophagus (metaplasia) - Repeat endoscopy every 2-3 years for lesions ≥ 3cm and every 3-5 years for lesions < 3 cm.
  • Low-grade dysplasia - Repeat endoscopy in 6 months.
  • High grade dysplasia - Distal esophagetomy.

This patient has metaplastic changes involving an area of more than 3cm; therefore, the next step in management would be repeating the endoscopy in 2-3 years.

Option A: Pneumatic dilation is not a management option for GERD or Barrett’s esophagus.

Option B: Although proton pump inhibitors (PPIs) may be associated with the appearance of squamous islands and possibly some degree of regression, there is no evidence that PPIs reduce the incidence of esophageal cancer in patient’s with Barrett’s esophagus.

Option D: Endoscopy after 3-6 months (current recommendations: every 6 months) was the choice of management if low-grade dysplasia (not metaplasia) would be found on histological studies.

Option E: This patient needs close surveillance and follow-up and action should be taken for early detection of esophageal malignancies.

Gastrointestinal System/Adomen/Abdomen Wall Flashcards by Lubi Mupwaya (9)

How well did you know this?

1

Not at all

2

3

4

5

Perfectly

23

Q

A 39-year-old Aboriginal man presents to your practice with a 6-week history of abdominal pain, nausea, vomiting and bowel motions that are difficult to flush down the toilet. The abdominal pain is described as a constant and disabling radiating to back. The pain is neither brought up, nor alleviated by eating. He is a chronic alcoholic and continues to drink alcohol in large amounts. Which one of the following investigations is most likely to reveal the underlying cause of his symptoms?

A. Investigations for exocrine function of pancreas.
B. Stools exam.
C. Transabdominal ultrasound.
D. ERCP.
E. Plain abdominal X-ray.

A

C. Transabdominal ultrasound.

The clinical picture and history of excessive alcohol use makes chronic pancreatitis the most likely diagnosis. Chronic pancreatitis presents with epigastric pain as the most dominant feature. The natural history of pain in chronic pancreatitis is highly variable. Most patients experience intermittent attacks of pain at unpredictable intervals, while a minority of patients have chronic pain. In most patients, pain severity either decreases or resolves over 5-25 years. In alcohol-induced disease, alcohol cessation may reduce the severity of pain.

The pain is constant, often radiates to back and may be associated with nauseas and vomiting. It is often felt in the epigastric area but may be felt on the left side or even right side. The pain may or may not be related to eating. If related, there may be weight loss due to fear of eating. At times of pain patients bend forward to the so-called “pancreatic position” or lie in the knee-chest position on their right or left side to decrease the pain intensity.

Patients with severe pancreatic exocrine dysfunction cannot properly digest complex foods and absorb partially digested breakdown products. Nonetheless, clinical significant protein and fat deficiencies does not develop until more than 90% of the pancreatic function is lost. However, fear of eating due to pain brought up by eating, can result in early weight loss. With severe chronic pancreatitis, insulin deficiency and diabetes mellitus may also develop.

Abdominal CT scan, MRI, ultrasound, plain X-ray films and ERCP can be used for diagnosis, but CT scan is the best initial test used for imaging studies when chronic pancreatitis is suspected. CT scans are 75-90% sensitive and 85% specific. Ultrasound is the second-line initial imaging study with a sensitivity of 60-70% and specificity of 80-90% and the best among option in the absence of CT scan.

Calcifications within the pancreatic duct are present on plain films in approximately 30% of cases, making palne films a less desirable option.

The characteristic finding on imaging is pancreatic duct calcifications, ductal dilation, enlargement of the pancreas and fluid collection.

Calcium deposition is most commonly seen in alcoholic pancreatitis, but is also present hereditary and tropical forms of the disorder. It is rare in idiopathic pancreatitis.

Magnetic resonance cholangiopancreatography (MRCP) is becoming the diagnostic test of choice since it can show calcification, without any radiation risks, but is expensive and not readily available. For this reason, it is not an ideal option for initial assessment.

Endoscopic retrograde cholangiopancreatography (ERCP) is reserved for situations where non-invasive modalities are not available, are equivocal and for intervention.

How well did you know this?

1

Not at all

2

3

4

5

Perfectly

24

Q

A 45-year-old man presents to the emergency department with acute onset epigastric and right upper quadrant abdominal pain that he describes as constant and severe. He is nauseous and has vomited 3 times since the pain started. He admits to chronic heavy alcohol consumption. On examination, he has a blood pressure of 140/90mmHg, pulse rate of 110bpm and temperature of 38.4°C. The epigastric area is mildly tender to palpation. Which one of the following is the most likely diagnosis?

A. Acute cholecystitis.
B. Acute pancreatitis.
C. Acute gastritis.
D. Acute gastro-esophageal reflux disease.
E. Alcohol intoxication.

A

B. Acute pancreatitis.

The clinical picture and the history of heavy alcohol use suggest acute pancreatitis as the most likely diagnosis.

The cardinal symptom of acute pancreatitis is abdominal pain. The pain is characteristically described as dull, boring and constant. The pain is often sudden-onset and gradually becomes worse. Most often, the pain is felt in the upper abdomen usually in the epigastric region, but sometimes is perceived more on the right or left side, depending on which part of the pancreas is involved. In about 50% of the patients, the pain directly radiates through the abdomen to the back. The duration of pain is variable but typically lasts more than 1 day.

Restless and agitation may be noted. At times of pain patients bend forward to the so-called “pancreatic position” or lie in the knee-chest position. Anorexia, nausea and vomiting, and diarrhea are other likely symptoms. Acute pancreatitis secondary to alcohol, frequently occurs 1 to 3 days after a binge drinking.It can also occur after cessation of drinking.

The patient should be asked about recent operative or other invasive procedures such as ERCP, family history of hypertriglyceridemia, previous biliary colic and binge alcohol drinking as the major causes of acute pancreatitis.

Option A: Acute cholecystitis causes pain in right upper quadrant, fever and leukocytosis. Murphy’s sign is usually positive: the patient is asked to inspire deeply while the right subcostal area is palpated. The Murphy sign is cessation of inspiration due to pain. Patients with acute cholecystitis may experience increased discomfort and hold in mid-inspiration. Abdominal examination may show voluntary and involuntary guarding.

Option B: Acute gastritis usually can present with pain in epigastrium associated with nauseas and vomiting and epigastric tenderness but fever would not be a feature.

Option D: Gastroesophageal reflux disease (GERD) presents with heartburn, chronic cough, a metalic taste in the mouth. The given clinical picture is completely different from that of GERD.

Option E: Patients with alcohol intoxication presents with slurred speech, nystagmus, disinhibited behavior, incoordination, unsteady gate, memory impairment, stupor, or coma depending on the severity of intoxication.

How well did you know this?

1

Not at all

2

3

4

5

Perfectly

25

Q

The following photograph is one cut from an abdominal CT scan in a 68-year-old man. Which one of the following is the most likely diagnosis?

A. Gastric outlet obstruction.
B. Cancer of the stomach.
C. Simple hepatic cyst.
D. Pancreatic pseudocyst.
E. Hepatic hemangioma.

Gastrointestinal System/Adomen/Abdomen Wall Flashcards by Lubi Mupwaya (10)

A

A. Gastric outlet obstruction.

The picture shows a contrast CT scan of the abdomen. On the right side of the picture (left side of the patient) a distended contrast-filled stomach is seen. Duodenum is on the left side of the picture. It also contains contrast media.

The right kidney is another structure seen on the CT scan. In front of the vertebral body and slightly to the left, the aorta (containing contrast material) is noticed. Other structures seen on this cut of the CT scan are lower part of the right hepatic lobe, left hepatic lobe, spleen and parts of the pancreas.

The distended and fluid filled stomach is suggestive of gastric outlet obstruction (GOO). The distended duodenum indicates that the obstruction has occurred at the duodenum level.

Clinical entities that can result in GOO generally are categorized into two groups – benign and malignant. In the past, peptic ulcer disease (PUD) was the most common cause of GOO, but currently, 50-80% of GOOs are due to malignancies such as pylorus adenocarcinoma, lymphoma and gastrointestinal stromal tumour (GIST).

Benign causes of GOO include pancreatic pseudocysts, gastric varices, infections such as tuberculosis, and rarely gall stones.

At this level no pancreatic or hepatic lesion is seen. Cancer of the stomach would have given an obstruction in upper parts if it involves the pylorus. Besides, this might be the underlying cause of the CT scan findings not the interpretation.

How well did you know this?

1

Not at all

2

3

4

5

Perfectly

26

Q

A 35-year-old woman presents with a 4-month history of dysphagia to both solid and liquid foods. A barium study is done demonstrating dilated esophagus with tapering at the lower end. Based on the history and radiological findings, achalasia is the diagnosis. Which one of the following is the most appropriate management in this patient?

A. Pyloromyotomy.
B. Pneumatic dilation.
C. Botulinum toxin injection.
D. Calcium channel blockers.
E. Nitrates.

A

B. Pneumatic dilation.

Achalasia is a primary esophageal motility disorder characterized by the absence of esophageal peristalsis and impaired relaxation of the lower esophageal sphincter (LES) in response to swallowing. These abnormalities cause a functional obstruction at the gastroesophageal junction.

The goal of treatment is to reduce the resistance of the LES and overcome the obstruction.

Currently, standard of care for achalasia is based on guidelines from the American College of Gastroenterology and is as follows:

  • Initial therapy should be either graded pneumatic dilation (PD) or laparoscopic surgical myotomy in patients who are fit for such surgeries.
  • Botulinum toxin injection into the LES is considered for patients who are not appropriate candidates for surgery.
  • Pharmacologic therapy can be used for patients not undergoing PD or myotomy and who have failed botulinum toxin therapy (nitrates and calcium channel blockers most common)

For this patient, either pneumatic dilation or endoscopic myotomy (not an option) is the most appropriate initial management option.

Option A: Pyloromyotomy is the procedure used for treatment of hypertrophic pyloric stenosis or obstruction.

Option C: In patients with mild symptoms or in those who are not appropriate candidates for pneumatic balloon dilation or surgery, injection of botulinum toxin is considered the next most appropriate treatment. Botulinum toxin injection has been associated with symptoms improvement, but may need to be repeated at intervals of 3-12 months.

Option D and E: Pharmacological therapy can be used for those patients not undergoing pneumatic dilation or myotomy, and who have failed botulinum toxin therapy. These medications, however, may not be desirably effective. First-line medical treatment is with glyceryl trinitrate (oral or sublingual spray). If the episodes are associated with spasms or are frequent or disabling, a trial of diltiazem, nifedipine, or isosorbide dinitrate can be considered.

Achalasia

How well did you know this?

1

Not at all

2

3

4

5

Perfectly

27

Q

A 72-year-old woman presents to the emergency department with sudden-onset severe pain in the right lower quadrant and and back pain. She has the history of deep vein thrombosis (DVT) two months ago, for which she was started on warfarin. On examination, she has a blood pressure of 110/75 mmHg, pulse rate of 140 bpm and temperature of 37.2°C. There is a tender mass in the right iliac fossa (RIF). Plain X-ray of her abdomen is obtained and is shown in the following photograph. Which one of the following is the most likely diagnosis?

A. Abscess of the appendix.
B. Rectus sheath hematoma.
C. Cecal volvulus.
D. Leaking abdominal aortic aneurysm.
E. Fecal impaction.

Gastrointestinal System/Adomen/Abdomen Wall Flashcards by Lubi Mupwaya (11)

A

C. Cecal volvulus.

The X-ray shows an extensively air-filled dilated proximal colon arising from the right side and extending to the left. With these X-ray findings, along with the history and physical findings, cecal volvulus is the most likely diagnosis.

Volvulus occurs when a segment of viscus twists around its axis and results result in obstruction as well as blood supply compromise.

The most common types of volvulus are sigmoid volvulus (more common) and cecal volvulus (less common). Cecal volvulus tends to occur in younger patients (30-60 years) compared to sigmoid volvulus that often is seen in elderly patients, who are bedridden or nursing home residents.

Option A: There are no suggestive clues to acute appendicitis as a precipitating event for an appendix abscess to form.

Option B: Although the clinical findings and history of warfarin use can make rectus sheath hematoma a likely diagnosis, the abdominal X-ray is consisetent with csecal volvulus.

Option D: With leaking abdominal aortic aneurysm, a tender pulsatile mass would be expected in the midline, not in the right iliac fossa.

Option E: With fecal impaction, fecal material was expected to be seen in colon (left side). Focal tenderness is not a dominant feature in fecal impaction.

Gastrointestinal System/Adomen/Abdomen Wall Flashcards by Lubi Mupwaya (12)

How well did you know this?

1

Not at all

2

3

4

5

Perfectly

28

Q

A 65-year-old man is being evaluated for assessment of the source of gastrointestinal bleeding after he presented with complaint of melena at several occasions and was found to be anemic. Due to the very low level of hemoglobin and symptoms such as severe weakness and chest pain on exertion, he received several transfusions; however, he was not able to maintain a satisfactory hemoglobin level. Upper and lower endoscopy and small bowel barium studies were performed failing to find a source of bleeding. Which one of the following is the next best step in management?

A. Capsule endoscopy.
B. Interventional angiography.
C. CT angiography.
D. Repeat endoscopy.
E. Radionuclide imaging.

A

C. CT angiography.

The case scenario is an example of obscure gastrointestinal tract (GIT) bleeding. Obscure GIT bleeding is defined as bleeding from the GIT that persists or recurs without an obvious etiology found on upper endoscopy and colonoscopy and radiologic evaluation of the small bowel (such as by small bowel follow-through or enteroclysis).

Obscure GIT bleeding accounts for approximately 5% of patients with GIT bleeding. In 75% of these patients, the source of bleeding is in small bowel. The remainder of cases are due to missed lesions in either upper or lower GI tract.

Obscure bleeding is subdivided into overt or occult, depending upon the presence or absence of clinically-evident bleeding:
Obscure occult (inactive) bleeding – This is manifested as iron deficiency anemia or recurrent positive Fecal Occult Blood Test (FOBT) results.

Obscure overt (active) bleeding – This is manifested as recurrent episodes of clinically evident bleeding (e.g., melena or hematochezia), or persistent blood loss. Patients with ongoing blood loss so significant to necessitate transfusion are considered to have life-threatening active bleeding.

Further management of obscure GI bleeding depends on whether the bleeding is clinically active (overt) or inactive (occult):

Active obscure GI bleeding:
In patients presenting with active bleeding, radiological investigations are the mainstay of diagnostic approach, given their high sensitivity and non-invasiveness. CT Angiography (CTA) can detect bleeding rates of 0.5 ml/min and above. A negative CTA signifies that catheter angiography is not indicated since the latter is less sensitive than the CTA and has been shown to detect the source of bleeding only if the rate of blood loss is greater than 0.5 ml/min.

With a negative CTA and continuous bleeding, technetium 99m – labeled RBC nuclear scan is used as the second-line investigation at some institutions while others may use this as the first-line investigation. Among available modalities, RBC nuclear scan is most sensitive for active GI bleeding and can detect bleeding rates as low as 0.1 mL/min.

NOTE - Life-threatening hemorrhage should indicate catheter angiography as first-line management. A hemorrhage so significant is very likely to light up on catheter angiography. With catheter angiography, the site of bleeding can be found and controlled at the same time.

Inactive obscure GI bleeding:
Capsule endoscopy is the option of choice. However, a negative test demands CT angiography or other radiologic modalities (radionuclide scan, etc.) for further assessment.
—–
This patient has low hemoglobin despite several transfusions; however, he does not have urgent life-threatening hemodynamic instability such as low blood pressure, tachycardia, shortness of breath, chest pain, or hypoperfusion-related altered mental state. For him a CTA would be the most appropriate next step in management to find the source of bleeding.

Option A: Capsule endoscopy is the first option for patients with occult obscure GI bleeding i.e. iron deficiency anemia or recurrent positive Fecal occult blood test (FOBT) results.

Option B: Interventional angiography was the option of choice if the patient had immediate life-threatening hemodynamic instability at the moment. Although this patient has low hemoglobin, more transfusion can temporarily stabilize him to fulfill the criteria for CTA.

Option D: Repeating the endoscopic studies is unlikely to add any information regarding the site of bleeding, as this measure has already failed to do so.

Option E: Radionuclide imaging is used when there is obscured GI bleeding of unknown origin despite CT angiography or capsule endoscopy. The test takes a long time to perform. This patient with brisk and significant blood loss is not a good candidate for this test.

How well did you know this?

1

Not at all

2

3

4

5

Perfectly

29

Q

Which one of the following is the most common presenting symptom of a patient with duodenal stricture secondary to duodenal ulcer?

A. Vomiting 3-4 times a day.
B. Abdominal distension.
C. Right upper quadrant pain.
D. Vomiting after one hour of meals.
E. Vomiting early in the morning.

A

D. Vomiting after one hour of meals.

Gastric outlet obstruction (GOO), is the consequence of any disease producing a mechanical obstruction to gastric emptying. Clinical entities that can cause GOO are generally categorized into benign and malignant.

Peptic ulcer disease (PUD) is among the benign causes of GOO; however, the incidence of ulcer-induced GOO has dramatically declined owing to adequate and efficient treatment of PUD. The mechanism of obstruction by PUD can be either edema around the ulcer, or scar formation after the ulcer heals.

The most common symptoms of GOO, regardless of the underlying etiology, are bloating, anorexia, nausea and vomiting. Vomiting usually is described as nonbilious, and characteristically containing undigested food particles.

Patients with gastric outlet obstruction from a duodenal ulcer or incomplete obstruction typically present with symptoms of gastric retention, including early satiety, bloating or epigastric fullness, indigestion, anorexia, nausea, vomiting, epigastric pain, and weight loss. They are frequently malnourished and dehydrated and have a metabolic insufficiency. Weight loss is frequent when the condition is chronic and is most significant in patients with malignant disease.

Abdominal pain is not frequent and usually relates to the underlying cause, e.g., PUD, pancreatic cancer.

The time of vomiting can suggest the site of obstruction. In pyloric obstruction (more proximal) the time of vomitus is usually within the first hour of eating, whereas in pyloric stenosis, or duodenal stenosis or obstruction (more distal) the vomiting occurs after one hour because normally it takes 45 minutes to 1 hour for the food to reach the duodenum.

Option A: If the patient has 3 or 4 meals a day he may vomit 3 to 4 times within one hour after each meal, but vomiting 3 to 4 times a day by itself is not a common presentation of gastric outlet obstruction.

Option C: Right upper quadrant abdominal pain is seen in hepatobiliary diseases and is not a characteristic feature of gastric outlet obstruction.

Option B: Abdominal distension is a feature of bowel obstruction, particularly obstruction of the large bowel.

Option E: Early morning vomiting is associated with raised intra-cranial pressure and pregnancy, and is not a feature of gastric outlet obstruction.

How well did you know this?

1

Not at all

2

3

4

5

Perfectly

30

Q

A 56-year-old man presents to the emergency department with acute-onset severe epigastric pain, fever, nausea and vomiting. Past medical history includes type 2 diabetes mellitus, cardiomyopathy, and hypertension. Abdominal ultrasound shows a stone in the bile duct, multiple gallstones in the gallbladder and a dilated common bile duct of 6mm. Laboratory studies are significant for elevated conjugated bilirubin, elevated white cell count, and a normal hemoglobin level. Liver function tests show a 6-time rise in alanine aminotransferase and aspartate aminotransferase. Lipase and amylase are also elevated. Which one of the following options is the most appropriate next step in management?

A. Abdominal CT scan.
B. ERCP.
C. MRCP.
D. Upper endoscopy.
E. Monitor liver function tests.

A

B. ERCP.

The clinical and laboratory picture suggests acute pancreatitis secondary to gallstones and common bile duct obstruction that has led to LFT abnormalities as well.

Gallstone pancreatitis necessitates specific therapeutic consideration in addition to conventional treatment of acute pancreatitis.

Multiple studies suggest that early endoscopic retrograde cholangiopancreatography (ERCP), within 24-48 hours, with papillotomy, or surgical intervention to remove bile duct stones is beneficial in patients with acute biliary pancreatitis and the presence of the following:

  • Concomitant acute cholangitis
  • Persisted common bile duct obstruction evident by worsening signs or symptoms of obstruction (clinical deterioration), or deteriorating liver function tests

Endoscopic papillotomy is typically accompanied by placement of a plastic biliary stent to decrease the risk of post-ERCP pancreatitis.

With increased liver function tests (LFTs), especially the elevated conjugated bilirubin, this patient should be considered to have concomitant cholangitis until proven otherwise. Considering the likelihood of acute cholangitis caused by CBD obstruction, this patient should undergo early ERCP as the most appropriate next step in management, after rehydration, analgesia, and antibiotics. LFTs are used to monitor the response to treatment.

An ultrasound scan (not an option) is the very first imaging investigation for patients with signs and symptoms of pancreatitis or pancreatic cancer if manifestations of biliary obstruction are also present. Otherwise, Abdominal CT scan is used; However, in the absence of clinical features of biliary obstruction, CT scan is the preferred diagnostic modality.

CT scan is the most important imaging test for the diagnosis of acute pancreatitis, its severity and potential complication Patients with clinical and biochemical features of acute pancreatitis. CT scan is indicated if:

  • there is no improvement with conservative therapy
  • complications are suspected
  • other diagnoses than pancreatitis is suspected

Acute Pancreatitis

How well did you know this?

1

Not at all

2

3

4

5

Perfectly

31

Q

A 70-year-old man presents to the emergency department with acute epigastric pain, nausea and vomiting. He has the history of gallstones. Laboratory tests show moderate elevation of lipase, amylase, alanine aminotransferase (ALT) and aspartate aminotransferase (AST). Both conjugated and unconjugated bilirubin are within the normal range. Inflammatory markers are not elevated. Abdominal ultrasound reveals a small stone in the common bile duct (CBD), but CBD is not dilated. Which one of the following is the most appropriate next step in management?

A. Abdominal CT scan.
B. MRCP.
C. ERCP.
D. Intravenous fluids, analgesics and antiemetics.
E. Monitoring the liver function tests.

A

D. Intravenous fluids, analgesics and antiemetics.

This patient has clinical diagnosis of acute pancreatitis, most likely secondary to gallstones; however, a non-dilated common bile duct (CBD) and normal bilirubin excludes CBD obstruction. Management of the acute pancreatitis should be started with intravenous fluids, analgesia and antiemetics. The patient should be nil by mouth (NPO) and NG tube inserted for GI decompression.

Further attention then should be paid to biliary stones as the underlying cause. Early ERCP (within 24-48 hours) is recommended for patients with acute biliary pancreatitis and either of the following:
* Concomitant acute cholangitis
* Persisted common bile duct obstruction evident by worsening sign or symptoms of obstruction, or deteriorating liver function tests

Elevated liver enzymes and/or CBD stone on ultrasound are pointers towards a biliary cause for acute pancreatitis; however, normal level of bilirubin and the non-dilated of CBD on ultrasound exclude cholangitis with high certainty. For this patient early ERCP is not indicated, but ideally the patient should undergo semi-elective cholecystectomy during this hospital stay.

CT scan is the most important imaging test for diagnosis of acute pancreatitis and its intraabdominal complications and also for assessment of severity. Patients with clinical and biochemical features if acute pancreatitis, who do not improve with initial conservative therapy or who are suspected of having complications or other diagnoses should undergo CT scan of the abdomen. This patient may need CT scan later (usually not earlier than 72 hours) in the course of the disease but not now as the most appropriate next step.

Monitoring LFTs are required for assessment of response to treatment, but not a priority in initial management.

How well did you know this?

1

Not at all

2

3

4

5

Perfectly

32

Q

A 49-year-old man with smoking history of 30 pack/year and longstanding gastro-esophageal reflux disease (GERD) presents with progressive difficulty in swallowing for the past 6 months. Initially, he had difficulty swallowing solid foods, but later he also developed dysphagia to liquids. He has malaise, fatigue, and has lost 15 kg in the past 3 months. On clinical examination, he looks pale and anemic. Which one of the following would be the next best step in management?

A. CT scan of the chest.
B. Barium Swallow.
C. Esophagoscopy.
D. Bronchoscopy.
E. Chest X-ray.

A

C. Esophagoscopy.

The clinical picture including the history of heavy smoking and GERD, as well as the characteristics of the dysphagia is suggestive of mechanical dysphagia most likely caused by esophageal cancer. Weight loss can be seen in any kind of dysphagia; however, a significant weight loss of 15 kg in only 3 months suggests a more malignant etiology. Anemia is another clue to malignancy.

When esophageal cancer is suspected, the best initial test is endoscopy (esophagoscopy) and biopsy for definite diagnosis.

Option B: Barium swallow is very sensitive for detection of strictures and intraluminal masses but does not allow staging and biopsy. It is now rarely used when cancer is suspected.

Option A, D and E: Bronchoscopy , chest X-ray , and CT scan of the chest are all used as tools for further evaluation and staging of the tumor, once esophagoscopy established a diagnosis of esophageal cancer.

Bronchoscopy may be used for malignancies of upper (and middle) esophagus because it is common for tumors of these parts to invade the tracheobronchial tree.

NOTE - The extent of tumor spread to the mediastinum and para-aortic lymph nodes should be assessed by CT scanning of the chest and abdomen and endoscopic ultrasound. Endoscopic ultrasonography is the most sensitive test for assessment of the tumor depth. Other studies to consider for staging are bone scan and liver function tests.

How well did you know this?

1

Not at all

2

3

4

5

Perfectly

33

Q

Which one of the following factors has the greatest impact on development of esophageal adenocarcinoma?

A. Achalasia.
B. Barrett’s esophagus.
C. Low-fiber diet.
D. Smoking.
E. Gastro-esophageal reflux disease.

A

B. Barrett’s esophagus.

While smoking and alcohol are the two most common risk factors for squamous cell carcinoma of the esophagus, Barrett’s esophagus is the most significant risk factor for development of esophageal adenocarcinoma.

Option A: Achalasia has been associated with esophageal cancer but is not as significant as other factors such as smoking and alcohol in squamous cell carcinoma and Barret’s esophagus in adenocarcinoma of the esophagus.

Option C: Although implicated in some studies, low-fiber diet is not as significant as other risk factors for esophageal cancer.

Option D: Smoking and chronic alcohol ingestion are the most common and important risk factors for development of esophageal squamous cell carcinoma.

Option E: Gastro-esophageal reflux disease is not a risk factor for esophageal adenocarcinoma unless it results in Barrett’s esophagus (indirect effect).

NOTE - Obesity is considered another important risk factor, mostly because it predisposes to reflux disease. However, the outcome of these conditions – GERD and the consequent Barrett’s esophagus, remains the main and most significant risk factor through producing metaplastic changes that can progress to dysplasia and eventually cancer.

How well did you know this?

1

Not at all

2

3

4

5

Perfectly

34

Q

A 42-year-old woman presents to the emergency department with right sided abdominal pain, mild jaundice and fever. She has a blood pressure of 130/85 mmHg, pulse rate of 90 bpm and temperature of 38.1°C. She is admitted to the hospital. After 3 hours, she develops rigors and chills. On re-exam, her blood pressure has fallen to 90/60 mmHg, her pulse has risen to 110 bpm and her temperature is 39.5°C. Which one of the following is the most likely diagnosis?

A. Septic shock.
B. Choledocholithiasis.
C. Acute pancreatitis.
D. Acute cholecystitis.
E. Acute cholangitis.

A

E. Acute cholangitis.

The clinical picture is highly suggestive of acute cholangitis as the most likely diagnosis. This patient has signs and symptoms consistent with diagnosis of acute cholangitis. Fever is present in approximately 90% of patients. Abdominal pain and jaundice occurs in 70% and 60% of patients, respectively. The patient can become quickly septic and hypotensive with altered level of consciousness.

Option A: Septic shock can result from acute cholangitis, and is a clinical syndrome not a diagnosis.

Option B: Choledocholithiasis is the presence of gallstones within the common bile duct. A patient with uncomplicated common bile duct stones typically develops abdominal pain with mild liver function abnormalities. Fever is not a usual fever in uncomplicated patients. The two major complications associated with choledocholithiasis are acute cholangitis and acute pancreatitis.

Option C: In acute pancreatitis, the abdominal pain is typically accompanied by nausea and vomiting, which may persist for many hours. The patient is often restless. Jaundice is not a feature unless there is concomitant common bile duct (CBD) obstruction and cholangitis.

Option D: Acute cholecystitis is inflammation of the gallbladder, characterized by right upper quadrant pain and tenderness, fever, and leukocytosis. Gallstones are the most common etiology. Jaundice is not a feature of acute cholecystitis.

How well did you know this?

1

Not at all

2

3

4

5

Perfectly

35

Q

A 56-year-old man presents with complaints of 10 kg weight loss in the past three months and jaundice. He denies any abdominal pain, but he describes symptoms of early satiety. Recently, he has noticed dark urine and pale stool. On examination, he has icteric sclera and a palpable right upper quadrant mass that moves with respiration. No abdominal tenderness is elicited. Which one of the following is the most appropriate initial investigation?

A. Abdominal CT scan.
B. Abdominal ultrasonography.
C. Chest X-ray.
D. ERCP.
E. MRCP.

A

B. Abdominal ultrasonography.

The clinical presentation is highly suggestive of pancreatic head carcinoma. This is evident by painless obstructive jaundice and a distended gallbladder (the right upper quadrant mass that moves with respiration).

Pancreatic cancer can present with the following (percentages are from one recent study of 186 patients with pancreatic cancer):
* Malaise and fatigue – 86%
* Weight loss – 85%
* Anorexia – 83%
* Abdominal pain – 79%
* Epigastric pain – 71%
* Dark urine – 59%
* Jaundice – 56%
* Nausea – 51%
* Back pain – 49%
* Diarrhea- 44%
* Vomiting – 33%
* Steatorrhoea – 25%
* Thrombophlebitis – 3%

The most frequent signs are:
* Jaundice – 55%
* Hepatomegaly- 39%
* Right upper quadrant mass – 15%
* Cachexia – 13%
* Courvoisier’s sign (nontender but palpable distended gallbladder at the right costal margin) – 13%
* Epigastric mass – 9%
* Ascites – 5%

Of all pancreatic malignant tumors, most are found in the pancreatic head as the most common site. Pancreatic body and tale are second and third most common sites, respectively.

Initial imaging study for assessment of suspected pancreatic cancer depends on the presenting symptoms. For patients with jaundice, the initial imaging study is typically a transabdominal ultrasound. It has high sensitivity for detecting biliary tract dilation and determining the level of the obstruction. It also has high sensitivity (>95%) for detection of a mass in the pancreas. The sensitivity is lower for tumours < 3 cm.

Option A: Abdominal CT , however, is the preferred initial imaging modality in patients with epigastric pain and weight loss, but without jaundice. A contrsast-enhanced CT scan of the abdomen is alwasy considered for such patients.

Option C: Chest X-ray is not indicated for diagnostic imaging of pancreatic cancer.

Option D and E: ERCP and MRCP may be indicated in further assessment, in cases with obstructive jaundice.

Cholestatic Jaundice

How well did you know this?

1

Not at all

2

3

4

5

Perfectly

36

Q

A 32-year-old woman presents to the Emergency Department with abdominal pain and fever since yesterday, which now has localized in the pelvis. On examination, he has a blood pressure of 110/85 mmHg, pulse rate of 98 bpm, respiratory rate of 22 breaths per minute, and fever of 38.2°C. Palpation of the abdomen elicits no tenderness, rebound, or guarding, but there is mild tenderness on rectal exam. Which one of the following would be the next best step in management of this patient?

A. Laparoscopic drainage and appendectomy.
B. Open drainage.
C. CT scan of the pelvis and the abdomen.
D. Ultrasonography of the pelvis and the abdomen.
E. Intravenous antibiotics.

A

D. Ultrasonography of the pelvis and the abdomen.

The clinical picture is suggestive of pelvic appendicitis. The most common symptom of appendicitis is abdominal pain. Typically, symptoms begin as periumbilical or epigastric pain migrating to right lower quadrant of the abdomen. This pain migration is the most characteristic feature in the patient’s history. However, in up to 30% of the time, the appendix may be hidden from the anterior peritoneum by being in pelvic, retroileal or retrocolic (retroperitoneal retrocecal) position. The hidden position of the appendix may significantly alter the clinical manifestation of the appendicitis.

A patient with a pelvic appendix may show no abdominal signs, but rectal examination may cause tenderness in cul-de-sac. Additionally, an obturator sign (pain on passive internal rotation of the flexed right thigh) may be present in a patient with pelvic appendix.

With typical presentation of appendicitis, imaging studies are not necessary and the patient will be taken to the operating room for appendicectomy, but when diagnosis of appendicitis is not certain (such as in this case) imaging studies are required for confirmation.

CT scan, specially the technique of appendiceal CT, is the most accurate imaging study (more accurate than ultrasonography). Appendiceal CT gives the highest diagnostic yield. If appendiceal CT is not available, standard abdominal/pelvic CT with contrast remains highly useful and may be more accurate than ultrasonography.

However, because of concerns about patient exposure to radiation during CT scans, ultrasonography has been suggested as a safer primary diagnostic modality for appendicitis, with CT scanning used secondarily when ultrasounds are negative or inconclusive. A healthy appendix usually cannot be viewed with ultrasonography. When appendicitis occurs, the ultrasonogram typically demonstrates a noncompressible tubular structure of 7-9 mm in diameter. CT scan is the next step if ultrasound is non-diagnostic.

NOTE - Pelvic appendicitis may present with:

  • An absence of abdominal wall rigidity and tenderness
  • Tenderness in the rectovesical pouch and Pouch of Douglas on rectal examination
  • Spasm of psoas muscle on right-hand-side
  • Diarrhea due to rectal irritation by the inflamed appendix
  • Frequency of micturition due bladder irritation by the inflamed appendix
  • Hypogastric pain on flexing and internally rotating the hip - due to contact of inflamed appendix with obturator internus muscle

How well did you know this?

1

Not at all

2

3

4

5

Perfectly

37

Q

A 57-year-old man presents to the emergency department with absolute constipation for two days and ongoing vomiting for the past eight hours. On examination, the abdomen is not distended or tender. The rest of the exam is unremarkable except signs of mild dehydration. Gastrograffin studies confirm the presence of small bowel obstruction. Which one of the following is the most common cause of small bowel obstruction?

A. Postoperative adhesions.
B. Bowel cancer.
C. Crohn’s disease.
D. Hernias.
E. Hypokalemia.

A

A. Postoperative adhesions.

Post-operative adhesions are the most common cause of small bowel obstruction (SBO). The risk of developing an obstruction after surgery from postoperative adhesions is estimated to be 9% within the first year after abdominal surgery, 19% by 4 years, and 35% by 10 years.

The incidence of adhesive SBO seems to be higher in open surgeries than in laparoscopic surgeries. The cause for such difference is not clear, but is likely to be related to the degree of manipulation and trauma to the intraabdominal tissues.

Malignant tumors are the second most common cause of SBO, accounting for about 20% of cases. SBO has been described in as many as 42% of women with ovarian carcinoma and 28% of patients with colorectal carcinomas.

Hernias are the third leading cause of SBO and account for approximately 10% of all cases. Ventral and inguinal hernias are most commonly associated with obstruction; however, internal hernias, femoral, obturator and parastomal hernias also can contribute to SBO.

Other less common causes of small bowel obstruction include:

  • Strictures e.g. caused by Crohn’s disease
  • Small bowel tumors
  • Trauma
  • Intussusception
  • Bezoars
  • Gallstone ileus (pseudo-obstruction)
  • Superior mesenteric artery syndrome
  • Hypokalemia (pseudo-obstruction)

Small-Bowel Obstruction

How well did you know this?

1

Not at all

2

3

4

5

Perfectly

38

Q

A 57-year-old man presents to the emergency department with fever and right upper quadrant pain. On examination, he has a blood pressure of 80/60mmHg, pulse rate of 110 bpm, respiratory rate of 22 breaths per minute and temperature of 38.9°C. The right upper quadrant is tender to palpation. Laboratory studies reveal high white cell count and mildly elevated bilirubin. Based on the findings you suspect acute cholangitis as the diagnosis. Which one of the following organisms is most likely to have caused sepsis?

A. Staphylococcus aureus.
B. Pseudomonas aeruginosa.
C. Enterococcus fecalis.
D. Escherichia coli.
E. Treponema pallidum.

A

D. Escherichia coli.

Acute cholangitis is a clinical syndrome characterized by fever, abdominal pain and jaundice, developing as a result of stasis and infection in the biliary tract. It can rapidly progress to sepsis. Biliary system obstruction is the most common predisposing factor.

Culture of bile, ductal stones, and blocked biliary stents are positive in over 90% of cases, yielding a mixed growth of gram negative and gram positive bacteria. The most common bacteria isolated have colonic origin.

Escherichia coli is the major bacterium isolated (25 to 50%), followed by Klebsiella (15-20% percent) and Enterobacter species (5-10%). Enterococcus species (e.g. enterococcus fecalis) are the most common gram positive bacteria (10-20%).

Anaerobes, such as bacteroides and Clostridia, are usually present as isolated pathogens , but they are rarely the sole infecting organisms and it is not clear if they play a role in acute cholangitis.

How well did you know this?

1

Not at all

2

3

4

5

Perfectly

39

Q

Which of the following is a contraindication to surgical intervention in esophageal malignancies?

A. Invasion to tracheobronchial tree.
B. Invasion to small blood vessels.
C. Lesions greater than 5cm.
D. Odynophagia.
E. Aspiration pneumonia.

A

A. Invasion to tracheobronchial tree.

Surgery is the cornerstone of treatment for esophageal cancer; however, in the following situations surgery is contraindicated:

  • Distant metastases – the presence of peritoneal, lung, bone, adrenal, brain, or liver metastasis, or extraperitoneal lymph node spread (e.g., paraaortic or mesenteric lymphadenopathy). Approach to patients with celiac lymph node involvement remains controversial.
  • Invasion to adjacent structures – invasion of the tumor to the aorta, trachea, heart, great vessels, or the presence of tracheoesophageal fistula (tumor stage: T4b)

NOTE - With invasion to the pleura, pericardium, or diaphragm the cancer is classified as T4a, and considered potentially resectable.

In addition, the presence of severe, associate comorbid conditions such as cardiovascular or respiratory diseases decreases a patient’s chance of survival and success of esophageal resection. Every patient should undergo cardiac and respiratory function assessment prior to the surgery. A forced expiratory volume in 1 second (FEV1) less than 1.2 L and a left ventricular ejection fraction of less than 40% are relative contraindications to the surgery.

Of the options, only invasion of the tumor to the tracheobronchial tree is a contraindication to surgical resection.

40

Q

During endoscopic evaluation of a 65-year-old man for dyspepsia, he is found to have an abnormal-appearing patch of mucosa in the distal part of his esophagus. Biopsy result is significant for Barrett’s esophagus and metaplasia. Which one of the following is correct regarding his condition?

A. He should be monitered every 2-3 years by endoscopy and biopsy.
B. He should be monitered by endoscopy every third month.
C. There is 70-80% risk of progression to adenocarcinoma.
D. Hiatus hernia has no association to adenocarcinoma.
E. Barrett’s esophaus is an indication of urgent surgical intervention.

A

A. He should be monitered every 2-3 years by endoscopy and biopsy.

With metaplasia present in this patient, surveillance with endoscopy and biopsy should be performed every 2-3 years. Other options are incorrect.

Barrett’s esophagus is a condition in which metaplastic columnar epithelium replaces the stratified squamous epithelium of the distal esophagus. The metaplastic epithelium is acquired as a consequence of chronic gastro-esophageal reflux disease (GERD). Hiatus hernia predisposes to GERD and indirectly to development Barrett’s esophagus.

Barrett’s esophagus is a premalignant condition with a 40- to 125-fold increase in the incidence of adenocarcinoma of the lower esophagus; however, the absolute risk relatively small and approximately 0.5% per year.

Every patient with the alarming symptoms of weight loss, anemia, heme-positive stool, and dysphagia, or GERD for more than 5 years should undergo endoscopy and biopsy. Further management depends on the histological findings and is summarized in the following table:

Gastrointestinal System/Adomen/Abdomen Wall Flashcards by Lubi Mupwaya (13)

41

Q

Which one of the following is most important to look for on an ultrasound scan in a patient with jaundice?

A. Gallstones.
B. Head of the pancreas.
C. Dilation of bile ducts.
D. Hepatic masses.
E. Thickening of the gallbladder wall.

A

C. Dilation of bile ducts.

Jaundice can be caused by a variety of etiologies that broadly are categorized into extrahepatic or intrahepatic.

A careful history and physical examination is essential. Laboratory tests follow for further evaluation.. The most important laboratory values to consider are bilirubin (total and unconjugated), hepatic transaminases, gamma glutamyl transpeptidase (GGT), alkaline phosphatase and coagulation profile.

If the history, physical exam, and initial laboratory studies suggest obstruction of the biliary tree as the underlying pathophysiology of jaundice, an imaging study is indicated to differentiate extrahepatic and intrahepatic causes of cholestatic jaundice. The imaging tests that can be used are abdominal ultrasonography, endoscopic ultrasonography, abdominal CT scan, ERCP, and MRI.

Abdominal ultrasonography is the initial imaging modality of choice, becasue it is inexpensive and readily available. In evaluation of jaundice, the most important finding to look for is obstruction of the biliary tree, evident by biliary duct dilation. Sensitivity of abdominal sonography ranges between 55% and 91% according to different studies. The sensitivity increases with increased serum bilirubin and duration of the jaundice.

Ultrasound can also show stones; however, common bile duct (CBD) stones may not be visualized enough due to the duodenal gas obscuring the distal CBD. Gallstones, hepatic masses and pancreas may also be visualized on ultrasonography. Gallbladder thickening suggests cholecystitis. Cholecystitis per se does not result in jaundice, unless cholangitis or distal biliary tree obstruction occurs.

42

Q

A 70-year-old woman presents to the Emergency Department with progressive abdominal distention for the past 3 days. She has not passed any stool or flatus. She has vomited several times since this morning. She relates that symptoms started 3 days ago after she fell off her bed. On examination, she has a blood pressure of 100/75 mmHg, pulse rate of 90 bpm, respiratory rate of 21 breaths per minute, and a temperature of 37.6°C. There is no tenderness, rebound, or guarding on abdominal examination. Which one of the following is most likely to be the cause of this presentation?

A. Sigmoid volvulus.
B. Cecal volvulus.
C. Pneumatosis coli.
D. Pseudo-obstruction.
E. Toxic megacolon.

A

D. Pseudo-obstruction.

Of the options, the most consistent one with the history of fall (trauma) and the given clinical presentation is acute colonic pseudo-obstruction, also known as Ogilvie syndrome.

Acute colonic pseudo-obstruction is a disorder characterized by gross dilatation of the cecum and the right hemicolon in the absence of an anatomic lesion obstructing intestinal flow.

Acute colonic pseudo-obstruction is associated with an underlying disease in 95% of patients, including:

  • Non-operative trauma
  • Infection
  • Myocardial infarction/ heart failure
  • Obstetric or gynecologic disease
  • Abdominal/pelvic surgery
  • Neurological problems e.g. Parkinson disease, spinal cord injury, multiple sclerosis, Alzheimer disease, etc
  • Orthopedic surgery
  • Miscellaneous medical conditions e.g. metabolic, cancer, respiratory failure, renal failure
  • Miscellaneous surgical conditions e.g. urologic, thoracic, neurosurgery

Studies suggest that non-operative trauma is among the most common causes of acute colonic pseudo-obstruction.

Acute colonic pseudo-obstruction is more common in men and in patients over the age of 60 years. Nausea, vomiting, abdominal pain, constipation, and, paradoxically, diarrhea are the primary manifestations. Abdominal distention is always present and can cause difficulty breathing. On physical examination, the abdomen is tympanic; however, bowel sounds are present in almost 90% of patients. Peritoneal signs are absent in the early stages of the disease; if they develop, they suggest impending perforation.

Option A and B: Volvulus (cecal or sigmoid) is associated with acute onset of pain and obstruction, but a volvulus of three-day duration would have resulted in a very ill and toxic patient due to ischemia of the affected part of the colon and signs of peritoneal irritation.

Option C: Pneumatosis coli is the presence of gas in the large intestinal wall. It is a condition not a diagnosis.

Option E: Patients with toxic megacolon typically appear very ill with fever, tachycardia and abdominal tenderness. There is often a history of bloody diarrhea or other signs and symptoms of chronic inflammatory bowel disease.

Acute colonic pseudo-obstruction (Ogilvie’s syndrome)

43

Q

Which one of the following is the most common type of esophageal cancer worldwide?

A. Adenocarcinoma.
B. Squamous cell carcinoma.
C. Lymphoma.
D. Metastatic lesions.
E. Sarcoma.

A

B. Squamous cell carcinoma.

The epidemiology of esophageal carcinoma has markedly changed over the past several decades. Worldwide, squamous cell carcinoma is the most common type of esophageal malignancies. Smoking and alcohol are the most significant risk factors for this type of esophageal cancer.

However, in developed countries such as United States of America, UK, and Australia, the incidence of adenocarcinoma of the distal esophagus and gastro-esophageal junction has progressively increased and accounts for over 70% of all cases of esophageal cancer. Gastroesophageal reflux disease (GERD) can result in Barrett’s esophagus which is the most significant risk factor for adenocarcinoma of the esophagus.

44

Q

A 24-year-old woman develops moderate, generalized abdominal pain of sudden onset, and faints shortly thereafter. On examination, she is pale and diaphoretic, with a blood pressure of 80/50 mmHg, pulse rate of 110 bpm, respiratory rate of 22 breaths per minute and temperature of 37.6°C. The abdomen is mildly distended and tender to touch. The rest of the exam is unremarkable. She has no history of recent abdominal trauma, but has been on oral contraceptive pills (OCP) since the age of 15 years. Which one of the following could be the most likely diagnosis?

A. Strangulated hernia.
B. Mesenteric artery occlusion.
C. Rupture of abdominal aortic aneurysm.
D. Ruptured a hepatic adenoma.
E. Acute appendicitis.

A

D. Ruptured a hepatic adenoma.

The age of the patient, the acute onset of the presentation and the history of OCP use make a ruptured hepatic adenoma the most likely diagnosis.

Hepatocellular adenomas (HCAs), also known as hepatic adenomas or liver cell adenomas, are rare, benign tumors of epithelial origin and occur in less than 0.004% of the population at risk.

HCAs occur mostly in women of childbearing age and are strongly associated with the use of oral contraceptive pills (OCPs) and other estrogens. This is reflected by a dramatic increase in the incidence of this disease since OCPs were introduced in the 1960s.

The clinical presentation varies widely. Main features in history and physical examination may include the following:

  • Pain in the right upper quadrant or epigastric region in 25-50% of patients with HCAs
  • Lesions may be noticed by patients as a palpable mass. Lesions may also be discovered incidentally during an abdominal imaging study for an unrelated reason
  • History of birth control or anabolic steroid use should be checked in patients with suspected hepatocellular adenomas.
  • Patients may present with severe, acute abdominal pain with bleeding into the abdomen, which results in signs of shock such as hypotension, tachycardia, and diaphoresis
  • Hemoperitoneum occurs more frequently if the patient is taking a high-dose OCP, is actively menstruating or pregnant, or is within 6 weeks postpartum. Location of the lesion also is important, with those near the surface of the liver more prone to causing hemoperitoneum.

Option A: A strangulated hernia is very unlikely to present with this clinical picture and an onset so acute.

Option B: Mesenteric artery occlusion is usually seen in elder patients with risk factors for thrombosis or embolus formation. There is often a concomitant medical condition predisposing the patient to embolus formation such as atrial fibrillation. The condition presents with postprandial diarrhea, abdominal pain and abdominal exam findings that are usually disproportionate to the symptoms. Rectal bleeding may be found on digital rectal exam.

Option C: Abdominal aortic aneurysm (AAA) is a disease seen in older male patients. Rupture of an AAA may present similarly; however, the age and gender of this patient make this diagnosis very unlikely.

Option E: Acute appendicitis presents with vague periumbilical pain that gradually localizes in the right lower quadrant. Symptoms of peritonitis and shock develop later following rupture of the inflamed appendix. Acute appendicitis is a very unlikely explanation for this presentation.

45

Q

A 65-year-old woman presents to the emergency department with a painful discrete swelling below and lateral to the pubic tubercle. On examination, the swelling is non-reducible and mildly tender to touch. Coughing or straining does not make any changes in the swelling. Which one of the following is the most appropriate next step in management of this woman?

A. Surgical exploration.
B. Ultrasonography of the swelling.
C. NPO, nasogastric tube and bowel rest.
D. Fine needle aspiration (FNA) of the swelling.
E. Excisional biopsy of the swelling.

A

A. Surgical exploration.

The clinical picture is suggestive of a femoral hernia as the most likely diagnosis. Irreducibility and the fact the swelling is not altered by cough or straining suggests that the hernia has been incarcerated. Tenderness to palpation, if present, indicates strangulation.

Femoral hernias account for 2-4% of groin hernias, are more common in women, and are more likely to present with strangulation requiring emergency surgery. An incarcerated hernia needs watchful monitoring for signs of strangulation while semi- elective surgery is arranged. Where clinical picture suggests strangulation, emergency surgical intervention should be performed without delay.

For most hernias (as in this case), the location will be obvious on physical examination. Femoral hernias most commonly present inferior to the inguinal ligament and medial to the femoral artery. They present with a swelling below and lateral to the pubic tubercle, in contrast to inguinal hernias that present as a swelling medial to the pubic tubercle.

Most of the time, the diagnosis is evident by physical exam; however, in situations when the physical exam is uncertain, such as in obese patients, additional imaging studies are recommended.

This is particularly important owing to the fact that femoral hernias are approached differently from inguinal hernias. While watchful waiting may be an option for asymptomatic or minimally symptomatic inguinal hernia, this is not recommended for femoral hernias due to the high risk of complications.

Option A: Ultrasonography is used when the diagnosis is uncertain clinically. CT scan is used if ultrasound results are equivocal and non-diagnostic. MRI has the highest accuracy but is not readily available and cost-effective.

Option C: Femoral hernias have a high rate of incarceration and strangulation and it is unlikely that an incarcerated femoral hernia resolves with bowel rest.

Option D and E: The diagnosis is certain in this case scenario. Fine needle or excisioanl biopsies are not necessary when hernia is suspected.

46

Q

A 45-year-old refugee Somali man presents with rectal bleeding. He describes the bleeding as bright blood covering the stool. Which one of the following should be the next investigation?

A. Rectal examination.
B. Fecal occult blood test.
C. CT scan of the abdomen and pelvis.
D. MRI of the abdomen and pelvis.
E. Colonoscopy.

A

A. Rectal examination.

In approach to every patient with rectal bleeding, a complete history should be taken and physical examination performed. Inspection of the anus and a digital rectal exam must be carried out for every patient presenting with rectal bleeding as the most important initial step. If available, anoscopy or proctoscopy is an appropriate means of inspecting the anorectal area, especially during active bleeding.

Although often benign in nature, rectal or occult bleeding should always be attributed to more serious causes until proven otherwise by endoscopic testing. Flexible sigmoidoscopy in low-risk younger patients or colonoscopy in most other patients establishes the diagnosis of hemorrhoids, and excludes other causes of bleeding such as colitis, tumors, polyps, or vascular malformations.

Option B: Occult blood testing in the presence if overt bleeding is not indicated.

Option C and D: Imaging modalities such as CT scan or MRI are not routinely used for evaluation of rectal bleeding.

Option E: Colonoscopy is considered if the source of bleeding is suspected to be more proximal. With colonoscopy the entire colon can be often visualized.

47

Q

A 56-year-old man presents to your practice with complaint of fresh rectal bleeding covering his stool. He has long-standing history of constipation. He mentions no pain on defecation. Which one of the following can be the most likely cause?

A. Colorectal cancer.
B. External hemorrhoid.
C. Internal hemorrhoid.
D. Anorectal fistula.
E. Anal fissure.

A

C. Internal hemorrhoid.

Fresh blood covering the stool in the background history of prolonged constipation is most likely caused by internal hemorrhoid.

Typical pattern of rectal bleeding in internal hemorrhoids is usually associated with a bowel movement. Bright-red blood typically coats the stool at the end of defecation. Blood may also drip into the toilet bowl or stain toilet paper. Hemorrhoidal bleeding is almost always painless, and painful defecation prompts investigation for other causes such as anorectal fissures.

Although the pattern of rectal bleeding in this man is more likely to have been caused by hemorrhoids, overt or occult rectal bleeding should not be merely attributed to hemorrhoids until other less common but more serious causes of rectal bleeding have been excluded by endoscopic testing. Flexible sigmoidoscopy or anoscopy in low-risk younger patients or colonoscopy in most other patients establishes the diagnosis of hemorrhoids, and excludes other causes of bleeding such as colitis, tumors, polyps, or vascular malformations.

Option A: Presence of rectal bleeding in this patient should also raise suspicion against colorectal cancer. However, this diagnosis is less likely because firstly, there are no other suggestive findings such as weight loss, altering bowel habits, etc. and secondly, hemorrhoids are more common than cancers statistically.

Option B and E: External hemorrhoid and anal fissure are associated with significant anal pain and are very unlikely to be the diagnosis in this patient who has no pain on defecation.

Option D: Anorectal fistulae are associated with soiling rather than rectal bleeding. It is unlikely that painless fresh rectal bleeding is the only presentation of anorectal fistulae.

Addendum:
Faecal soiling occurs when sticky faeces stays behind in the anorectal canal and gives a local reaction at the anodermal skin which results in itching, dermatitis and loss of small amounts of brown fluid.

48

Q

A 67-year-old man presents to your GP clinic with complaint of fresh rectal bleeding covering the outer surface of his stool for the past 3 weeks. He is otherwise in good health. Which one of the following is the most appropriate next step in evaluation?

A. Digital rectal exam (DRE).
B. Proctoscopy.
C. Flexible sigmoidoscopy.
D. Rigid sigmoidoscopy.
E. Colonoscopy.

A

A. Digital rectal exam (DRE).

Rectal bleeding is often caused by benign conditions such as hemorrhoids or anal fissures. However, it is possible that this assumption masks serious and more proximal pathologies such as colorectal cancer. To avoid missing patients with sinister conditions, either as the cause of bleeding or a concomitant problem, all such patients should be assessed even if based on the history the etiology is clear.

Approach to patients with rectal bleeding starts with a thorough history followed by physical exam. The physical examination should be directed at identifying possible or definite sources of bleeding and at finding worrisome distal lesions that may be detectable on examination. A detailed physical examination must include** external inspection of the anus and a digital rectal exam as initial investigations**.

Having the patient bear down during the examination may induce prolapse of a hemorrhoid or bleeding of a superficial lesion. Moreover, when available, office-based anoscopy or proctoscopy (option B) should be performed in patients who present with acute rectal bleeding. These are simple measures not requiring bowel preparation.

Sigmoidoscopy (option C and D) or colonoscopy (option E) is indicated in patients in whom initial assessment suggests more proximal lesions as the cause.

49

Q

A 38-year-old man underwent gastric bypass surgery due to obesity (BMI>42) a few months earlier. Now, he has presented with complaints of lightheadedness, palpitation, and diarrhea within the first hour of meals, especially after he takes milk, toast, or cereal. Which one of the following would be the most appropriate management option?

A. Addition of vitamin B12.
B. Endoscopy.
C. Diet counselling.
D. Esomeprazole.
E. Ranitidine.

A

C. Diet counselling.

The presentation and the history of gastric bypass surgery make dumping syndrome the most likely diagnosis. Dumping syndrome is the effect of altered gastric reservoir function, abnormal gastric motor function, and/or pyloric emptying mechanism.

In terms of temporal relation to eating, dumping syndrome can have early or late manifestations:

Early dumping
Symptoms of early dumping syndrome occur 30-60 minutes after a meal, and are hypothesized to result from accelerated gastric emptying of hyperosmolar contents into the small bowel. The hyperosmolarity of the bowel content leads to fluid shift from the intravascular compartment into the bowel lumen, resulting in rapid small bowel distention and increase in the frequency of bowel contractions.

With rapid shift of fluid into the intestinal lumen, circulating volume contraction follows, triggering a vasomotor response that presents with tachycardia, lightheadedness, etc

Late dumping
Late dumping occurs 1-3 hours after a meal, probably through development of hyperinsulinemic (reactive) hypoglycemia. Rapid delivery of a meal to the small intestine results in an initial high concentration of carbohydrates in the proximal small bowel and rapid absorption of glucose, hypoglycemia, and a hyperinsulinemic response. The high insulin levels stay for longer period and are responsible for the subsequent hypoglycemia.

The following diagram summarizes the mechanism through which the dumping syndrome develops: (see photo)

The clinical presentation of dumping syndrome can be categorized as abdominal versus vasomotor symptoms, and based on the time of onset as early versus late.

Early dumping systemic symptoms:

  • Desire to lie down
  • Palpitations
  • Fatigue
  • Faintness
  • Syncope
  • Diaphoresis
  • Headache
  • Flushing

Early dumping abdominal symptoms:

  • Epigastric fullness
  • Diarrhea
  • Nausea
  • Abdominal cramps
  • Borborygmi

These symptoms can be collectively referred to as dyspepsia.

Late dumping symptoms:
* Perspiration
* Shakiness
* Difficulty to concentrate
* Decreased consciousness
* Hunger

For patients with dumping syndrome, dietary modification is the first-line management option. The following pieces of advice are appropriate:

  • Daily energy intake is divided into 6 meals.
  • Fluid intake during and with meals be restricted and liquids be avoided for at least half an hour after a meal.
  • Avoiding simple sugars.
  • Milk and milk products are generally not tolerated and should be avoided.
  • Because carbohydrate intake is restricted, protein and fat intake should be increased to fulfill energy needs.

Additional points to consider:
Supplementation with dietary fiber has proven effective in the treatment of hypoglycemic episodes. Many medical therapies have been tested, including pectin, guar gum, and glucomannan. These dietary fibers form gels with carbohydrates, resulting in delayed glucose absorption and prolongation of bowel transit time.

Dietary change to a low-carbohydrate, high protein diet, as well as the use of alpha-glucosidase inhibitors, may be useful to control the symptoms of dumping. This is preferential to subtotal or total pancreatectomy in those persons with severe symptoms.

Most patients have relatively mild symptoms and respond well to dietary changes. In some patients with postprandial hypotension, lying supine for 30 minutes after meals may delay gastric emptying and also increase venous return, thereby minimizing the chances of syncope.

Gastrointestinal System/Adomen/Abdomen Wall Flashcards by Lubi Mupwaya (14)

50

Q

A 57-year-old man presents with constipation and abdominal distention for the past three days. On examination, a lump is noted in the left inguinal region. The lump has no cough impulse and is not tender or painful. Which one of the following is the best next investigation to consider in this patient?

A. Ultrasound of the lump.
B. Ultrasound of the abdomen.
C. Erect and supine x-ray films of the abdomen.
D. CT scan of the abdomen.
E. Biopsy.

A

C. Erect and supine x-ray films of the abdomen.

The lump can be a segment of a small bowel loop entrapped in the inguinal region, namely inguinal hernia. Absent cough impulse is a pointer towards incarceration of the hernia.

There are two points in the question. The first and most important one is the presence of constipation and abdominal distention consistent with bowel obstruction. The second point is the presence of an inguinal lump with no cough impulse, no pain, and no tenderness.

The history and physical findings make small bowel obstruction (SBO) the most likely diagnosis and the first condition to exclude. The initial imaging of choice in assessment of suspected bowel obstruction is plain X-rays films. At least two views are required: upright view and supine view. Plain abdominal films can confirm the diagnosis in most patients without the need for further diagnostic testing, but they can be equivocal in 20-30% of patients, or normal or misleading in 10-20%. Abdominal X-rays are always the first imaging step in diagnosis.

Currently the best and most accurate diagnostic test for SBO is abdominal CT scan (option D) with both intravenous and oral contrast. CT scan can simultaneously provide information about the presence, level, severity and cause of obstruction. In addition, other abdominal pathologies can be detected. In some cases, closed-loop or strangulating obstruction may be revealed. CT scanning has been replaced with barium follow-through, which previously was the gold-standard diagnostic test for SBO. Follow-through series are now reserved for cases in which CT is not diagnostic. The question, however, asks about the initial investigation not the most accurate one.

Abdominal ultrasound scan (option B) is sometimes used for assessment of bowel obstruction, and is the only modality with the ability to assess the bowel pristaltism. However, plain X-rays remain the standard initial investigation.

This lump is very likely to be an incarcerated inguinal hernia which justifies both the lump and the obstruction. However, other differential diagnoses such as an inguinal lymph node should be considered as well once SBO, as the most likely cause, is excluded. In that case, ultrasound of the lump (option A) and biopsy (option E) if the lump turns out to be a lymph node, are indicated.

51

Q

You prescribed triple therapy against Helicobacter pylori with clarithromycin, amoxicillin and esomperazole for a 42-year-old patient of yours after his test results came back positive for the infection and you considered it as the cause of his intractable dyspepsia. One week after completion of the treatment, he comes back with reduced yet persisting symptoms. You arrange for a breath urea test. The result of the test is positive indicating treatment failure. Which one of the following is the most likely cause of the failed treatment?

A. Resistance to clarithromycin.
B. Resistance to amoxicillin.
C. False positive breath urea test result.
D. Poor adherence to the therapy.
E. Misdiagnosis of the cause of dyspepsia.

A

A. Resistance to clarithromycin.

Triple therapy with clarithromycin, amoxicillin and a proton pump inhibitor (PPI) for 7-14 days is the regimen of choice for treatment of H.pylori infection with over 90% response. Clarithromycin is the most effective antibiotic against H.pylori.

The most common cause of treatment failure with this regimen is the increasing rates of resistance of H.pylori to clarithromycin. This resistance cannot be overcome by dose increase or lengthening the duration of therapy.

NOTE - In those with penicillin hypersensitivity, in whom amoxicillin cannot be used, metronidazole can replace amoxicillin. So the regimen would be a combination of metronidazole, clarithromycin and a PPI. In such cases, resistance to either metronidazole or clarithromycin, or even both, can be the cause of treatment failure.

52

Q

Which one of the following conditions is least likely to be associated with Helicobacter pylori infection?

A. Gastric carcinoma.
B. B cell lymphoma of MALT tissue.
C. Gastroesophageal disease (GERD).
D. Atrophic gastritis.
E. Peptic ulcer disease.

A

C. Gastroesophageal disease (GERD).

Option A: Correct- There is strong evidence that H. pylori contributes to the development of gastric cancer.

Option B: Correct- H. pylori infection can also lead to the development of mucosa-associated-lymphoid-tissue (MALT) lymphoma, a type of cancer of the stomach. Treatment and eradication of H. pylori infection can result in regression of this malignancy in up to 75% of cases.

Option C: Incorrect- To date, there has been NO study suggesting a strong link between H.pylori and GERD. Eradication of H.pylori has NOT shown effective in treatment of GERD.

Option D: Correct- Chronic H.pylori-induced inflammation can eventually lead to loss of normal gastric mucosal architecture, with destruction of gastric glands and replacement by fibrosis and intestinal-type epithelium. This process of atrophic gastritis and intestinal metaplasia occurs in approximately 50% of the H.pylori-colonized population.

Option E: Correct- Helicobacter pylori (H.pylori) has been strongly linked to the development of gastric and duodenal ulcers. Eradication of H. pylori can prevent ulcers forming. Indeed patients presenting with ulcers should be tested for H. pylori and treated because eradication of H. pylori in patients with pre-existing ulcers cures ulcer disease and can prevent most recurrences.

53

Q

Which one of the following is the most common type of esophageal malignancy in Australia?

A. Squamous cell carcinoma.
B. Lymphoma of the esophagus.
C. Adenocarcinoma.
D. Sarcoma.
E. Metastatic tumors.

A

C. Adenocarcinoma.

The epidemiology of esophageal carcinoma has markedly changed over the past several decades. Previously, squamous cell carcinoma was the most common type of esophageal malignancies worldwide (90-95% of cases); however, over the past 4 decades and in developed countries such as United States of America, UK, and Australia, the incidence of adenocarcinoma of the distal esophagus and gastro-esophageal junction has progressively increased. Currently, adenocarcinomas account for more than 70% of all cases of esophageal cancers in Australia. Barrett’s esophagus caused by gastroesophageal reflux disease (GERD) is the most significant risk factor for development of esophageal carcinoma of adenocarcinoma type.

NOTE - Squamous cell carcinoma remains the leading type of esophageal carcinoma worldwide with smoking and chronic alcohol as the most significant risk factors.

54

Q

A 42-year-old man presents with 10-month history of nocturnal gastric acid reflux. He smokes 40 cigarettes a day, drinks alcohol on a regular basis, and 4 cups of coffee a day. He loves spicy foods and eats a lot of them. On examination, his height is 178 cm and weighs 110 kg. He has a waist circumference of 115 cm. Which one of the following is the most appropriate advice for him regarding long-term management of the condition?

A. He should stop drinking alcohol.
B. He should stop smoking.
C. He should avoid spicy foods.
D. He should reduce weight.
E. He should stop drinking coffee.

A

D. He should reduce weight.

Advice regarding lifestyle modification has been historically a part of management plan, in conjunction to antacid therapy, for gastroesophageal reflux disease (GERD). Mild cases will respond to simple measures (e.g., bed head elevation), while most will need proton pump inhibitors (PPIs) as the cornerstone of the treatment.

Lifestyle modifications that are often advised include:

  • Weight reduction
  • Bed head elevation during sleep
  • Avoiding spicy foods
  • Last meal not later than 2-3 hours before bed time
  • Avoiding or reducing alcohol and caffeine intake
  • Quitting smoking

Of these, however, only elevation of the head of the bed and weight reduction seems to have an effect more than placebo. While widely encouraged, other measures appear to be associated with improved overall health with no specific effect on GERD, at least not in all patients.

Gastro-oesophageal reflux disease

55

Q

A 71-year-old man is being evaluated for iron deficiency anemia. On initial laboratory tests, he has a positive fecal occult blood test (FOBT) result. An upper endoscopy is negative, and he is planned for colonoscopy. During the procedure, the colonoscope does not pass beyond the hepatic flexure. No abnormal finding is noted up to there. Which one of the following is the most appropriate next step in management?

A. Force the colonoscope to pass the hepatic flexure.
B. Tagged red blood cells.
C. Colon capsule endoscopy(CCE).
D. CT colonography(CTC).
E. Barium enema.

A

C. Colon capsule endoscopy(CCE).

In the elderly, gastrointestinal bleeding should be attributed to colorectal malignancies until proven otherwise. In such patients, the whole length of the colon should be thoroughly investigated with colonoscopy as the most appropriate diagnostic modality.

An incomplete colonoscopy examination may occur in up to 5-10% of cases due to patient discomfort, a colon with many twists, postoperative adhesions, or hernia. In such cases, either a CT colonography (CTC) or colonic capsule endoscopy (CCE) may be performed. The latter, based on recent studies, is associated with higher diagnostic value.

Option A: Forcing the colonoscope is likely to cause damages such as mucosal tear or even perforation.

Option B: Tagged red cells are used in cases of acute lower gastrointestinal bleeding when colonoscopy cannot spot the bleeding site. it is not useful for tumor detection.

Option D: Compared to first-generation capsules, CTC was superior to CCE, but with the advent of newer capsules, diagnostic value of CCE has surpassed that of CTC.

Option E: Double contrast barium enema was traditionally used in cases of incomplete colonoscopy, but retrospective studies demonstrated that this method could miss more malignant lesions than CTC and CCE; hence, these procedures have superseded double contrast barium enema.

56

Q

Kim, 9 years old, is diagnosed with hepatitis A after she developed jaundice and abdominal pain 8 days ago. Today, her mother calls and asks you when she can go back to school. Which one of the following would be your answer?

A. When her tests are normal.
B. She can go back to school now if there is a separate sanitation system.
C. She can go to school now.
D. After 4 to 6 months.
E. After one month.

A

C. She can go to school now.

The incubation period of hepatitis A is 15 to 50 days, but more commonly 28 to 30 days. The usual clinical presentation is acute fever, malaise, anorexia, nausea and abdominal discomfort (prodromal symptoms), followed by dark urine and jaundice a few days later. Symptoms usually last several weeks.

The likelihood that symptoms will follow infection increases with age: jaundice occurs in only a small proportion of infants and young children, but a majority of adults. Infants and children infected with HAV may have mild or no symptoms.

The viral load in the stool (and infectivity) is highest just before the onset of jaundice. Patients are considered infectious from a few days before onset of prodromal symptoms to a few days after onset of jaundice, and non-infectious one week after onset of jaundice (if it occurs) or two weeks (14 days) after onset of prodromal symptoms, whichever comes first.

Kim has had jaundice 8 days ago and she does not seem to be infectious anymore because more than 7 days has past since jaundice. She can go back to school now.

Serum IgM anti-HAV is the gold standard for diagnosis. The antibody becomes positive at the onset of symptoms, peaks during the acute or early convalescent phase, and remains positive for approximately 4 to 6 months. Liver function tests may also remain deranged beyond the infections period. Liver function tests or HAV antibodies are not appropriate indicators for infectivity.

NOTE - exclusion of contacts is not necessary.

57

Q

Which one of the following is the most likely presentation of a rectal cancer?

A. Melena, constipation and colicky lower abdominal pain.
B. Diarrhea, bloody stool and abdominal pain.
C. Constipation.
D. External hemorrhoids.
E. Fatigue.

A

B. Diarrhea, bloody stool and abdominal pain.

Bloody stool (rectal bleeding), bowel habit changes mostly in form of diarrhea or altered stool caliber, and abdominal pain are the most common presenting symptoms in order of incidence in rectal cancer.

Rectal cancers present with a variety of sign and symptoms including the following:

  • Rectal bleeding – bleeding is the most common presenting symptom of rectal cancer, occurring in 60% of patients. It should be noted, however, that profuse bleeding and anemia are rare. Occult bleeding detected through a fecal occult blood test (FOBT) is present in 26% of all cases.
  • Changes in bowel habits – this is the second most common presenting symptom, occurring in 43% of patients. Bowel habit changes is often in form of diarrhea rather than constipation. Some patients report a decrease in stool caliber. Large tumors can cause obstruction and obstructive symptoms. Tumors located low in the rectum can cause a feeling of incomplete evacuation and tenesmus.
  • Back pain is usually a late sign caused by a tumor invading or compressing nerve trunks.
  • Urinary symptoms may also occur if the tumor is invading or compressing the bladder or prostate.
  • Malaise is a nonspecific symptom and present in 9% of rectal cancer cases.
  • Bowel obstruction due to a high-grade rectal lesion is rare, occurring in 9% of all cases.
  • Pelvic pain is a late symptom, usually indicating nerve trunk involvement, and is present in 5% of all cases.
  • Other manifestations include emergencies such as peritonitis from perforation (3%) or jaundice, which may occur with liver metastases (< 1%).

Option A: Melena is the black stool caused by oxidation of blood in the GI. Melena is a feature in upper gastrointestinal rather than lower gastrointestinal bleeding, and not a presenting symptom in rectal cancers. Although a change in bowel habit is the second most presenting symptom of rectal cancers, this change is more often in form of diarrhea rather than constipation. Lower abdominal pain due to partial or complete obstruction can be a feature.

Option C: As mentioned earlier, diarrhea is more common than constipation in rectal cancers.

Option D: External hemorrhoids (a thrombosed anal vein) is not a common presenting complaint in patients with rectal cancer.

Option E: Fatigue is feature more commonly seen in right-sided colon cancers. As the blood mixes with the stool, bloody stool is a less frequently encountered presentation. On the other hand, bleeding from right-sided lesions result in diarrhea (blood is cathartic) and anemia. The latter is the main cause of fatigue in patients with right-sided malignant lesions. Fatigue or malaise is seen in only 9% of patients with rectal cancer.

58

Q

Which one of the following is the most common mode through which hepatitis B infection is transmitted in Australia?

A. Sexual intercourse.
B. Transfusion of blood and blood products.
C. Injecting drug use.
D. Vertical transmission from the mother to the infant.
E. Breast feeding.

A

A. Sexual intercourse.

Hepatitis B virus is a blood-borne virus, meaning that the virus should enter the blood stream in order to cause infection. Modes of transmission include:

  • sharing contaminated objects that pierce the skin or mucous membranes, such as needles, tattoo equipment, body-piercing equipment, acupuncture equipment, razor blades and toothbrushes
  • sexual contact (heterosexual or hom*osexual, more common with hom*osexual contact)
  • perinatal transmission from an infected mother to her infant (vertical transmission)
  • needlestick injuries
  • contact between infective fluid and mucous membranes, such as a splash of blood into eyes or mouth
  • invasive medical or dental procedures if there has been inadequate infection control
  • transfusion of infected blood or blood products
  • human bites and other direct contact with the blood or open spores of an infected person

Of these routes, some are more common than the others in different parts of the world, depending on the regional prevalence of the chronic hepatitis B infection:

  • Low-prevalence areas: Such areas include the United States, Canada, Western Europe, Australia, and New Zealand. The prevalence of hepatitis B in such areas is ≤2% (0.1- 2%). Less than 12% HBV infected individuals live in these areas. The most common route of transmission in these countries are sexual transmission and percutaneous transmission.
  • Intermediate-prevalence areas: Sexual and percutaneous transmission and transmission during delivery are the major transmission routes in areas of intermediate prevalence (rate of 3-5%). These regions include Eastern and some areas of Northern Europe, Japan, some Mediterranean countries, Latin and South America, and Central Asia.
  • High-prevalence areas: Such areas include China, Southeast Asia, sub-Saharan Africa, Pacific Islands, Parts of Middle East and Amazon Basin. The prevalence in ≥8% (often 10-20%). Perinatal infection including vertical transmission from the mother to the infant is the predominant mode of transmission.

Australia has a chronic hepatitis B prevalence of less than 0.5% and is among low-prevalence regions of the world. The most common mode of transmission of hepatitis B infection in Australia is heterosexual and hom*osexual sexual intercourse with the latter being associated with higher risk of transmission. This is followed by percutaneous transmission such as in injecting drug use and non-sterile procedures such as piercing, tattooing, etc.

Option B: In Australia, screening of blood and organ donors using serological, and subsequently nucleic acid amplification testing, has virtually eliminated the risk of transmission of hepatitis B through blood transfusion and organ transplants.

Option C: Injecting drug use predominates other modes of transmission in newly infected individuals with hepatitis C in Australia. It is also an important mode of transmission in hepatitis B after sexual intercourse.

Option D: Perinatal transmission including vertical transmission from the mother to the infant is the predominant route of transmission of hepatitis B infection in areas with high prevalence of HBV infection. In such areas the prevalence is ≥8% (generally 10-20%). These areas include China, Southeast Asia, Indonesia, sub-Saharan Africa, Pacific Islands, parts of the Middle East, and the Amazon Basin. Due to perinatal care and administration of hepatitis B vaccine within the 12 hours of birth, this route of transmission is not common in Australia.

Option E: Breast feeding does not appear to be a significant route of transmission, neither are fecal-oral and vector-borne modes.

NOTE - Saliva may contain levels of virus that are likely to be infective only if inoculated directly into tissue (ocular or mucous membranes). The risk of transmission by inadvertent inoculation by other means, such as by toothbrush, razor etc., or through close personal contact in households in which one or more infected persons reside, is low but not negligible.

59

Q

A 24-year-old man is being evaluated for dysphagia as his main chief complaint. He has past medical history of childhood asthma. Endoscopy is arranged showing narrowing and inflammation of middle esophagus. Biopsy is significant for the presence of eosinophils in the esophageal mucosa. Which one of the following is the most appropriate treatment option to consider for this patient at this stage?

A. Oral antifungal agents.
B. A trial of proton pump inhibitors.
C. Swallowed budesonide.
D. Systemic corticosteroids.
E. Triple therapy against H.pylori.

A

B. A trial of proton pump inhibitors.

Primary eosinophilic esophagitis (EoE) is an increasingly recognized medical condition characterized clinically by symptoms related to esophageal dysfunction, and histologically by eosinophilic inflammation in the esophagus.

EoE is hypothesized to be an atopic inflammatory disease caused by an abnormal immune response to antigenic stimulation, mostly foods. Normally, eosinophils are normal component of mucosal infiltrates in all-length of the gastrointestinal tract except the esophagus. Eosinophils in the esophageal mucosa are always pathologic.

Generally, the clinical symptoms of EoE are nonspecific, and the patients are in good physical condition resulting in a delayed diagnosis (years) in some cases.

The presenting symptoms vary depending on the age of the onset:
Children - children tend to present with nausea and vomiting, weight loss, anemia, and failure to thrive. In neonates and infants, refusal of food is the most common presenting symptom.

Adults - the characteristic symptoms in adults include dysphagia for solid foods, retrosternal pain and food impaction. Some patients also present with gastroesophageal reflux disease (GERD) symptoms unresponsive to medical anti-reflux therapy. A subset of patients have been recognized to have a typical clinical presentation of EoE in the absence of GERD who show a clinicopathologic response to PPIs. This condition is currently referred to as PPI-responsive EoE.

There is no Australian guidelines for diagnosis and management of EoE and current recommendation is based on the guidelines by the American College of Gastroenterology (ACG).

According to the ACG, diagnostic criteria for EoE include all of the following:

  • Symptoms related to esophageal dysfunction.
  • ≥15 eosinophils/hpf on esophageal biopsy
  • **Persistence of eosinophilia after a proton pump inhibitor (PPI) trial **
  • Secondary cause of esophageal eosinophilia excluded

This patient has symptoms related to esophageal dysfunction (dysphagia and odynophagia) and established eosinophilia on histological studies. In order for EoE to be the definite diagnosis, it is necessary that eosinophilia persists after an 8-week trial of a PPI.

The rationale behind this trial is that GERD may mimic EoE, coexist with it, or contribute to it. Conversely, EoE may contribute to GERD; therefore, the diagnosis of EoE is generally made after the symptoms persist after an 8-week course of proton pump inhibitors (PPIs) as the best initial step in management. PPIs may benefit patients with EoE either by reducing acid production in patients with co-existent GERD, or by other unknown anti-inflammatory mechanisms.

The main three components of treatment in established EoE are (1) dietary advice and alteration, (2) pharmacotherapy, and (3) surgical intervention. For pharmacological intervention topical swallowed steroids (e.g., fluticasone, budesonide) are considered the main treatment options, once the diagnosis of EoE is established either after failed PPI therapy or normal pH studies.

For patients unresponsive to the above measure, oral (systemic) predniso(lo)ne maybe indicated.

Option A: The condition is not infectious and antifungal or antimicrobial agents have no role in treatment.

Option C: Topical swallowed steroids (e.g., budesonide) are considered as the next step, once the diagnosis of EoE is established either after failed PPI therapy or normal pH studies.

Option D: Systemic corticosteroids are a treatment option for patients unresponsive to PPIs or swallowed corticosteroids.

Option E: Eradication of H.pylori have not shown efficacy in treatment of EoE.

TOPIC REVIEW
Causes of esophageal eosinophilia:

  • Eosinophilic esophagitis
  • GERD
  • PPI-responsive eosinophilic esophagitis
  • Achalasia
  • Crohn’s disease
  • Parasitic infections
  • Drug hypersensitivity
  • Connective tissue disease (e.g., scleroderma, dermatomyositis)
  • Celiac disease
  • Hypereosinophilic syndrome

60

Q

After evaluation of a 67-year-old woman with right upper quadrant abdominal pain, dyspepsia and weight loss, she is found to have a solitary tumor in the right lobe of her liver. CT scanning of the liver suggests the tumor to be malignant. Which one of the following could be the most likely diagnosis?

A. Hepatoma.
B. Metastasis from colon cancer.
C. Metastasis from stomach cancer.
D. Metastasis from breast cancer.
E. Metastasis from ovarian cancer.

A

A. Hepatoma.

Approximately 90% of malignant liver tumors are metastases from other primary malignancies elsewhere. Hepatoma, also known as hepatocellular carcinoma (HCC) is a primary liver tumor accounting for 10% of malignant liver lesions.

In metastatic liver disease, metastases are usually multiple rather than single; therefore, a solitary malignant liver tumor is more likely to have caused by HCC rather than metastasis.

61

Q

A 35-year-old woman presents to the emergency department with acute onset right upper quadrant pain which started one hour ago. When visited by the emergency physician, she mentions that the pain has gone away. An ultrasound scan of the liver and biliary tree reveals a simple cyst in the liver. Which one of the following is the most appropriate next step in management?

A. Reassure her.
B. Aspiration of the cyst.
C. Abdominal CT scan.
D. Laparoscopic removal of the cyst.
E. MRI of the liver.

A

C. Abdominal CT scan.

Simple liver cysts are often congenital and hypothesized to develop from intrahepatic biliary ducts because they have a lining similar to biliary ducts.

Simple cysts generally cause no symptoms but may produce dull right-upper-quadrant pain if large in size. Patients with symptomatic simple liver cysts may also report abdominal bloating and early satiety. Occasionally, a cyst is large enough to produce a palpable abdominal mass. Very rarely, jaundice caused by bile duct obstruction or rupture and acute torsion of a mobile cyst may occur. Patients with cyst torsion may present with an acute abdomen. When simple cysts rupture, patients may develop secondary infection, leading to a presentation similar to a hepatic abscess with abdominal pain, fever, and leukocytosis.

`It is very important to differentiate a simple cyst from malignant hepatic lesion. One differential diagnosis to exclude is a liver tumor with central necrosis that is often misdiagnosed as simple hepatic cyst. Genuine intrahepatic neoplastic cysts are rare.

If a cystic lesion is found on ultrasound, often contrast-enhanced CT scan must be considered to characterize the lesion.` Images are taken during the pre-contrast, arterial and portal venous phase.

Option A: Reassuring a patient with a symptomatic cystic lesion is not appropriate unless further investigations are conducted the other causes of such presentation has been excluded. Symptomatic hepatic cysts usually require treatment.

Option B: Percutaneous aspiration guided by ultrasonography or computed tomography (CT) is technically simple but has been abandoned because the recurrence rates are nearly 100%. Aspiration combined with sclerosis with alcohol or other agents has been successful in some patients but has high failure and recurrence rates. Successful sclerosis depends on complete decompression of the cyst and apposition of the cyst walls. This is not possible if the cyst wall is thickened or if the cyst is large.

Percutaneous catheters should not be placed to drain simple cysts, because the cavity becomes contaminated, leading to the development of hepatic abscess. Unlike the typical pyogenic hepatic abscess, this problem is difficult to resolve with repeated catheter placements because of continued secretion from the cyst epithelium.

Option D: Laparoscopic unroofing of the cyst by excising the portion of cyst wall that extends to the surface the liver is the treatment of choice for symptomatic liver cysts. This, however, should be considered once further investigations establishes the diagnosis of a simple cysts and excludes other differential diagnoses.

NOTE - No medical therapy has proved effective in reducing the size of simple hepatic cysts.

Option E: MRI has a limited role in evaluation of simple cysts and is often unnecessary.

62

Q

A 35-year-old man is brought for assessment by his wife with complaints or frequent episodes of confusion, dysarthria and wide-based gait. He is an office manager and has been behaving peculiarly recently at work. Of importance, his father and paternal aunt have died with the same symptoms at ages of 50 and 53, respectively. Which one of the following is most likely to make a diagnosis?

A. CT scan of the head.
B. MRI of the head.
C. Serum copper and ceruloplasmin level.
D. Lumbar puncture.
E. Liver function tests.

A

C. Serum copper and ceruloplasmin level.

Neuropsychiatric problems in a young patient in the presence of a family history of such presentation is suggestive of Wilson disease. Wilson disease is a rare (3 in 100000) inherited disease (autosomal recessive) which is caused by accumulation of copper in the liver and central nervous system. The genetic basis is a mutation of a gene on chromosome 13 that codes for a copper transporting ATPase, ATP7B. Over 200 mutations have been identified so far.

Total body copper content is 100-150mg with an average intake of 1-5mg/day. The dietary copper is absorbed in the proximal small bowel. Copper is incorporated into ceruloplasmin in the liver. In Wilson disease copper absorption and transport in the liver are intact but incorporation into ceruloplasmin in the hepatocytes and biliary excretion is impaired. This results in accumulation of copper in the liver and later on other organs (e.g. CNS). The most common presentations are with liver disease or neuropsychiatric disturbances. Asymptomatic patients are most often detected by family screening.

The clinical hallmark of Wilson’s disease is the corneal Kayser–Fleischer ring, which is present in 95% of patients with neurologic symptoms and somewhat over half of those without neurologic symptoms.

While most affected children present with hepatic involvement (hepatitis, cirrhosis, or fulminant liver failure) adults often present with neuropsychiatric manifestations as the presenting symptoms.

Neuropsychiatric manifestations of Wilson disease include dysarthria, dysphagia, dyskinesias, purposeless stereotyped movements, dementia, Parkinsonism, micrographia, clumsiness. There can be also decreased memory, anger outbursts, labile emotions, decreased libido and personality changes.

Other possible clinical feature are:

  • Coombs negative hemolytic anemia
  • Polyarthritis
  • Hypermobile joints
  • Grey skin
  • Abortions
  • Hypoparathyrodism

Diagnostic tests include serum copper (increased) and ceruloplasmin (decreased). It should be noted, however, that ceruloplasmin is an acute phase reactant and may be increased in response to hepatic inflammation, pregnancy, estrogen use, or infection.)

The urinary copper excretion rate in patients with Wilson disease is greater than 100 mcg/d (normal < 40mcq/day). Although the sensitivity and the specificity of this test are suboptimal for use as a screening test; it may be useful to confirm the diagnosis and to evaluate the response to chelation therapy.

NOTE - The criterion standard for diagnosis of Wilson disease is liver biopsy. Copper levels of more than 250mcq per each gram of dry weight of the liver in an asymptomatic patient establishes the diagnosis.

Chelation with D-penicillamine or trientine are mainstay of therapy. Patients should be on diets with decreased sources of dietary copper.

CT scan of the head (option A) or preferably MRI of the head (option B) are used for the degree of brain involvement in Wilson disease.

Option D: Lumbar puncture has no role in diagnosis of Wilson disease.

Option E: Liver function tests are impaired in most patients with Wilson disease. These changes are, however, nonspecific and can be seen in a variety of other liver diseases; hence, not diagnostic.

63

Q

A 57-year-old man is brought to the Emergency Department of the local hospital you are working in after an episode of vomiting large amount of fresh blood. A couple of hours before presentation, he had binge drinking which was followed by retching several times and vomiting before he vomits blood. This is the first time he is having such problem. He has enjoyed good health otherwise and has not had any complaints except indigestion after meals for which he is taking over-the-counter antacids. He does not smoke but drinks equivalent to 30 grams of alcohol every day. Which one of the following could be the most likely cause of his presentation?

A. Duodenal ulcer.
B. Gastric cancer.
C. Gastric ulcer.
D. Mallory-Weiss tear.
E. Esophagitis.

A

D. Mallory-Weiss tear.

Although peptic ulcer disease including duodenal ulcers (option A) and gastric ulcers (option C) is the most common cause of bloody vomiting, the fact that the bleeding has followed frequent retching, makes Mallory-Weiss tear more likely as the diagnosis.

NOTE - Without the retching in history, peptic ulcer disease would have been the most likely diagnosis.

Mallory-Weiss tear is a linear laceration at the gastroesophageal junction. This tear is hypothesized to occur either by a rapid increase in intragastric pressure and distention, which increases the forceful fluid ejection through the esophagus, or secondary to a significant change in transgastric pressure, which is a difference in pressure across the gastric wall due to negative intrathoracic pressure and positive intragastric pressure. Such change leads to distortion of the gastric cardia, resulting in a gastric or esophageal tear.

Mallory-Weiss tears are the cause of hematemesis in 3-5% of cases in adults. The condition is frequently encountered in binge alcohol drinkers, mostly due to repeated retching and/or vomiting; however, even one single episode of retching or vomiting can result in the tear.

Hematemesis is the presenting symptom in virtually all patients with Mallory-Weiss tears. The diagnosis is not dependent on the amount of hematemesis because it can vary from blood flecks or streaks of blood mixed with gastric contents or mucus to a significantly large amount of bright red bloody emesis. Melena is another presentation reported to occur in as many as 10% patients. Other possible symptoms include:

Patients with severe vomiting can develop light-headedness or dizziness, or even syncope. These symptoms usually occur secondary to dehydration from the underlying cause of vomiting and are not secondary to blood loss from the Mallory-Weiss tear. Only in rare cases a Mallory-Weiss tear lead to anemia requiring transfusions.

Abdominal pain and dyspepsia - As many as 40% of patients may experience epigastric pain or heartburn. These symptoms are often related to the underlying cause of vomiting and not specifically to the Mallory-Weiss tear.

Option B: vomiting of a large amount of fresh blood is an unusual presentation for gastric cancer. Additionally, if the cancer was the cause, features such as weight loss as well as other cancer-associated symptoms would have been expected.

Option E: Esophagitis can result in a frail mucosa that bleeds easily; however, symptoms related to such bleeding are expected to be iron deficiency anemia (in the long run) and melena (or occult blood in the stool) rather than copious bloody vomitus.

Mallory-Weiss Syndrome

64

Q

A 45-year-old patient presents to your GP clinic with complaint of dyspepsia. Evaluations unveil peptic ulcer disease and H.pylori infection. You start him on a 14-day course of triple therapy with amoxicillin, metronidazole and pantoprazole. Ten days after completion of the treatment, the patient is only partially improved and the dyspepsia still exists, though to a lesser extent. You arrange for a breath urea test which turns out positive. Which one of the following could be the most likely cause of treatment failure in this patient?

A. Amoxicillin resistance.
B. Metronidazole resistance.
C. Inappropriate timing of the test after treatment.
D. Unreliable urea breath test.
E. Patient’s non-compliance to treatment.

A

B. Metronidazole resistance.

The most common cause of treatment failure against H.pylori infection is antibiotic resistance.

Antibiotics used in multiple drug regimens against H.pylori are clarithromycin, metronidazole, and amoxicillin. Of these three, resistance to metronidazole and clarithromycin are the most common causes of treatment failure.

While resistance to metronidazole can be overcome by increasing the dose of metronidazole and duration of therapy, this method is not effective where clarithromycin resistance is the issue.

Option A: H.pylori is still susceptible to amoxicillin and resistance to this antibiotic has not been reported as a cause of treatment failure.

Option C: A breath urea test is often performed within 4-6 weeks of the commencement of therapy. This patient has presented 10 days after completion of a 14-day course of antibiotics (so 24 days after the commencement of treatment) and nearly about the time of the test.

Option D and E: Patients’ non-compliance or an unreliable test result with a standard kit are less likely explanations for this patient’s persisting symptoms.

65

Q

A 27-year-old woman presents to the Emergency Department with severe perianal pain started 10 hours ago that has increased progressively since then. The pain is exquisite and the patient is unable to sit. The anal region is shown in the following photograph. Which one of the following would be the most important part of management for this patient?

A. Incision under local anesthesia.
B. Hemorrhoidectomy under general anesthesia.
C. Topical antihemorrhoid agents.
D. Antibiotics.
E. Hemorrhoidectomy under topical anesthesia.

Gastrointestinal System/Adomen/Abdomen Wall Flashcards by Lubi Mupwaya (15)

A

A. Incision under local anesthesia

The picture shows an erythematous raised area near the anus almost covering it. Such appearance, as well as the symptoms and clinical findings, is consistent with a perianal abscess as the most likely diagnosis. Physical examination in patients with perianal abscess demonstrates an erythematous, well-defined, fluctuant, subcutaneous mass near the anal orifice. Vital signs are often within the normal ranges.

As a rule, abscesses have to be treated with incision and drainage. Simple perianal abscesses can be treated in the emergency department under local anesthesia followed by administration of systemic antibiotics. More complex perianal abscesses may need treatment in the operating room.

(Options B, C and E) Hemorrhoids have a different clinical picture depending on their grade. Bright rectal bleeding is the most common symptom in internal hemorrhoids. Other manifestations include pruritus, mucous discharge and anal pain and discomfort. A thrombosed anal vein (formerly called external hemorrhoid) presents with a painful and tender bluish mass protruding out of the anus. This patient does not have hemorrhoid; therefore, any option suggesting treatment of hemorrhoids is incorrect.

(Option D) Antibiotics alone are inadequate for treatment of a perianal abscess. Clinical suspicion of a perianal abscess warrants surgical incision and drainage. Delaying surgical intervention results in chronic tissue destruction, fibrosis, and stricture formation that may result in impaired anal continence.

  • Medscape - Anorectal Abscess

66

Q

A 32-year-old woman, who has just recently migrated from Somali to Australia, presents to your practice with complaint of severe anal pain, particularly on defecation, for the past 3 months. She mentions that at times she notices blood streaking her stool or dripping in the toilet bowl after a painful bowel movement. Examination, along with the history, confirms the diagnosis of anal fissure. Which one of the following options is the most expected predisposing factor for this presentation in this patient?
A. Rectal cancer.
B. Hemorrhoids.
C. Perianal abscess.
D. Rectal schistosomiasis.
E. High-fiber diet.

A

D. Rectal schistosomiasis

Rectal cancer (option A) can cause altered bowel motions and bleeding. Anal fissures are not a feature of rectal cancer though. Given the patient is young and has anal fissure, rectal cancer is unlikely to be the predisposing factor to this presentation.

Hemorrhoids (option B) present with painless rectal bleeding, and there is no pain on defecation. Although chronic constipation can be a predisposing factor for both hemorrhoids and anal fissure, hemorrhoids do not cause anal fissure.

Perianal abscesses (option C) can present with painful or tender swellings around the anus. Although they might cause pain on defecation as the feces passes and compresses an abscess, they do not cause rectal bleeding or anal fissure. Perianal fistulae, however, can develop as a consequent of perianal abscess.

High-fiber diet (option E) do not predispose to anal fissure. In fact such diet, by constipation prevention, is a management step in both hemorrhoids and anal fissures

Given the fact that the patient is an immigrant from Somali, where schistosomiasis is endemic, rectal schistosomiasis is most likely to have predisposed to the anal fissure and such presentation.

Schistosomiasis is either intestinal or urogenital, depending on where the adult flukes are located. In intestinal schistosomiasis, adult worms occupy mesenteric veins, and their eggs pass into the lumen of the intestine and reach the faeces. In the Eastern Mediterranean Region, Somalia and Sudan remain the most endemic countries.

Gastrointestinal schistosomiasis due to S. mansoni, S. japonicum and S. mekongi can cause bowel lesions such as ulceration, pseudopolyps, and microabcesses. Gastrointestinal schistosomiasis clinically presents with abdominal pain, altered bowel habits, and blood in stool. Liver enlargement and periportal fibrosis are common in advanced cases, and is generally linked with ascites and portal hypertension. Other well-documented clinical signs include superficial abdominal blood vessel dilatation, spleen enlargement, and bleeding- prone esophageal varices.

Patients with severe hepatosplenic schistosomiasis may die from ruptured esophageal varices. Some studies suggest an association of S. mansoni and S. japonica and cancer development in the liver and colon, but no hard evidence exists to support the association.

Chronic schistosomiasis commonly affects the sigmoid colon and rectum and usually presents with abdominal pain, diarrhea, bleeding, or ulcerations. Given its location of infection, it can also lead to anal fistulas, fissures, or abscesses.

  • WHO – Schistosomiasis

67

Q

Tim, 46 years old, is an established case of AIDS who has presented with a complaint of painful swallowing. The condition started two months ago and has been progressive. Endoscopy is arranged and reveals linear large ulcers in the middle and distal parts of the esophagus. A biopsy of the abnormal mucosa is remarkable for tissue destruction and the presence of intranuclear and intracytoplasmic inclusion bodies. Which one of the following would be the most appropriate pharmacological treatment option for him?

A. Proton pump inhibitors (PPIs).
B. H2-receptor antagonists.
C. Swallowing inhaled corticosteroids.
D. Ganciclovir.
E. Acyclovir.

A

D. Ganciclovir

Individuals with HIV infection are at an increased risk for esophagitis caused by cytomegalovirus (CMV) as an AIDS-defining illness when the CD4+ lymphocyte counts decline below 100 cells/µL.

Such as in any other type of esophagitis, CMV esophagitis presents with odynophagia (pain on swallowing) as the main symptom. The most constant endoscopic finding in CMV esophagitis is mucosal ulceration. The ulcers may be single or multiple, deep, or shallow, and could be several centimeters or more in diameter. The ulcers mostly affect the middle and distal parts of the esophagus.

Histopathologic findings include:

  • Basophilic inclusion bodies in the nucleus and cytoplasm (intranuclear and intracytoplasmic inclusion bodies)
  • Arrogates of macrophages

The treatment of choice for CMV esophagitis is ganciclovir or valganciclovir, usually in intravenous form. Foscarnet is used as an alternative if resistance occurs.

Acyclovir (option E) is not potent enough for the treatment of CMV esophagitis.

Combination therapy with IV ganciclovir and IV foscarnet is used when monotherapy fails; however, this regimen is associated with significant toxicity. Longer-term oral ganciclovir is considered for patients who require long-term suppressive treatment, and patients with HIV infection with CD4+ lymphocyte counts of less than 50 cells/µL.

PPIs (option A) and H2-receptor antagonists (option B) (e.g., ranitidine) are used for reflux disease, peptic ulcer disease, or gastritis none of which is the case here.

Swallowing inhaled corticosteroids (option C) is the treatment of choice, after a trial of PPIs, for eosinophilic esophagitis. The clinical picture does not fit eosinophilic esophagitis.

Medscape - Cytomegalovirus Esophagitis
NIH - Cytomegalovirus Esophagitis

68

Q

A 56-year-old man has been brought to the Emergency Department with a 12-hour history of abdominal pain and distention since this morning. He mentions no bowel movement or gas pass since last night, and describes the pain as colicky and mostly around the umbilicus. He has vomited 3 times since this morning. On examination, he has stable vital sounds. On abdominal exam, there is moderate abdominal distention with no tenderness, rigidity, guarding or rebound tenderness. On auscultation, hyperactive bowel sounds are noted. Digital rectal exam reveals an empty rectum with no blood or tenderness. There are dark spots on his lips and buccal mucosa. He remembers having these spots since childhood. The spots on his lip are shown in the following photograph. Which one of the following could be the most likely diagnosis?

A. Cecal cancer.
B. Acute appendicitis.
C. Sigmoid volvulus.
D. Intussusception.
E. Bowel perforation.

Gastrointestinal System/Adomen/Abdomen Wall Flashcards by Lubi Mupwaya (16)

A

D. Intussusception.

The dark spots on the lips and buccal mucosa of this patient is highly suggestive of Peutz Jeghers syndrome.

**Peutz Jeghers syndrome (PJS) is an autosomal dominant inherited condition characterized by freckling, gastrointestinal polyposis, and increased risk of certain cancers. **

Features seen in PJS are:

  1. Freckling - Most people with PJS have flat deeply pigmented spots around and inside their mouth, on their lips, fingers and toes. Less commonly, these spots can be seen on hands and feet, in the mucosa of the nose, conjunctiva or rectum. Freckles are benign and develop early in childhood and may disappear into adulthood.
  2. Polyposis - Patients with PJS will typically develop dozens to thousands of hamartomas, usually beginning in childhood. These polyps primarily develop in small bowel (jejunum, ileum and duodenum); however, they can also occur anywhere in the in the gastrointestinal (GI) tract.
  3. GI problems - GI symptoms arise mostly from the complications of hamartomas. Bleeding can occur as a GI hamartoma rubs against intestinal lining. GI bleeding in PJS may preset as blood in stool either occult or overt, anemia, etc. Bloating and abdominal pain may be seen.

One important well-known complication in GI polyps of PJS is intussusception (infolding of one part of the intestine into another) of the small bowel.

Other - Very early puberty can occur and occasionally male patients develop gynecomastia.

The disease is different in each patient meaning that not all patients show all of the features of the disease or any particular feature to the same extent as other patients

NOTE - PJS is associated with an increased risk of specific cancers such those of stomach, small intestine, colon, pancreas, breasts, ovaries, cervix or testes. Not everyone with PJS will develop cancer, but those affected are at an increased risk and with a younger age of onset than those in the general population.

This patient has the clinical picture of small bowel obstruction. Given the PJS in history, small bowel intussusception would be the most likely diagnosis. Small bowel obstruction presents with central colicky abdominal pain, early development of nausea and vomiting, mild to moderate abdominal distention, and constipation/ obstipation. As the condition deteriorates, bowel ischemia, necrosis, perforation, and peritonitis can develop.

(Option A) Although people with PJS are at increased risk of GI cancers at younger ages, cecal cancer is a less likely diagnosis in this scenario. Cecal cancer presents with fatigue due to chronic blood loss rather than signs and symptoms of bowel obstruction.

(Option B) Acute appendicitis can also present with periumbilical pain that gradually shifts and fixes in the right lower quadrant. Nausea and probably vomiting may follow the pain. The patient usually has anorexia and regional tenderness and often rebound in the right lower quadrant. However, bowel obstruction, as seen in this patient, is not a feature.

(Option C) In sigmoid volvulus, the sigmoid colon twists around the mesenteric axis and causes bowel obstruction. Abdominal distention is a prominent feature as are other signs and symptoms of large bowel obstruction such as an empty rectum on DRE. Sigmoid volvulus is typically seen in elderly patients who are bed-ridden and have chronic constipation. The location of the pain, the age of the patient, and the quick start of the nausea and vomiting are against large bowel obstruction on this patient. Moreover, with Peutz Jeghers syndrome, intussusception of the small bowel and small bowel obstruction are stronger possibilities.

(Option E) Bowel perforation presents with signs and symptoms of peritonitis such as abdominal tenderness and rebound tenderness, board-like rigidity, and guarding in an ill and toxic patient.

  • Cancer Council – Peutz-Jeghers syndrome * Medscape – Peutz Jeghers Syndrome

69

Q

Which one of the following is the most important diagnostic feature of achalasia?

A. Dysphagia for solids.
B. Dysphagia for liquids.
C. Dysphagia for both solids and liquids.
D. Regurgitation.
E. Weight loss.

A

C. Dysphagia for both solids and liquids

Achalasia is caused by lack of a relaxation of the lower esophageal sphincter and aperistalsis of the esophageal body. The condition is characterized by degeneration of the nerves innervating the esophagus of unknown cause.

Symptoms of achalasia include the following:

  • Dysphagia – the most common symptom: like other motility disorders of the esophagus there is dysphagia for both liquids and solids at the same time.
  • Regurgitation – regurgitation of achalasia is worse at night when the patient is in lying position, but that of retropharyngeal pouch (Zenker’s diverticulum) occurs even when the patient is standing.
  • Chest pain
  • Heart burn
  • Weight loss – weight loss can occur with all types of dysphagia

Of the symptoms, dysphagia to both liquids and solids is the cardinal and the most common symptom of achalasia.

(Option A) Dysphagia for solids as the prominent feature is characteristic of conditions that cause a narrowing within the lumen such as a tumor.

(Option B) Dysphagia for liquids as the cardinal symptom seen in oropharyngeal dysphagia such as in Zenker’s diverticulum, Parkinsonism, stroke and myasthenia gravis.

(Option D) Regurgitation in recumbency is also a feature of achalasia but it is not as common as dysphagia (~40% vs. 80%). Regurgitation is also seen in other conditions such as retropharyngeal pouch and is not a feature specific to achalasia.

(Option E) Weight loss can be a manifestation of all esophageal diseases. It is not an exclusive feature of achalasia.

70

Q

In which one of the following conditions of the colon, occurrence of malignant changes is most likely?

A. Adenomatous polyp.
B. Melanosis coli.
C. Diverticulitis.
D. Ulcerative colitis.
E. Familial polyposis coli.

A

E. Familial polyposis coli.

Of the given options, Familial polyposis coli has the strongest association with colorectal cancer and is very significant risk factor for that.

Familial polyposis coli, also termed familial adenomatous polyposis (FAP) is caused by mutations in the APC gene that a person inherits from his or her parents. About 1% of all colorectal cancers are caused by FAP. In the most common type of FAP, hundreds or thousands of polyps develop in a person’s colon and rectum, often starting at ages 10 to 12 years. Cancer usually develops in 1 or more of these polyps as early as age 20 years. By age 40, almost all people with FAP will have colon cancer if their colon hasn’t been removed to prevent it. People with FAP also have an increased risk for cancers of the stomach, small intestines, pancreas, liver, and some other organs.

(Option A) Adenomatous polyp: About 70 % of all polyps are adenomatous, making it the most common type of colon polyp. When this type of polyp is found, it is tested for cancer. Only a small percentage actually become cancerous, but nearly all malignant polyps began as adenomatous.

(Option B) Melanosis coli: is a condition that can be caused by chronic laxative abuse. Studies have not found an association between melanosis coli and an increased risk of colorectal cancer in humans. Or it may be that the factors that contributed to constipation itself, such as a diet that’s too low in fiber, are the reason why this possible higher risk of cancer exists

(Option C) Diverticulitis: the risk of colorectal cancer is increased in the short-term period after hospitalization related to diverticular disease. According to a recent systematic review and meta-analysis, the prevalence of colorectal cancer is 1.6% in patients with acute diverticulitis who underwent colonoscopy.

(Option D) Ulcerative colitis: The risk of colorectal cancer for patients with ulcerative colitis is increased, and is estimated to be 2% after 10 years, 8% after 20 years and 18% after 30 years of disease

  • Cancer Council Australia – Familial Bowel Cancer

71

Q

A 57-year-old man presents to your GP clinic with complaints of dysphagia and hoarseness. The first symptom was hoarseness, which started almost one month ago, followed by development of dysphagia after 2 weeks. His past medical history is significant for surgery for achalasia 10 years ago. On examination, normal-appearing but paralyzed vocal cords are noted. A CT scan is arranged that shows a mass in the thoracic inlet. Which one of the following could be the most likely diagnosis?

A. Stricture formation due to the previous surgery.
B. Thyroid cancer.
C. Esophageal cancer.
D. Laryngeal cancer.
E. Lung cancer.

A

C. Esophageal cancer

Dysphagia and hoarseness can be features seen in esophageal cancer, thyroid cancer or laryngeal cancer. In esophageal cancer, the tumor growth obstructs the esophagus. Dysphagia often occurs to solid foods first, followed by to liquids, as the lumen progressively becomes narrower. Dysphagia often becomes a symptom once the lumen diameter is less than 13 mm. In esophageal cancer, hoarseness occurs if the tumor invades the recurrent laryngeal nerve. This is an indicator of advanced disease and unresectability.

A malignant thyroid tumor can also invade the recurrent laryngeal nerve, and cause hoarseness. Dysphagia can also occur due to impingement of the proximal esophagus.

Laryngeal cancer, by invasion to structures within the larynx, causes hoarseness or aphonia. Once large enough in size, a laryngeal tumor can compress the proximal esophagus and cause dysphagia as a symptom.

Of these three, and given the history of achalasia as a risk factor for esophageal cancer and the presence of a mass in the thoracic inlet on CT scan, esophageal cancer is the most likely diagnosis compared to thyroid and laryngeal cancers. Because a stricture from the previous surgery (option A) in esophagus can explain the dysphagia but not the hoarseness.

(Option B) As a malignant thyroid tumor grows, it can compress the adjacent structures in the neck. It can also invade the laryngeal nerve and cause hoarseness. However, a thyroid cancer is most likely to present with a thyroid nodule. It is unusual for a thyroid tumor to be so large to compresses the esophagus but is missed on physical examination of the neck.

(Option D) Although laryngeal cancer can cause hoarseness and, if large enough, dysphagia, a neck mass would be expected on imaging. This patient has a mass in the thoracic inlet. This makes laryngeal cancer almost unlikely. Furthermore, the presence of previous achalasia as a risk factor for esophageal squamous cell carcinoma makes esophageal cancer a more likely diagnosis. Important risk factors for laryngeal cancer are smoking and alcohol consumption.

(Option E) An apical lung cancer can present with a mass in the thoracic inlet on imaging. A lung cancer can cause hoarseness but is unlikely to cause dysphagia.

TOPIC REVIEW
Common clinical manifestations of lung, laryngeal, esophageal, and thyroid cancers:

  1. Lung cancer
    - Cough
    - Chest pain
    - Shortness of breath
    - Coughing up blood
    - Wheezing
    - Hoarseness
    - Recurring infections such as bronchitis and pneumonia
    - Weight loss and loss of appetite
    - Fatigue
    - Metastatic signs and symptoms may include the following:
    - Bone pain
    - Spinal cord impingement
    - Neurologic problems such as headache, weakness or numbness of limbs, dizziness, and seizures
  2. Laryngeal cancer
    - Dysphonia/ aphonia
    - Dysphagia
    - Dyspnea
    - Aspiration
    - Blood-tinged sputum
    - Fatigue and weakness
    - Cachexia
    - Pain
    - Halitosis
    - Expectoration of tissue
    - Neck mass
    - Otalgi (ear pain)
  3. Esophageal cancer:
    - Dysphagia (most common); initially for solids, eventually progressing to liquids
    - Weight loss (second most common) due to dysphagia and tumor-related anorexia.
    - Bleeding (leading to iron deficiency anemia)
    - Epigastric or retrosternal pain
    - Bone pain with metastatic disease
    - Hoarseness (due to the involvement of the recurrent laryngeal nerve)
    - Persistent cough or frequent pneumonia due to tracheobronchial fistulas caused by direct invasion of tumor through the esophageal wall and into the main stem bronchus
  4. Thyroid cancer
    - A painless, palpable, solitary thyroid nodule (the most common presentation)
    - Hoarseness (suggests involvement of the recurrent laryngeal nerve and vocal fold paralysis)
    - Dysphagia (a sign of esophageal involvement)
    - Heat intolerance and palpitations (in autonomously functioning nodules)

References * Medscape – Esophageal Cancer * Medscape – Thyroid Cancer * Medscape – Malignant Tumors of the Larynx * Medscape – Non-Small Cell Lung Cancer

72

Q

A 43-year-old man presents to your Genral Practice with a midline abdominal swelling shown in the following photograph. The swelling is not painful but feels uncomfortable. It significantly decreases on lying down and accentuates when he coughs or raise his legs or shoulder while supine. On physical examination, the lump feels soft and non-tender. Which one of the following is the most appropriate next step in management?

A. Physiotherapy.
B. Abdominal truss.
C. Hernioplasty with mesh insertion.
D. Laparoscopic surgery.
E. Classic open abdominoplasty.

Gastrointestinal System/Adomen/Abdomen Wall Flashcards by Lubi Mupwaya (17)

A

A. Physiotherapy

The photograph shows a midline swelling starting from the umbilicus extending to halfway between the umbilicus and the xiphoid process of the sternum. The appearance alongside the history and physical finding makes abdominal recti diastasis (ARD) the most likely diagnosis.

Abdominal rectus diastasis is a condition where the abdominal muscles are separated by an abnormal distance due to widening of the linea alba, which causes the abdominal content to bulge. The rectus fascia is intact, and the condition should therefore not be confused with a ventral hernia. Rectus diastasis can be congenital but is most commonly acquired during pregnancies and/or larger weight gain causing laxity of linea alba.

ARD presents with visible bulging on exertion or when the intrabdominal pressure is increased (Valsalva maneuvers, coughing, straining, etc.) DRA often develops around the umbilicus but can also occur anywhere between the xiphoid and the pubic bone.

It is a common condition in pregnancy with an incidence as high as 66% in the third trimester. In 30–60% cases it persists post-partum. Not all patients with abdominal rectus diastasis are symptomatic. Symptoms, if present, include pain and discomfort in the abdomen, musculoskeletal problems like pelvic instability and lumbar back pain, and urogynecological symptoms such as urinary incontinence, fecal incontinence, and pelvic organ prolapse. In addition, patients with rectus diastasis also experience lower perception of body image, lower body satisfaction, and lower quality of life compared with the general population.

Patients with symptoms should receive treatment. Although the efficacy has not been studied adequately, treatment starts with physiotherapy of abdominal wall muscles.

If a course of months of physiotherapy fails to improve the condition surgical repair comes next. The procedure can be performed via classic abdominoplasty (option E) or endoscopic surgery and stabilization of the defect (option D).

Hernioplasty and mesh insertion (option C) is used for hernias. ARD is not a true hernia.

An abdominal (hernia) belt or truss (option B) is used to keep the content of a hernia inside the abdominal wall y application of pressure over the defect. They are used to relieve the symptoms of hernia, bridge the surgery, or provide additional support during recovery from the surgery. They are not used as a treatment modality of DRA.

  • Frontiers in surgery – Treatment Options for Abdominal Rectus Diastasis

73

Q

Brad is 55 years old and has presented to the Emergency Department with complaint of sudden onset abdominal pain. The pain was initially felt in the center of the abdomen but progressed to become generalized. He denies other symptoms including diarrhea, rectal bleeding, and nausea and vomiting. On physical examination, he has blood pressure of 145/95 mmHg, pulse rate of 120 bpm and irregular, respiratory rate of 18 breaths per minute, and temperature of 37.4°C. Abdominal examination, reveals mild generalized tenderness with no rebound, guarding, or rigidity. The rest of the exam in unremarkable. Which one of the following is most likely to assist to help with a diagnosis?
A. Abdominal ultrasound scan.
B. Abdominal CT scan.
C. Plain abdominal film.
D. Electrocardiogram (ECG).
E. Colonoscopy.

A

B. Abdominal CT scan

The scenario represents a case of acute-onset abdominal pain with minimal abdominal exam findings but the presence of a rapid irregular pulse suggestive of atrial fibrillation. This constellation of symptoms in a patient over the age of 50 years, especially if male, suggests acute mesenteric ischemia as a likely diagnosis.

Acute mesenteric ischemia (AMI) is interruption of intestinal blood flow by embolism, thrombosis, or a low-flow state. It leads to mediator release, inflammation, and ultimately intestinal infarction, bowel perforation and peritonitis.

The early hallmark of mesenteric ischemia is severe pain which is surprisingly out of proportion to abdominal physical findings. The abdomen is often soft, with mild or no tenderness at early stages. Later, as necrosis develops, signs of peritonitis appear, with marked abdominal tenderness, guarding, rigidity, and absent bowel sounds. The stool may become heme positive. The usual signs of shock develop eventually and frequently result is high mortality.

Cited causes for AMI are arterial embolus, atherosclerosis, non-occlusive ischemia such as in heart failure, shock, etc. In some patients, no cause is identified. AF is infamous cause of arterial embolus and a classic risk factor for AMI.

Early diagnosis of mesenteric ischemia is particularly important because mortality increases significantly once intestinal infarction sets in. Mesenteric ischemia must be considered in any patient > 50 with known risk factors or predisposing conditions who develops sudden, severe abdominal pain (AF in this scenario).

Patients with clear peritoneal signs should proceed directly to the operating room for both diagnosis and treatment. For others, selective mesenteric angiography or CT scan with angiography is the diagnostic procedure of choice. Other imaging studies and serum markers can show abnormalities but lack sensitivity and specificity early in the course of the disease when diagnosis is most critical. CT scanning has also the added benefit of assessment of other abdominal pathologies.

Plain abdominal films (option C) are useful mainly in ruling out other causes of pain (e.g., perforated viscus), although portal venous gas or pneumatosis intestinalis may be seen late in the disease.

Doppler ultrasonography can sometimes identify arterial occlusion, but sensitivity is low. MRI is very accurate in proximal vascular occlusion, less so in distal vascular occlusion. Serum markers (e.g., creatine kinase, lactate) rise with necrosis but are nonspecific findings that are seen later.

Abdominal ultrasound scan (option A) is the choice if conditions such as gallbladder disease or acute appendicitis suggested based on the presentation.

While an electrocardiogram (ECG) (option D) should be in the initial workup of this patient to further assess the rapid irregular rhythm as well as cardiac ischemic pain as a possible cause of abdominal pain, it is not diagnostic for AMI.

Colonoscopy (option E) is specifically valuable in assessment of intracolonic pathologies such as in gastrointestinal bleeding but is of little if any diagnostic value in this patient with suspected AMI.

References * Medscape – Acute Mesenteric Ischemia * John Murthag General Practice - Abdominal Pain

74

Q

Alfred is a 40 years old patient in your clinic, who has presented with heart burn and chest pain of 12 months duration. The chest pain is unrelated to physical activity, but both the heart burn and chest pain are worse with meals. He started developing difficulty swallowing to both liquids and solids and has lost 3 kg in this period. He also has regurgitation, specially when he lies down which he finds the most disturbing of his problems. He also mentions that rolling back his shoulders or raising his arms eases swallowing for him. Physical examination is unremarkable. Which one of the following could be the most likely diagnosis?
A. Esophageal cancer.
B. Esophageal strictures.
C. Scleroderma.
D. Esophagitis.
E. Achalasia.

A

E. Achalasia.

This scenario is a very typical picture of achalasia. Achalasia is a primary esophageal motor disorder of unknown etiology characterized manometrically by insufficient relaxation of the lower esophageal sphincter (LES) and loss of esophageal peristalsis; radiographically by aperistalsis, esophageal dilation, with minimal LES opening, ‘bird-beak’ appearance on barium studies, poor emptying of barium; and endoscopically by dilated esophagus with retained saliva, liquid, and undigested food particles in the absence of mucosal stricturing or tumor. Achalasia occurs equally in both genders with prevalence that ranges up to 1 per 10,000 persons. The majority of cases are idiopathic, but the syndrome can be associated with malignancy (especially involving the gastroesophageal junction) and as a part of the spectrum of Chagas disease. Rarely, achalasia is genetically transmitted.

In the early stages of the disease, dysphagia may be very subtle and misinterpreted as dyspepsia. The presence of heartburn due to food stasis can add to the diagnostic confusion. As achalasia progresses, dysphagia to both solids and liquids develops but it is worse to solids. To ease the progression of the food bolus, patients may modify their eating habits including eating more slowly or arching the back (rolling back of the shoulders) or raising the arms when swallowing.

Substernal chest pain during meals in the setting of dysphagia, weight loss, and even heartburn may be accompanying symptoms that often lead to misdiagnosis of achalasia as gastroesophageal reflux disease (GERD). Achalasia must be suspected in those with dysphagia to solids and liquids and in those with regurgitation unresponsive to initial trial of proton pump inhibitor (PPI) therapy.

NOTE - Of the symptoms, regurgitation is often more bothersome and more likely as a presenting symptom.

(Options A) occasionally, achalasia may be a result of esophageal malignancy, especially at the gastroesophageal junction. However, esophageal cancer in a young patient with no known risk factors is far less likely than idiopathic achalasia. Moreover, with malignancy, a more rapid course of dysphagia and a more profound weight loss compared to benign causes of dysphagia are expected.

(Option B) In esophageal stricture, dysphagia for solids occurs first, and dysphagia to liquids develop later as the stricture progressively narrows the esophageal lumen.

(Option C) Scleroderma (systemic sclerosis) is a complex and heterogeneous disease with clinical forms ranging from limited skin involvement (limited cutaneous systemic sclerosis) to forms with diffuse skin sclerosis and severe and often progressive internal organ involvement (diffuse cutaneous systemic sclerosis). Skin involvement (including Raynaud’s phenomenon) is almost always present at the time of presentation and diagnosis. Esophageal involvement in scleroderma may mimic the symptoms of achalasia; however, in the absence of other features of scleroderma, it is a remote possibility for this patient.

(Option D) Esophagitis (inflammation of the esophagus) is associated with pain on swallowing (odynophagia) rather than dysphagia.

  • UpToDate – Achalasia: Pathogenesis, clinical manifestations, and diagnosis * Gut and Liver – The Pathogenesis and Management of Achalasia: Current Status and Future Directions

75

Q

A 40-year-old man presents to your practice with an inguinoscrotal swelling. Which one of the following findings is consistent with the diagnosis of indirect inguinal hernia?

A. Swelling of the posterior part of the scrotum.
B. Swelling of the anterior part of the scrotum.
C. The examiner’s finger cannot get above the swelling.
D. The swelling is transilluminable.
E. The scrotum is painful.

A

C. The examiner’s finger cannot get above the swelling.

In indirect inguinal hernia, the bowel enters the scrotum through the inguinal canal. The hernia is often felt as a swelling filling the scrotum. Since the swelling originates from the abdomen and the inguinal area, the examiner’s finger cannot get above the swelling and this the most important finding that distinguishes a swelling arising from within the scrotum such as a testicular tumor, an epididymal cyst, hydrocele, sebaceous cyst from a swelling originating from out of scrotum.

(Options A and B) An indirect inguinal hernia fills the entire scrotum not just the anterior or posterior part of it.

(Option D) Although a gas-filled loop of bowel in the scrotal sac, as a result of indirect inguinal hernia, can be transilluminable, transillumination is more likely to due to a hydrocele. Even so, what differentiates between these two is that the examiner’s finger cannot get above the swelling in indirect hernia.

(Option E) A painful scrotum can be due to a variety of reasons such as epididymo-orchitis, incarcerated or strangulated indirect hernias, testicular torsion or trauma. Presence of pain in a patient with scrotal swelling is not diagnostic for an indirect inguinal hernia.

  • Medscape – Abdominal Hernias

76

Q

A 76-year-old man is brought to the Emergency Department with colicky abdominal pain and abdominal distention for the past 24 hours. He has not passed any stool or gas in this period. He gives the history of constipation for the past month. His past medical history is significant for appendectomy at the age of 42 years. On examination, he has stable vital signs. Abdominal exam reveals a distended abdomen, tympanic to percussion. No tenderness, rebound tenderness or guarding is elicited. She has also a reduceable mass in her left inguinal region suggestive of an inguinal hernia. Rectum is empty with no mass. Which one of the following can be the most likely cause to this presentation?

A. Sigmoid volvulus.
B. Sigmoid cancer.
C. Adhesions from previous surgery.
D. An obstructed hernia.
E. Fecal impaction.

A

A. Sigmoid volvulus.

The history of prolonged constipation, as well as the exam findings of a tympanic distended abdomen, colicky abdominal pain and an empty rectum is more consistent with sigmoid volvulus as the most likely diagnosis.

A colonic volvulus occurs when a part of the colon twists around its mesentery and causes colonic obstruction. Such obstruction can be acute, subacute or chronic. Although volvulus can occur in any portion of the large bowel, the sigmoid colon is the most frequently affected part, followed by volvulus of the right colon and terminal ileum, namely cecal or cecocolic volvulus. In very rare cases, volvulus may develop in the transverse colon or the splenic flexure.

Patients with colonic volvulus are commonly elderly, debilitated, and bedridden. A history of dementia or neuropsychiatric impairment is often present. The symptoms are acute in more than 60-70% of patients, while the rest present with subacute or chronic symptoms. A history of chronic constipation is common. The patient may describe previous episodes of abdominal pain, distention, and obstipation, which suggest repeated subclinical episodes of volvulus.

Regardless of its anatomic site, colonic volvulus presents the same way with cramping abdominal pain, distention, constipation and/or obstipation. Abdominal distention often increases progressively. The distention is characteristically tympanic over the gas-filled, thin-walled colon loop. Tenderness or rebound tenderness indicates that peritonitis has occurred or is imminent.

With progressive obstruction, nausea and vomiting occur. The development of constant abdominal pain is an ominous sign indicating the development of a closed-loop obstruction with significant intraluminal pressure. This leads to the development of ischemic gangrene and bowel wall perforation.

Depending on the extent of bowel ischemia or fecal peritonitis, signs of systemic toxicity may be present. Because of the massive abdominal distention, the patient may have respiratory and cardiovascular compromise.

Plain abdominal films are the initial imaging choice. Massive dilation of the sigmoid colon loop arising from the pelvis and extending to the diaphragm is a typical finding of sigmoid volvulus. The walls of the loop are evident as three bright lines converging in the pelvis to create a beak-like appearance.

Option B: A sigmoid tumor can also be the underlying cause of this clinical picture. Left-sided colon cancer can present with bloody stool, changes in bowel habits and abdominal pain specially if the tumor has caused partial or complete obstruction. However, a sigmoid tumor large enough to cause complete obstruction of the colon to result in obstipation is expected to have more pronounced exam findings.

Option C: Obstruction caused by the entrapment of a loop of the small bowel in an adhesion band formed from previous surgery can cause small bowel obstruction. In fact, adhesions from previous abdominal surgeries are the most common cause of small bowel obstruction. In this patient, however, the clinical picture more favors large bowel rather than small bowel obstruction because in small bowel obstruction nausea and vomiting are prominent features occuring earlier in the course of the process. This patient has not vomited until after 24 hours of symptoms onset.

The clinical presentation of small bowel obstruction includes the following:

  • Nausea/vomiting (60-80%): The vomitus can often be bilious in nature
  • Constipation/absence of flatus (80-90%): Typically, a later finding of SBO
  • Abdominal distention (60%)
  • Fever and tachycardia: Late findings and may be associated with strangulation

Option D: An obstructed hernia is expected to cause constant abdominal pain and tenderness. In early stages and before the strangulation occurs, there is no tenderness. This patient has a mass suggestive of hernia. However, the mass is reducible indicating that the hernia is not yet incarcerated or strangulated. A reducible hernia does not cause obstruction. Even in case of incarceration, it is the small bowel that becomes entrapped; therefore, symptoms would be expected to be in favor of small bowel obstruction. This patient has the clinical presentation of a large bowel obstruction.

Option E: As the name implies, this kind of obstruction is caused by impaction of fecal matter often in the rectum. An empty rectum excludes this diagnosis.

77

Q

A 72-year-old man is brought to the emergency department with abdominal pain. He reports constipation in the past two months and absolute constipation and not even gas pass since yesterday. On examination, the abdomen is mildly distended, tympanic to percussion, and there is mild tenderness in the lower abdomen in particular. On digital exam, the rectum is empty and otherwise normal. An abdominal X-ray shows sudden cut –off the sigmoid gas at its lower end and proximal dilation of the colon. Which one of the following is the most accurate tool to reach a diagnosis with high certainty?

A. CT scan of the abdomen.
B. Barium enema.
C. Colonoscopy.
D. Abdominal ultrasound scan.
E. Barium meal and follow through.

A

A. CT scan of the abdomen.

This patient is very likely to have bowel obstruction, and with the 2-month history of constipation and radiological finding of a sudden cut-off of the sigmoid and dilated proximal colon, sigmoid volvulus is the most likely diagnosis.

Once colonic volvulus is suspected based on the history and physical findings and the patient is stable, confirmatory imaging should follow for confirmation of the diagnosis and prompt action should follow to reduce the risk of complications (if not already developed). The initial imaging to consider is a plain abdominal x-ray. Abdominal x-rays reveal a distended loop of colon that may resemble a coffee bean or a dilated U-shaped colon with a cut-off point at the site of obstruction. Plain abdominal x-rays may also show distention of the small bowel with air-fluid levels and decompressed colon distal to the point of volvulus. Further diagnostic modalities to consider for more accurate diagnosis and determination of the exact anatomical site of obstruction are contrast-enhanced CT scan and water-soluble contrast enema.

Currently, contrast-enhanced abdominal CT scan is the preferred confirmatory diagnostic tool for both cecal and sigmoid volvulus because it is noninvasive, almost readily available, accurate for both cecal and sigmoid volvulus, and has the advantage of identification of incidental pathology that may be missed with plain radiographs or contrast studies. In addition, abdominal CT may facilitate the diagnosis of colonic ischemia (A is correct).

Option B: Barium, as a contrast matter, is contraindicated if bowel perforation is suspected. Although this patient does not have signs of bowel perforation or even ischemia, barium should be avoided. Should enema be considered as a diagnostic tool, a water-soluble contrast is the option.

NOTE - Avoid barium if viscus perforation is suspected or if the goal is to assess the post-operative leak.

Option C: Rigid or flexible sigmoidoscopy is the most appropriate initial treatment option for patients in whom bowel perforation or ischemia has been excluded with high certainty. After endoscopic detorsion, a rectal tube is left in place for 1 to 3 days to maintain reduction. Colonoscopy is only used in unusual cases in which the obstruction point is beyond the reach of shorter scopes. Definitive treatment is with surgery.

Option D: Ultrasound scan is not an appropriate diagnostic tool for suspected colonic volvulus. It has no benefit over plain radiograph for this purpose.

Option E: Barium meal and follow through is an investigation to consider for suspected small bowel obstruction.

78

Q

Robert, 54 years of age, has presented to your practice with complaint of painless intermittent rectal bleeding. You arrange for colonoscopy. After a good colonic preparation, the entire colon is visualized showing one pedunculated polyp in transverse colon, one in descending colon and one in the rectum. All three polyps are resected endoscopically and sent for histopathology exam. The polyp resected from the transverse colon shows high-grade dysplasia. Which one of the following would be the most appropriate investigation to consider for Robert?

A. CT colonography.
B. CT scan of the abdomen and pelvis.
C. Rectal ultrasound.
D. Surveillance colonoscopy in 3 years.
E. Serum alpha-feto protein (AFP) and carcinoembryonic antigen (CEA) tumor markers assay.

A

D. Surveillance colonoscopy in 3 years.

Although adenomatous polyps (adenomas) of the colon and rectum are benign growths arising from the colonic and rectal mucosa, there is a risk of progression to carcinoma in-situ and eventually to invasive colorectal cancer, therefore, surveillance is required.

The nature and interval of such surveillance depends on the size, number, and histopathological examination of the polyps (adenomas). As per current National Guidelines by the National Health and Medical Research Council (NHMRC), any colorectal polyps observed during colonoscopy should be resected and sent for histopathological exam, and surveillance program is considered.

Current Australian guidelines recommend the following surveillance program for colorectal polyps:

Group A (low risk) – any patient with the following falls into this group:

  • one to 2 adenomas; AND
  • all < 10mm; AND
  • no villous features; AND
  • no high-grade dysplasia

For group A surveillance includes colonoscopy at 5-year intervals, or fecal occult blood test (FOBT) every 1-2 years.

Group B (high risk) – any patient with any of the following falls into this group:

  • three to 4 adenomas; OR
  • any adenoma ≥10mm; OR
  • villous features; OR
  • high-grade dysplasia

For group B surveillance includes colonoscopy at 3-year intervals.

Group C (multiple) – this group includes patients with ≥5 adenomas.
For group C, surveillance includes colonoscopy at 1-year intervals if the number of adenomas is 5-9 and sooner if ≥10.

Group D (possible incomplete or piecemeal excision of large or sessile adenomas)
Surveillance in this group consists of colonoscopy at 3- to 6-months intervals until no residual adenoma is detected after that time surveillance continues at 1-year intervals.
—–
For Robert, either the number of adenomas (3) or the high-grade dysplasia histopathology of the adenoma resected from the transverse colon puts him in group B (high-risk); therefore, colonoscopic surveillance at 3-year intervals is the most appropriate investigation for him.

Option A: CT colonography is a new safe and well tolerated tool for diagnosis of colonic polyps and masses and, when performed by experienced hands, is as accurate as traditional colonoscopy. However, cost and training issues are needed to be addressed before it is widely accepted beyond the current indications of incomplete colonoscopy or patients who are unfit to undergo optic colonoscopy.

Option B and C: CT scan of the abdomen and pelvis is not an effective tool for surveillance of colorectal adenomas. It is used, however, in cases of colorectal cancer to assess the extent of the disease and staging. Rectal ultrasound is not an assessment tool for surveillance of colorectal adenomas. It is neither sensitive nor specific.

Option E: Serum tumor markers such as AFP and CEA are used if the diagnosis is invasive carcinoma. Such markers are used for assessment of the tumor burden as well as monitoring for any post-treatment recurrence.

79

Q

A 55-year-old man presents to your clinic with complaint of recurrent epigastric pain radiating to back. Each episode of pain lasts for 3 to 4 hours. No specific trigger is mentioned, and the pain does not seem to be related to eating or hunger. An ECG is obtained, and is normal. Abdominal ultrasound exam is inconclusive. Laboratory tests, including serum amylase, lipase and troponin, are all within normal parameters. Which one of the following is the most likely diagnosis?

A. Duodenal ulcer.
B. Esophagitis.
C. Acute coronary syndrome.
D. Chronic pancreatitis.
E. Cholecystitis.

A

D. Chronic pancreatitis.

Epigastric pain can have a myriad of etiologies. Cardiac ischemia, peptic ulcer disease, hepatobiliary diseases, pancreatitis, and lower lobe pneumonia can all cause epigastric pain.

Of the options, however, chronic pancreatitis is the most likely diagnosis.

Chronic pancreatitis presents with epigastric pain that typically radiates to back. The pain may or may not be triggered by eating and lasts for several hours. Serum amylase and lipase are often normal, or just slightly elevated. Characteristic finding on imaging is calcification of the pancreas. Ultrasound shows calcifications in only 60 to 70% of patients. 30 to 40% of patients may have normal ultrasound scan results.

Option A: Duodenal ulcer presents with epigastric pain. It can radiate to back. Laboratory findings and ultrasound scan are inconclusive; however, it is worse with hunger and relieved by eating.

Option B: Esophagitis presents with dysphagia (difficulty swallowing) and/or odynophagia (pain on swallowing). Laboratory tests and ultrasound exam findings are unremarkable; however, it is uncommon for the pain to radiate to back. Retrosternal chest pain and heart burn are other features associated with esophagitis.

Option C: A normal ECG makes acute coronary syndrome (ACS) less likely. On the other hand, chronicity of the pain and the episodic nature excludes ACS with high certainty.

Option E: Cholecystitis can presents with pain often aggravated by eating, especially eating fatty foods. The pain can be felt in the right upper quadrant or epigastrium. The pain can radiate to the tip of right shoulder. Furthermore, fever is a common feature. Alkaline phosphates and gamma glutamyl transpeptidase (GGT) are often elevated. Lipase and amylase are normal or only slightly elevated. It is unlikely that sonography misses the diagnosis.

Chronic Pancreatitis

80

Q

Which one of the following is the most common finding on ultrasound examination in patient with right upper quadrant pain?

A. Gall bladder wall thickness and extravasation.
B. Gallstones.
C. Pericholecystic edema.
D. Bile duct dilation.
E. Normal findings.

A

B. Gallstones.

Most RUQ pains in adults are usually due to biliary or hepatic diseases.

Biliary causes of the RUQ pain include:

  • Gallstones
  • Acute cholecystitis
  • Acute cholangitis
  • Dysfunction of the Sphincter of Oddi

Hepatic etiologies include:

  • Hepatitis
  • Perihepatitis
  • Liver abscess
  • Budd-Chiari syndrome
  • Portal vein thrombosis

Gallstones are very common among different populations. They could be asymptomatic and incidentally found during evaluation for another reason, or cause problems such as biliary colic, cholecystitis, choledocholithiasis, and cholangitis.

Of the options, gallstones are the most common ultrasonographic findings in patients with RUQ pain.

Thickened gallbladder wall and extravasation is the diagnostic finding seen in patients with RUQ with cholecystitis. Gallstones, either as the cause, or as an incidental finding may be noted. Pericholecystic edema can be noticed as well.

Bile duct dilation can be seen in patients with RUQ caused by bile duct obstruction.

81

Q

Which one of the following conditions is not associate with dysphagia?

A. Achalasia.
B. Parkinson’s disease.
C. Esophageal varices.
D. Esophageal cancer.
E. Esophageal stricture.

A

C. Esophageal varices.

Dysphagia means difficulty in swallowing and can be classified as three types:

Oropharyngeal dysphagia – difficulty in passing liquids more than solids due to neurologic or muscular problems such as:

  • Parkinsonism
  • Myasthenia gravis – the upper one-third of the esophagus has striated muscle. If myasthenia gravis affects this part, dysphagia ensues.
  • Prolonged intubation
  • Zenker’s diverticulum

Esophageal dysphagia – difficulty in passing solids more than liquids due to local obstruction in the following conditions:

  • Strictures
  • Schatzki ring
  • Webs
  • Carcinoma.

Motility-related dysphagia – difficulty in passing both solids and liquids at the same time, seen in conditions such as:

  • Achalasia
  • Scleroderma

At occasions, there is odynophagia that is painful swallowing. In fact, in odynophagia the difficulty in swallowing is caused by the pain, not by structural abnormalities. Monilial (candida) esophagitis and pill esophagitis are two examples of conditions leading to odynophagia.

Of the given options, the only condition which is not associated with dysphagia (or odynophagia) is esophageal varices.

82

Q

Sarah, 48 years of age, underwent cholecystectomy 6 weeks ago for frequent episodes of biliary colic caused by gallstones. An intra-operative cholangiogram was clear with no left stones evident at that time. Today, she has presented because after being symptom-free for one week after the operation, she has started to have bloating and belching again. Which one of the following best explains her symptoms?

A. Pancreatitis.
B. Cystic duct stump syndrome.
C. Stone left in the common bile duct.
D. Symptoms are not related to gallstones.
E. Post-cholecystectomy syndrome.

A

E. Post-cholecystectomy syndrome.

The dyspepsia manifested by bloating and belching after cholecystectomy is suggestive of post-cholecystectomy syndrome (PCS) as the most likely diagnosis.

PCS affects between 10-15% of patients with cholecystectomy and is characterized by a heterogenous group of symptoms including:

  • Upper abdominal pain
  • Nausea and vomiting
  • Diarrhea
  • Jaundice
  • Bloating
  • Excessive gas
  • Dyspepsia.

These symptoms can be the continuation of symptoms thought to be caused by gallbladder pathology, the development of new symptoms normally attributed to the gallbladder, or symptoms caused by removal of the gallbladder.

In 90% of the time an etiology can be found. The most common etiologies are:

  • Choledocholithiasis – stones remained or formed in the common bile duct or cystic duct remnant
  • Biliary dyskinesia
  • Continuously increased bile flow to the GI tract
  • Dilation of cystic duct remnant

NOTE - choledocholithiasis is the most common cause of PCS. It can involve the common bile duct of the cystic duct remnant. Choledocholithiasis is classified as retained, if found within 2 years of cholecystectomy or recurrent, if the stone is found 2 years after the surgery. Recurrent stones formed as result of the biliary stasis are often caused by strictures, papillary stenosis, and biliary dyskinesia.

NOTE - ultrasonography is the initial imaging study of choice for patients with suspected PCS.

PCS is a provisional diagnosis and the presentation should be renamed after a specific cause as an explanation for the symptoms is clinched.

Option A: The presentation is quite different from acute pancreatitis due to chronicity of symptoms and lack of the typical presentation. Chronic pancreatitis can cause dyspepsia but other features suggestive of the diagnosis are absent. Chronic pancreatitis presents with abdominal pain that can radiate to the back, chronic diarrhea due fat malabsorption and other signs and symptoms caused by pancreatic insufficiency.

Option B: Cystic duct stump syndrome or cystic duct remnant syndrome refers to gastrointestinal symptoms of bloating, dyspepsia, nausea, etc caused by dilation of the remnant cystic duct. It is among the etiologies for PCS. The patient is likely to have PCS either due to this condition or other causes. Unless further studies establish the exact etiology, it cannot be said that cystic duct stump syndrome is the most likely cause to this presentation.

Option C: Retained stones can also be an etiology for PCS and the presentation; however an intra-operative cholangiogram has been clear, making this condition less likely, yet not impossible.

Option D: A clear cholangiogram makes this option correct but again this is not an etiology.

83

Q

Which one of the following is not in favor of achalasia as a diagnosis?

A. Dysphagia to solids.
B. Dysphagia to liquids.
C. Regurgitation of food.
D. Painful swallowing.
E. Nocturnal regurgitation.

A

D. Painful swallowing.

Achalasia is a primary esophageal motility disorder characterized by the absence of esophageal peristalsis and impaired relaxation of the lower esophageal sphincter (LES) in response to swallowing. The LES is hypertensive in about 50% of patients. These abnormalities cause a functional obstruction at the gastroesophageal junction.

Option A and B: Dysphagia for solids (91%) and liquids (85%) are the primary clinical feature of achalasia. Although dysphagia for liquids can occur in patients with other esophageal motility disorders (e.g., progressive systemic sclerosis), this symptom is most characteristic of achalasia and strongly suggests the diagnosis.

Regurgitation of food (option C) is another important feature of achalasia. It can be worse at night (nocturnal regurgitation) (option E) when the patient is recumbent. Patients can find undigested food particles on their pillow in the morning. Regurgitation is a problem in 40-60% of patients with achalasia.

Weight loss in the range of 5-10 kg is common (40-60%). Profound weight loss can also occur. Difficulty belching is another important symptom that is seen in 85% of patients. Chest pain and heart burn can also be present.

Painful swallowing (odynophagia) is not a feature of achalasia and, if present, another diagnosis must be thought of.

84

Q

A 48-year-old man presents to your practice several months after gastrectomy with complaint of dyspepsia. This problem occurs mostly 30 minutes after meals. Dyspepsia is worse when he takes toast and cereal for his breakfast. Which one of the following will be the most important advice to give?

A. He should use more pillows when sleeping.
B. Revision of gastrectomy.
C. Ranitidine.
D. Esomeprazole.
E. Advice dietary changes.

A

E. Advice dietary changes.

With gasterectomy in history, dumping syndrome is the most likely underlying cause to this presentation. Dumping syndrome is the effect of altered gastric reservoir function, abnormal gastric motor function, and/or pyloric emptying mechanism.

In terms of temporal relation to eating, dumping syndrome can have early or late manifestations:

Early dumping
Symptoms of early dumping syndrome occur 30-60 minutes after a meal, and are hypothesized to result from accelerated gastric emptying of hyperosmolar contents into the small bowel. The hyperosmolarity of the bowel content leads to fluid shift from the intravascular compartment into the bowel lumen, resulting in rapid small bowel distention and increase in the frequency of bowel contractions.

With rapid shift of fluid into the intestinal lumen, circulating volume contraction follows, triggering a vasomotor response that presents with tachycardia, lightheadedness, etc

Late dumping
Late dumping occurs 1-3 hours after a meal, probably through development of hyperinsulinemic (reactive) hypoglycemia. Rapid delivery of a meal to the small intestine results in an initial high concentration of carbohydrates in the proximal small bowel and rapid absorption of glucose, hypoglycemia, and a hyperinsulinemic response. The high insulin levels stay for longer period and are responsible for the subsequent hypoglycemia.

The following diagram summarizes the mechanism through which the dumping syndrome develops: (see photo)

The clinical presentation of dumping syndrome can be categorized as abdominal versus vasomotor symptoms, and based on the time of onset as early versus late.

Early dumping systemic symptoms:

  • Desire to lie down
  • Palpitations
  • Fatigue
  • Faintness
  • Syncope
  • Diaphoresis
  • Headache
  • Flushing

Early dumping abdominal symptoms:

  • Epigastric fullness
  • Diarrhea
  • Nausea
  • Abdominal cramps
  • Borborygmi

These symptoms can be collectively referred to as dyspepsia.

Late dumping symptoms:

  • Perspiration
  • Shakiness
  • Difficulty to concentrate
  • Decreased consciousness
  • Hunger

For patients with dumping syndrome, dietary modification is the first-line management option. The following pieces of advice are appropriate:

  • Daily energy intake is divided into 6 meals.
  • Fluid intake during and with meals be restricted and liquids be avoided for at least half an hour after a meal.
  • Avoiding simple sugars.
  • Milk and milk products are generally not tolerated and should be avoided.
  • Because carbohydrate intake is restricted, protein and fat intake should be increased to fulfill energy needs.

Additional points to consider:
Supplementation with dietary fiber has proven effective in the treatment of hypoglycemic episodes. Many medical therapies have been tested, including pectin, guar gum, and glucomannan. These dietary fibers form gels with carbohydrates, resulting in delayed glucose absorption and prolongation of bowel transit time.

Dietary change to a low-carbohydrate, high protein diet, as well as the use of alpha-glucosidase inhibitors, may be useful to control the symptoms of dumping. This is preferential to subtotal or total pancreatectomy in those persons with severe symptoms.

Most patients have relatively mild symptoms and respond well to dietary changes. In some patients with postprandial hypotension, lying supine for 30 minutes after meals may delay gastric emptying and also increase venous return, thereby minimizing the chances of syncope.

RACGP - AFP - The bariatric surgery patient

Gastrointestinal System/Adomen/Abdomen Wall Flashcards by Lubi Mupwaya (18)

85

Q

A 56-year-old man is brought to the Emergency Department with upper gastrointestinal bleeding evident by bright-red hematemesis. He is a known case of alcoholic liver disease. Two endoscopic attempts to stop the bleeding fail. Which one of the following management options should be considered if the least likelihood of hepatic encephalopathy is desired?

A. Octreotide.
B. Repeat endoscopy.
C. Transjugular intrahepatic portosystemic shunt(TIPS).
D. Distal splenocaval shunt.
E. Renocaval shunt.

A

D. Distal splenocaval shunt.

The history of alcoholic liver disease and the bright red bleeding indicates variceal bleeding with high certainty. Varicose esophageal veins are the most common cause of variceal bleeding. Although at occasions gastric varices can be the source.

Currently, endoscopic therapy is the definitive treatment for active variceal bleeding. It can be performed at the same time of the diagnostic endoscopy. Two forms of endoscopic treatment are commonly used:

Sclerotherapy - Sclerotherapy includes injection of a sclerosant solution into the varices through an injection needle that is placed through the endoscope.

Variceal band ligation – The procedure is similar to hemorrhoidal banding and involves placing small elastic bands around varices in the distal 5 cm of the esophagus.

Surgical intervention is another option. Surgical procedures fall into either of these two categories:
1- Shunt surgeries – a shunt is created between portal and systemic venous system
2- Non-shunt surgeries - the two procedures used are (a)esophageal transection and (b) devascularization of gastro-esophageal junction

Shunt surgeries can be categorized as:

Nonselective - those that decompress the entire portal tree and divert all flow away from the portal system, such as portacaval shunts

Selective - those that compartmentalize the portal tree into a decompressed variceal system while maintaining sinusoidal perfusion via a hypertensive superior mesenteric-portal compartment, such as a distal splenorenal shunt, or a distal splenocaval shunt

Partial - those that incompletely decompress the entire portal tree and thereby, maintain some hepatic perfusion

Selective shunting maintains near normal portal flow while decompresses the varicose veins. Since the portal flow is normal the risk of hepatic encephalopathy is minimal compared to TIPS, non-selective, or partial shunting.

Of the options, the only one that can be applied as the next best step in management to both stop the bleeding and decrease the risk of hepatic encephalopathy is splenocaval shunting. Of all patients with selective shunting, only 10-15% develop encephalopathy; however, since retroperitoneal lymphatics are diverted, ascites is a common complication.

NOTE - The routine procedure after endoscopic measures fail is TIPS despite the higher risk of hepatic encephalopathy; however, the question does ask about the procedure to choose with the lowest risk of hepatic encephalopathy.

Attempting a third endoscopy (option B) or medical treatment with octreotide (option A) are not likely to stop the bleeding and are incorrect options.

Renocaval shunting (option E) does not lead to decompression of portal system.

86

Q

A 35-year-old woman is admitted with a hematemesis. On examination she has a blood pressure of 95/60 mmHg and a pulse rate of 100 bpm. She appears reasonably well-nourished, her sclerae are yellow and she has scratch marks and bruises on her arms. Two spider nevi are noted on her chest. Her abdomen is distended and soft with evidence of shifting dullness. Dilated veins are easily visible in the subcutaneous tissue and the liver is palpable three finger-breadths below the costal margin. Her hemoglobin is 85g/L (115-165), serum albumin 28g/L (32-45) and the serum alanine aminotransferase 50U/L (< 35). Which one of her clinical and biochemical findings is likely to have the greatest impact on her long-term prognosis?

A. Her blood pressure on admission.
B. The serum albumin concentration.
C. The number of spider nevi.
D. The size of the liver on admission.
E. The serum aminotransferases concentration.

A

B. The serum albumin concentration.

The clinical picture is suggestive of ascites in the background of chronic liver disease. The Child-Pugh system is used to assess the prognosis of patients with liver disease and cirrhosis by calculating 1-year survival rate. This system takes into account the following 5 parameters for determination of the prognosis:
1. The presence of ascites
2. Bilirubin level
3. Albumin level
4. Prothrombin time
5. Encephalopathy

The Child-Pugh score is then calculated according to the following table: (see photo)

Based on the Child-Puch score, the patients are categorized as follows:

  • Class A (scores 5-6): well-compensated disease – one- and two-year survival are 100% and 85% respectively.
  • Class B (scores 7-9): functional compromise – one- and two-year survival are 80% and 60% respectively.
  • Class C (scores 10-15): decompensated disease - one- and two-year survival are 45% and 35% respectively.

Of the given options, the only one contributing to prognosis is serum albumin concentration.

Gastrointestinal System/Adomen/Abdomen Wall Flashcards by Lubi Mupwaya (19)

87

Q

A 34-year-old woman presents to your clinic complaining of abdominal pain and diarrhea one week after she returned from a trip to Thailand. While she was on the trip, she first noticed abdominal pain in the right iliac fossa which resolved subsequently. On examination, the abdomen is non-tender and soft with no rigidity or guarding. However, digital rectal exam is tender. Which one of the following is the most likely diagnosis?

A. Giardiasis.
B. Celiac disease.
C. Rota virus infection.
D. Urinary tract infection.
E. Appendicitis.

A

E. Appendicitis.

Although giardiasis, viral gastroenteritis and celiac disease can all present with diarrhea and abdominal pain, NO rectal tenderness on is elicited on rectal exam. Urinary tract infection is not associated with tenderness on rectal exam either.

Of the given options, the only one that can justify the clinical presentation is acute pelvic appendicitis.

Clinical features of pelvic appendicitis are:

  • Absence of abdominal wall rigidity and tenderness
  • Tenderness in the rectovesical pouch and/or pouch of Douglas on rectal examination
  • Right-sided spasm of psoas muscle
  • Diarrhea due to irritation of the rectum by the inflamed appendix
  • Increased frequency of urination caused by irritation of bladder due to an inflamed appendix
  • Hypogastric pain brought on by internal rotation of a flexed hip due to contact of the inflamed appendix with the obturator internus muscle
Gastrointestinal System/Adomen/Abdomen Wall Flashcards by Lubi Mupwaya (20)

88

Q

A 71-year-old man presents with progressive jaundice, pale stool and dark urine. On examination, a mass is palpated in the right upper quadrant that moves with respiration. Ultrasonography shows a dilated common bile duct (CBD) and no masses in the head of the pancreas. Which one of the following could be the most likely diagnosis?

A. Chronic pancreatitis.
B. Carcinoma of the tail of pancreas.
C. Peri-ampullary tumor.
D. Biliary cirrhosis.
E. Budd-Chiari syndrome.

A

C. Peri-ampullary tumor.

The clinical picture is highly suggestive of common bile duct obstruction (CBD). CBD obstruction presents with obstructive jaundice and often a palpable mass (distended gallbladder) in the right upper quadrant that can be tender or non-tender depending on the underlying etiology. The following are the most common causes of CBD obstruction:

  • Stones – the most common cause
  • Strictures
  • Periampullary tumors – these tumors arise within 2cm of the ampula of Vater in the duodenum and include the following:
    Pancreatic head / uncinate process tumors: includes pancreatic ductal adenocarcinoma involving head and uncinate process of the pancreas.
    Lower common bile duct tumors: includes types of cholangiocarcinoma involving the intra-pancreatic distal bile duct.
    Ampullary tumors: those originating from the ampula of Vater.
    Periampullary duodenal carcinoma.

The pancreatic head tumors are the most common periampullary tumors, but not in this case, as sonography shows that the pancreatic head is clear; therefore, other types of periampullary tumors should be considered as the most likely diagnosis.

Option A: Chronic pancreatitis presents with abdominal pain, malabsorption and diarrhea. Obstructive jaundice is not a presentation.

Option B: Anatomically, tumors of pancreatic tail are far from biliary system and do not cause biliary obstruction.

Option D: Biliary cirrhosis does not cause CBD obstruction.

Option E: Budd-Chiari syndrome is thrombotic occlusion of hepatic veins, presenting with a different clinical picture.

89

Q

Which one of the following complications of acute diverticulitis carries the highest mortality rate?

A. Bleeding
B. Abscess formation.
C. Peritonitis.
D. Perforation.
E. Intestinal obstruction.

A

D. Perforation.

Diverticular disease carries a number of potential complications including:

  • Bleeding, especially in the elderly
  • Bowel perforation
  • Intra-abdominal abscess formation
  • Peritonitis
  • Fistula formation
  • Intestinal obstruction

Rupture of an inflamed diverticulum with fecal contamination of the peritoneum occurs in only 1 to 2% of cases but is associated with a 20% mortality rate, which is the highest when compared to other complications of diverticulitis.

Perforation of diverticula into the abdominal cavity presents with the following features:

  • Abdominal distention
  • Diffuse tenderness of the abdomen even to light palpation.
  • Guarding
  • Rigidity
  • Rebound tenderness
  • Absent bowel sounds

90

Q

Which one of the following is correct about hereditary hemochromatosis?

A. Venepuncture should not be performed when the patient is not symptomatic.
B. The incidence is around 1/500,000.
C. Ninety percent of patients are hom*ozygous for C282Y.
D. The incidence is more common among Asians.
E. The life expectancy is reduced regardless of hepatic involvement.

A

C. Ninety percent of patients are hom*ozygous for C282Y.

Hereditary haemochromatosis (HH) affects 1:200-300 people in Australia. Of the affected persons, approximately 90% are C282Y hom*ozygotes.

HH is much more prevalent among those of Northern European descent and is rare in Asians and Africans. Screening of the population at risk is strongly recommended to identify those with the disease, who are not still clinically symptomatic for early intervention and preventing the progression of disease; therefore, venepuncture as the first-line therapy should be started in those with abnormal iron studies but no symptoms yet.

The most common cause of mortality in HH is liver involvement and liver cirrhosis. If treatment is started before liver is involved, those with HH can expect a normal lifespan.

Hereditary haemochromatosis

91

Q

Which one of the following is the most common cause of acute bile duct obstruction in tertiary care hospitals?

A. Choledocholithiasis.
B. Benign strictures.
C. Tumors.
D. Post-biliary access / manipulation by ERCP or PTC.
E. Acute Cholecystitis.

A

D. Post-biliary access / manipulation by ERCP or PTC.

The most common cause of bile duct obstruction in community is choledocholithiasis. Gallstones contribute to approximately 70% of cases presenting with biliary tree obstruction. Benign strictures and tumors are responsible for 15% of cases with obstructed bile passage. However, it should be noted that the question does not ask about the most contributing factor to obstruction in tertiary hospitals rather than the community.

The most common cause of bile duct obstruction in tertiary hospitals is biliary access/manipulation by endoscopic retrograde cholangiopancreatography (ERCP) or percutaneous transhepatic cholangiography (PTC).

92

Q

Which one of the following is a sign of pyloric stenosis due to peptic ulcer?

A. Vomiting within the first 1 hour of eating.
B. Vomiting immediately after eating.
C. Vomiting after 2 hours of eating.
D. Regurgitation.
E. Epigastric pain radiating to back.

A

A. Vomiting within the first 1 hour of eating.

Pyloric obstruction also known as gastric outlet obstruction (GOO) is the consequence of any disease producing a mechanical barrier to gastric emptying. Clinical entities that can cause GOO are generally categorized into benign and malignant.

Peptic ulcer disease (PUD) is among the benign causes of GOO. The incidence of ulcer-induced GOO has dramatically declined owing to the adequate and efficient treatment of PUD. The mechanism of obstruction by PUD can be either edema around the ulcer or scar formation after the ulcer heals.

The most common symptoms of GOO, regardless of the underlying etiology, are bloating, anorexia, nausea and vomiting. Vomiting is usually nonbilious, and characteristically contains undigested food particles.

Patients with gastric outlet obstruction from a duodenal ulcer or incomplete obstruction typically present with symptoms of gastric retention, including early satiety, bloating or epigastric fullness, indigestion, anorexia, nausea, vomiting, epigastric pain, and weight loss. They are frequently malnourished and dehydrated and have metabolic insufficiency. Weight loss is frequent when the condition is chronic and is most significant in patients with malignant disease. Abdominal pain is not frequent and usually relates to the underlying cause, e.g., PUD, pancreatic cancer.

The time of vomiting can suggest the site of obstruction. In pyloric obstruction (more proximal) the time of vomitus is usually within the first hour of eating, whereas in pyloric stenosis, or duodenal stenosis or obstruction (more distal) the vomiting occurs after one hour because normally it takes 1 hour for the food to reach the duodenum. Within 2-4 hours of eating the food is in the small intestine. Vomiting after 2-4 hours should raise suspicion against another cause.

Gastric Outlet Obstruction

93

Q

A 65-year-old woman presents to your practice because she has been noting streaks of blood on her stool on different occasions in the past week. These occasions were preceded by a period of constipation. The only medication she is currently on is panadeine for a back pain started 3 weeks ago. She is otherwise healthy. Which one of the following would be the most likely diagnosis?

A. Ulcerative rectal cancer.
B. Colorectal cancer metastasizing to the lumbar spine.
C. Internal hemorrhoid.
D. External hemorrhoid.
E. Ulcerative colitis.

A

C. Internal hemorrhoid.

Blood on stool can be caused by several conditions, some being benign and others malignant. Colorectal cancer, particularly if arising from rectum, can cause blood covering the stool. The blood may be bright or dark red, depending on the site of the tumor. The darker the blood, the more proximal the tumor. Proximal colon cancers tend to present with lethargy and fatigue rather than blood-stained stool because blood mixes with the stool and is barely visible. Rectal bleeding may be the only symptom of colorectal cancer; however, weight loss, abdominal pain or discomfort, bloating, anorexia, and other symptoms may be seen.

By the time a colorectal is clinically evident, it has often already metastasized to the liver (the most common site) and lymph nodes most of the time. Lungs and bones are rarely involved several months after the tumor has metastasized to the liver and/or lymph nodes.

Internal hemorrhoids are one of the most common causes of benign rectal bleeding. Patients may notice blood covering the stool, as streaks on the stool, dripping in the toilet bowl, or staining the toilet paper. The most common predisposing factor for development of an internal hemorrhoid is chronic constipation. This woman has been on panadeine (paracetamol 500mg + codeine 8 mg). This can justify the constipation and internal hemorrhoid as the most likely cause of this presentation.

Ulcerative rectal cancer (option A) or colorectal cancer (option B) in general, can cause rectal bleeding (often painless), but a rectal tumor so large to cause constipation is expected to be associated with more pronounced presentation including weight loss, anemia, fatigue, or decreased stool caliber.

Ulcerative colitis (option E) is often diagnosed at an earlier age is associated with other manifestations such as bloody diarrhea, weight loss, join pain, uveitis, etc. With bloody stool as the mere complaint in a 65-year-old woman, ulcerative colitis is an unlikely diagnosis.

As a general rule external hemorrhoids (option D) do not bleed but are painful; therefore, less likely to be the diagnosis.

NOTE - Although internal hemorrhoid is the most likely diagnosis, colorectal cancer should be excluded by thorough investigation.

94

Q

A 45-year-old man presents with complaints of polyarthritis, impotence and decreased libido. Which one of the following investigations is the most appropriate step to consider?

A. Iron studies.
B. Fasting blood sugar.
C. HFE gene testing.
D. CT scan of the head.
E. Prolactin level.

A

A. Iron studies.

It is very important to consider hemochromatosis in patients with decreased libido and manifestations related to other sites of the body such as joints, liver, CNS, etc.

Hemochromatosis, through iron overload and deposition of iron in several organ systems, can present with the following features:

  • Chronic hepatitis and cirrhosis (the most common cause of mortality).
  • Abdominal pain.
  • Arthralgia – often MCPs and large joints, due to chondrocalcinosis and pseudogout. The pain is like that of osteoarthritis.
  • Skin darkening (tanned skin) – deposition of iron in the skin.
  • Small testes, infertility, impotence, and decreased libido –small testes are due to hypopituitarism and/or liver disease. Iron deposition in gonads occurs but is not the cause hypogonadism.
  • Damage to the pancreas and diabetes mellitus (known as bronze diabetes).
  • Restrictive cardiomyopathy and ensued congestive heart failure (15% of patients). Other less common cardiac manifestations are supraventricular tachycardias, atrial fibrillation, atrial flutter and varying degrees of atrioventricular block.
  • Accumulation of iron in the pituitary and panhypopituitarism.
  • Hepatocellular carcinoma (hepatoma) – in 10% of cases with liver involvement.
  • Osteoporosis (25% of cases) and osteopenia (41% of case)
  • Sparse body hair especially pubic hair (62% of patients)
  • Spoon nails (50% of cases)

NOTE - the most common cause of death from haemochromatosis is liver cirrhosis, followed by cardiac disease, which affects approximately 15% of the patients.

When hemochromatosis is suspected clinically, iron studies are the most appropriate next step to make a diagnosis.

The transferrin saturation (ratio of serum iron to iron binding capacity) reflects increased absorption of iron, which is the underlying biological defect in hereditary hemochromatosis (HH). A fasting transferrin saturation >45% is the most sensitive test for detecting early iron overload, but not diagnostic of HH. Ferritin can be used to assess iron overload, but it is not as accurate because it is an acute phase reactant and may be elevated in response to several physiologic stresses, alcohol consumption, and liver disease. Serum ferritin is abnormal when it is >250 μg/L in pre-menopausal women and >300 μg/L in men and post-menopausal women.

If fasting transferrin saturation or serum ferritin is increased on more than one occasion, HH should be suspected, even if there are no clinical symptoms or abnormal liver function tests (LFT). In this situation, the HFE gene test (option C) should be considered as the next and also most diagnostic tool.

Option B: Fasting blood sugar is a good step for this patient because diabetes is a possible feature with hemochromatosis; however, it is not diagnostic for the condition.

Option D: CT scan of the head adds no diagnostic value in this patient.

Option E: Although hemochromatosis can cause panhypopituitarism and decreased pituitary hormone levels, measurement of prolactin level alone does not add any diagnostic value.

95

Q

Which one of the following is the most common cardiac manifestation of hemochromatosis?

A. Supra ventricular tachyacardias.
B. Congestive heart failure.
C. Atrial fibrillation.
D. Atrial flutter.
E. Atrioventricular (AV) block.

A

B. Congestive heart failure.

The most common cardiac manifestation of hemochromatosis is congestive heart failure. The underlying pathology is deposition of iron in the myocardium leading to restrictive cardiomyopathy. Other manifestations are supraventricular tachycardias , conductive disorders such as AV block, and atrial fibrillation and atrial flutter. However, these are not as common as congestive heart failure.

Hemochromatosis

96

Q

Ten years ago, a 75-year-old man underwent a successful right hemicolectomy for colon cancer, followed by chemotherapy after the cancer was found to be Duke C stage. Now, he has presented for surveillance. He has no specific complaint and the physical examination is completely normal. Which one of the following would be the investigation of choice for him?

A. Abdominal CT scan.
B. Colonoscopy.
C. Liver function tests (LFT), renal function tests (RFT) and full blood exam (FBE).
D. Carcinoembryonic antigen(CEA).
E. Sigmoidoscopy.

A

B. Colonoscopy.

Colonoscopy should be performed one year after the resection of a sporadic cancer, unless a complete post-operative colonoscopy has been performed sooner. Recommendations for familial adenomatous polyposis (FAP) and hereditary nonpolyposis colorectal cancers (HNPCC) are different from sporadic cancers.

If a peri-operative colonoscopy performed at one year reveals advanced adenoma, next colonoscopy should be performed in 3 years. If the colonoscopy performed at one year is normal or identifies no advanced adenomas, the next colonoscopy should be performed in 5 years. Since this man has not have a colonoscopy in the past 10 years, he should undergo colonoscopy now.

Apart from colonoscopy according to the above recommendations, patients undergoing either local excision (including transanal endoscopic microsurgery) of rectal cancer or advanced adenomas or ultra-low anterior resection for rectal cancer should be considered for periodic examination of the rectum at 6-month intervals for 2-3 years with digital rectal examination, rigid proctoscopy, flexible proctoscopy, and/or rectal endoscopic ultrasound. These examinations are considered to be independent of the colonoscopic examination schedule described above.

Other tests that may be used during this period for early detection of metastases may include:

  • Chest X-ray
  • CEA (carcinoembryonic antigen) – diagnostically nonspecific but useful for monitoring recurrence.
  • PET scan (distant metastases will light up on PET scan)
  • Pelvic/abdominal CT scan

Option A and B: Abdominal CT scan and CEA are acceptable options for surveillance but not as crucial as colonoscopy.

Option E: Sigmoidoscopy cannot visualize beyond the sigmoid colon. Patients with the history of colon cancer need to have visualization of their entire colon for possible tumors or premalignant lesions by colonoscopy.

97

Q

A 72-year-old man presents to the emergency department with complaint of perianal pain for the past 2 days. His anal area is illustrated in the accompanying photograph. Which one of the following is the most likely diagnosis?

A. Thrombosed internal hemorrhoid.
B. Thrombosed external hemorrhoid.
C. Rectal carcinoma.
D. Crohn’s disease of the anus.
E. Perianal abscess.

Gastrointestinal System/Adomen/Abdomen Wall Flashcards by Lubi Mupwaya (21)

A

A. Thrombosed internal hemorrhoid.

Traditionally, hemorrhoids are classified as internal and external; however, some authors believe that since these two have different origins and mechanisms of development, they are better termed hemorrhoids (instead of internal hemorrhoids) and perianal hematoma (instead of external hemorrhoids). Perianal hematoma and external hemorrhoids are often interchangeably used.

Hemorrhoid (internal hemorrhoid):

The anus is mainly lined by discontinuous masses of spongy vascular tissue termed anal cushions, which contribute to anal closure and differentiating flatus from stool. Viewed from the lithotomy position, these cushions are located at 3, 7, and 11 o’clock. These cushions are attached together and to the surrounding structures by supporting fibromascular tissue. Hemorrhoids occur when these structures become bulky and protruded due to gravity, straining, or increased tone of anal sphincter (unlike the common belief, hemorrhoid is not a varicose anal vein).

Bulky and protruded cushion are at risk of trauma from hard stool and bright red bleeding from the capillaries of the underlying lamina propria.

Constipation and straining are the most common causes of hemorrhoids; however, bowel habit is normal in many patients. Congestion from a pelvic tumor, pregnancy, congestive heart failure, nephrosis, or portal hypertension plays a role in only a minority of patients.

Hemorrhoids are classified as following:

  • 1st degree – remains in the rectum.
  • 2nd degree – prolapses through the anus on defecation but reduces spontaneously afterwards.
  • 3rd degree – like 2nd degree but needs digital reduction.
  • 4th degree – remains prolapsed persistently.

Hemorrhoids are painless unless acute thrombosis superimposes, in which case it may become painful. Thrombosed hemorrhoids are managed conservatively with analgesics, stool softeners, bed rest (elevation of the bed foot) and ice packs for 2- 3 weeks. The drawback of this method is the long time off work. Hemorrhoidectomy is the second option if conservative management is not an option for any reason.

Perianal hematoma (external hemorrhoid)

Perianal hematoma is not hemorrhoid; however, it is usually called an external hemorrhoid. It presents as a painful tense blue swelling at the anal verge caused by a recent thrombosis of an anal vein, often after straining at stool.

The picture in the question shows a fleshy red lesion protruding out of the anus consistent with an (internal) hemorrhoid. Presence of pain suggests acute thrombosis.

Option B: External hemorrhoid has a different appearance.

Option C: Although hemorrhoids can be associated with rectal carcinoma, the lesion itself is not a carcinoma. Moreover, a carcinoma this large would have been associated with more pronounced systemic and local symptoms.

Option D: It is important to note that for every anorectal lesion, thorough investigation should be conducted in an attempt to exclude serious underlying pathologies such as inflammatory bowel disease or malignancies.

Option E: Perianal abscesses present with painful and tender red perianal swelling and induration, not a lesion protruding out of the anus.

Hemorrhoids

Gastrointestinal System/Adomen/Abdomen Wall Flashcards by Lubi Mupwaya (2024)
Top Articles
Latest Posts
Article information

Author: Otha Schamberger

Last Updated:

Views: 6465

Rating: 4.4 / 5 (75 voted)

Reviews: 90% of readers found this page helpful

Author information

Name: Otha Schamberger

Birthday: 1999-08-15

Address: Suite 490 606 Hammes Ferry, Carterhaven, IL 62290

Phone: +8557035444877

Job: Forward IT Agent

Hobby: Fishing, Flying, Jewelry making, Digital arts, Sand art, Parkour, tabletop games

Introduction: My name is Otha Schamberger, I am a vast, good, healthy, cheerful, energetic, gorgeous, magnificent person who loves writing and wants to share my knowledge and understanding with you.